Vous êtes sur la page 1sur 91

Primary Care Medicine:Question 1

A 77-year-old man is found to have asymptomatic atrial fibrillation on a routine


examination. His medical history includes hypertension, hypercholesterolemia,
diabetes mellitus, and osteoarthritis, but he has no history of heart disease. His
medications include atenolol and simvastatin.
His weighs 72 kg (159 Ib), blood pressure is 138/76 mm Hg, and resting ventricular
rate is 78/mm. On examination, his jugular venous pressure is less than 5 cm H2O.
His lungs are clear. There are no cardiac murmurs or gallops. There is no edema.
Neurologic evaluation, including mental status examination and gait, are normal. An
echocardiogram shows no wall motion abnormalities and the ejection fraction is
45%. Thyroid studies are normal.
What is the most appropriate next step in the management of this patient?
A. Initiation of therapy with warfarin, 5 mg/d
B. Referral for cardioversion and maintenance with low-dose amiodarone therapy.
C. Initiation of therapy with aspirin, 325 mg/d.
D. Three days of warfarin, 10 mg/d, followed by 5 mg/d.
E. Simultaneous initiation of therapy with low-molecular-weight heparin and
warfarin, 5 mg/d.
Primary Care Medicine:Question 2
A 20-year-old woman presents to your clinic for an initial visit. Her primary concern
is her weight. She has always been heavier than she would like, but she has gained
almost 13.5 kg (30 Ib) since starting college 2 years ago. She has a diet rich in
convenience and snack foods. She has several alcoholic beverages on most
weekends, but does not consider alcohol to be a problem for her socially. She has
never smoked cigarettes. She used to play soccer competitively, but finds little time
for regular physical activity. Her father is overweight and has type 2 diabetes
mellitus and hypertension. The patient does not have polyuria, polydipsia, blurred
vision, changes in her skin or hair, changes in her regular menses, or cold
intolerance.
On physical examination, she is 167.5 cm (66 in) tall and weighs 99.5 kg (219 Ib);
her body mass index (BMI) is 35. Her blood pressure is 148/92 mm Hg.
What would be the most appropriate management of obesity in this young woman?
A. Sibutramine, 10 mg/d, plus a protein-rich, carbohydrate-restricted diet
B. Orlistat, 120 mg with meals, plus a calorie-restricted diet with a calorie deficit of
500 kcal/d, less than 30% of calories from fat, and limited refined sugars
C. An exercise prescription for 30 minutes of brisk walking five or more times a
week, progressing to 45 to 60 minutes of vigorous exercise on most days
D. A calorie-restricted diet with a calorie deficit of 500 kcal/d, less than 30% of
calories from fat, and limited refined sugars, combined with a progressive exercise
program
E. Base recommendations on the results of laboratory testing, including serum
thyroid stimulating hormone, fasting lipid panel, and determination of fasting serum
glucose
Primary Care Med icine:Question 3
A 23-year-old woman presents with a 1-hour history of hematemesis. She became ill
at a wedding reception and vomited once with a large amount of blood. On
examination, she has a pulse rate of 100/min and a blood pressure of 100/60 mm
Hg; her abdomen is soft and nontender. The serum sodium is 136 meq/L, potassium
2.8 meq/L, chloride 86 meq/L, bicarbonate 34 meq/L, blood urea nitrogen 20 mg/dL,
and serum creatinine 1 .0 mg/dL. Endoscopy reveals erosive esophagitis without
active bleeding and a fresh Mallory-Weiss tear with an adherent clot.

What is her most likely diagnosis?


A. Staphylococcus aureus toxin ingestion
B. Bacillus cereus toxin ingestion
C. Scleroderma
D. Bulimia
E. NSAID-induced esophagitis
Primary Care Medicine:Question 4
A 56-year-old previously healthy woman reports difficulty swallowing over the past 2
months. The difficulty occurs several seconds after attempting to swallow solid food,
with a feeling of food getting stuck at the suprasternal notch. She has never had
regurgitation of acid or food, or other symptoms of heartburn. She has not lost
weight, and has no nasal regurgitation or history of choking. Physical examination is
normal; the hematocrit is 40%.
What is the next step in managing the patients symptoms?
A. Barium swallow
B. Upper endoscopy
C. Esophageal manometry
D. Surgical myotomy
E. Calcium channel blocker
Primary Care Medicine:Question 5
A 55-year-old woman was recently found to have mild hypertension; her blood
pressure has ranged from 140-151/90-96 mm Hg. She would like to avoid taking
medication and asks what else she can do to lower her blood pressure.
Which of the following lifestyle changes would most effectively lower this patients
blood pressure?
A. Biking vigorously for 30 minutes 3 days per week
B. Walking 1 mile 4 days per week
C. Sodium restriction to 6 g/d
D. Potassium supplementation of 10 meq/d
E. Calcium supplementation, 1000 mg/d
Primary Care Medicine:Question 6
A 33-year-old man comes to a clinic for evaluation of a rash. He has erythema on his
face with scale in the nasolabial folds and at the hairline. He has erythema in both
axillae and erythema with scale on the central chest where he has abundant chest
hair. He has tried nonprescription hydrocortisone cream on his face without much
benefit. What is the most appropriate next step in the management of this patient?
A. Risk factor assessment for HIV disease
B. Skin biopsy
C. Ketoconazole, 2% shampoo
D. Hydrocortisone, 1% cream, and ketoconazole, 2% cream
E. Coal tar shampoo
Primary Care Medicine:Question 7
A healthy 43-year-old woman presents as a new patient. Her only chronic medical
problem is hypertension. She works as a flight attendant and lives with a roommate.
She does not smoke; she drinks alcohol in moderation. She has never been sexually
active and has not been assaulted. Her father developed colon cancer at age 75
years, and a maternal aunt developed ovarian cancer in her 60s. The patient has
never had a Pap test and wonders whether she is at risk for ovarian cancer. Her
most recent mammogram was 2 years ago.

What is the most appropriate screening test to recommend to this patient?


A. Papanicolau smear
B. Mammography
C. Colonoscopy
D. CA-125 testing
E. Pelvic ultrasonography
Primary Care Med icine:Question 8
A 57-year-old woman is evaluated for constipation of 2 months duration. She
describes excessive straining at defecation and hard stools. She has tried stool
softeners with only modest improvement. Her medical history is significant for mild
edema managed with hydrochlorothiazide, 25 mg/d. A screening colonoscopy 3
years ago was normal.
On physical examination, her blood pressure is 126/80 mm Hg. The abdomen is
nondistended, with normal active bowel sounds and no tenderness or masses. The
rectal examination shows normal sphincter tone, no rectal masses, and hard brown
stool that is negative for occult blood. A plain radiograph of the abdomen shows stool
in the splenic flexor of the large bowel, but otherwise is normal. Which of the
following is the most appropriate next step in the evaluation of this patient?
A. Prescribe daily stimulant laxatives.
B. Replace hydrochlorothiazide with a nondiuretic antihypertensive medication.
C. Repeat colonoscopy.
D. Order a colonic transit study.
E. Start therapy with sorbitol.
Primary Care Medicine:Question 9
A 72-year-old man is evaluated in clinic for progressive decline in libido and an
inability to maintain an erection sufficient for intercourse. His medical history
includes diabetes mellitus, peripheral vascular disease, and coronary artery disease
with stable angina. His medications include aspirin, atenolol, isosorbide, and
glipizide. He is married and relates feeling depressed by his medical problems and
his inability to perform sexually, but denies problems with sleep, appetite, or
anhedonia. On physical examination, vital signs are normal, as are heart and lung
examination; lower extremity pulses are diminished with sluggish capillary refill, and
hair is sparse below the knee. Testicles are normal in size and consistency, and the
serum testosterone level is 400 ng/dL.
Which of the following is the most appropriate intervention to treat this patients
erectile dysfunction? A. Testosterone replacement therapy
B. Sildenafil therapy
C. Antidepressant therapy
D. lntra-urethral aiprostadil therapy
E. Yohimbine therapy
Primary Care Medicine:Question 10
A 46-year-old woman presents with a right breast lump that she noticed 3 weeks ago
while bathing. She assumed it was a breast cyst, but with onset of menses 1 week
ago the lump did not get smaller. She is healthy, G2P2, with menarche at age 13
years and first delivery at age 25 years; she breastfed each of her children for a
year. Family history is negative for breast or ovarian cancer. She drinks 2 cups of
coffee each morning. On examination, she appears anxious but well. Breasts are
normal on inspection, without retraction. On palpation, there is a firm 8-mm lump
distinct in the right upper outer quadrant, minimally tender, with limited mobility.
There are no other dominant masses and no axillary adenopathy. Bilateral

mammogram performed the same day is normal. Breast ultrasonography is not


available. What is the most appropriate next step in the management of this patient?
A. Reassure her that this is not cancer because of the normal mammogram and that
no further evaluation is needed.
B. Reassure her that this is not likely cancer because of the normal mammogram,
but she should return for repeat clinical breast examination in 3 months.
C. Advise her that this is likely a cyst, that she should stop caffeine, and that she
should return for repeat clinical breast examination in 3 months.
D. Advise her that this lump needs further evaluation and arrange additional
mammographic imaging. E. Advise her that this lump needs further evaluation and
arrange fine-needle aspiration.
Primary Care Medicine:Question 11
During a routine physical examination, a healthy 65-year-old man is found to have a
loud, harsh, systolic murmur heard maximally in the second intercostal space that is
also heard at the apex and radiates to the carotids. The cardiac examination is
otherwise normal, with normal carotid upstrokes. The patient has no history of chest
pain or tightness, shortness of breath, dizziness, syncope, or other cardiac
symptoms. He has no history of cardiac disease and has never had an
echocardiogram. You consult a textbook and learn that the positive likelihood ratio
for such a murmur is 1 .8 in predicting severe aortic stenosis.
Based upon this information, what do you tell the patient?
A. He likely has an innocent flow murmur that warrants no further evaluation.
B. He likely has severe narrowing of his aortic valve.
C. He has a narrowing of his aortic valve with a 40% chance that it is severe.
D. He may have a narrowing of his aortic valve but the chance that it is severe is
low.
Primary Care Med icine:Question 12
A 37-year-old white woman presents as a new patient. She is in good health. She
wonders, however, whether she should be tested for the breast cancer gene because
her 45-year-old sister was just diagnosed with invasive breast cancer. She has no
personal history of breast problems, and has not yet had a mammogram. She has
menarche at age 13 years, and has no children.
Which of the following statements about genetic susceptibility testing for BRCA1 or
BRCA2 in this patient is most accurate?
A. She has a 50% chance of testing positive for a BRCA1 or a BRCA2 mutation and
should be tested. B. This patients risk of breast cancer by age 40 years is 30%;
mammography is indicated regardless of testing results.
C. A positive test for BRCA1 or BRCA2 would have no impact on decision making
about screening mammography, so testing is not indicated.
D. Knowing the result of BRCA1/BRCA2 testing on her sister would be helpful in
decision making for this patient.
Primary Care Medicine:Question 13
A 61-year-old man is evaluated for routine follow-up of type 2 diabetes mellitus and
hypertension. He feels well but has noted a decrease in vision without diplopia.
Examination shows a pulse rate of 82/min and blood pressure of 174/98 mm Hg;
visual acuity is 20/30 on the left, and 20/50 on the right with usual lenses.
Extraocular muscle function is normal. Funduscopic examination shows a few
scattered areas of exudates and a few dot-blot hemorrhages bilaterally, and right
fundus showing tree-like projections off vessels near the disk. The hemoglobin A1c is
10%.

Which of the following is the most appropriate next step in the patients
management?
A. Aggressive blood pressure control for hypertensive retinopathy
B. Immediate ophthalmology referral for laser therapy for macular edema
C. Immediate ophthalmology referral for laser therapy for proliferative retinopathy
D. Routine ophthalmology referral within the next several months for background
diabetic retinopathy
E. Rapid aggressive blood glucose control to a target hemoglobin A1c of 7.0%
Primary Care Med icine:Question 14
A 52-year-old man presents as a new patient requesting an annual examination and
a serum prostate-specific antigen (PSA) measurement. He exercises, does not
smoke, and drinks alcohol infrequently. His medical records disclose well-controlled
blood pressure, a negative exercise tolerance test 4 years ago, and various
laboratory evaluations, all of which were normal. His only medication is atenolol
Previous health maintenance included a colonoscopy 2 years ago and serum PSA
values of 4.2 ng/mL 1 year ago and 3.8 ng/mL 2 years ago. The patient recalls his
prior physician telling him the value was borderline and recommending a repeat
measurement in 1 year.
Physical examination, including digital rectal examination, is unremarkable. Serum
PSA value is 5.0 ng/mL.
Which of the following is the most appropriate next step in the management of this
patient?
A. Prescribe tamsulosin therapy
B. Perform serial PSA measurements
C. Order free PSA assay
D. Refer for urologic evaluation
E. Prescribe finasteride
Primary Care Medicine:Question 15
A 60-year-old man is evaluated for painful leg ulcers. He has a history of
degenerative joint disease, obesity, hypertension, and chronic venous stasis. His
medications are acetaminophen and atenolol.
On examination, he has markedly edematous lower legs, with brawny skin
discoloration, several areas of dermal atrophy from lipodermatosclerosis, and two
shallow, tender, red-based, ulcers weeping clear exudate with some associated
crusting over the medial ankle and lower shin on the left leg. He has been using
graded compression hose over dry gauze dressings.
Which of the following is the best strategy for decreasing the patients leg ulcer pain?
A. Nonadherent dressings
B. Occlusive dressings
C. Zinc paste-impregnated bandage (Unna boot)
D. Vaseline gauze
E. Topical antibiotic ointment
Primary Care Medicine:Question 16
A 31-year-old woman seeks advice about acne. She has had acne since menarche
but continues to have cyclical lesions that embarrass her. On examination, she has
pustules on her face, back, and chest, with two cysts on her chin and forehead. She
has noticeable scarring that is well concealed with make-up. She has taken oral
contraceptives for about 5 years, without resolution of her cysts.

What is the most important factor for choosing further treatment for this patient?
A. Whether open comedones are present
B. Her willingness to avoid pregnancy
C. Her alcohol intake
D. Her plasma triglyceride levels
E. Whether she can avoid sun exposure
Primary Care Medicine:Question 17
A 26-year-old woman presents with a 3-week history of severe anal pain. She has
noticed some blood on the toilet paper and has had a 3.6-kg (8-Ib) weight loss in the
past 2 months. On physical examination, an anal fissure is present in the 3 oclock
position. The fissure bleeds minimally when palpated.
What is the best next step in the management of this patient?
A. Anoscopy
B. Colonoscopy
C. Trial of psyllium husk (MetamucilTM)
D. Trial of nitroglycerin ointment
E. Trial of botulinum toxin injections
Primary Care Medicine:Question 18
A 48-year-old man with a long history of alcohol abuse presents for ongoing care
after an inpatient stay for alcohol withdrawal. This is his third episode of acute
detoxification, with the longest period of abstinence being 4 months. He denies any
current symptoms of depression and states that he is not using any illicit drugs or
prescription medications. His last alcohol intake was 2 weeks before his visit to your
office, and he is now enrolled in an alcohol treatment program.
Which of the following pharmacologic agents would be the best adjunct to his
treatment?
A. Naltrexone
B. Paroxetine
C. Diazepam
D. Buspirone
E. Disulfiram
Primary Care Medicine:Question 19
A 35-year-old woman comes to your office to discuss family planning. She has been
in a monogamous relationship with her current partner for 9 years, and they have
both been tested for HIV infection and are negative. She hopes to become pregnant
and plans to discontinue using condoms. Her most recent Pap test was about 1 year
ago and was normal. She has had annual Pap tests for the past 15 years that have
all been normal.
When does she need to have her next Pap test?
A. Today
B. In 6 months
C. In 1 year
D. In 2 years
E. In 3 years

Primary Care Medicine:Question 20


A 23-year-old man presents with a pruritic, blistering rash over his back and elbow.
He was in good health until 6 weeks ago when he developed intermittent watery
diarrhea, occurring six times per day, with frequent nocturnal episodes. He has
abdominal cramping. His symptoms improve with fasting. He does not have fevers,
chills, night sweats, nausea, vomiting, or arthralgias, but has recently lost 2.7 kg (6
Ib). He has had no recent travel or sick contacts.
Physical examination shows a papulovesicular eruption on the lower back and left
elbow. Abdomen is soft, but there is no organomegaly. Stool is light brown and
negative for occult blood.
Which of the following tests would establish the diagnosis?
A. Examination for ova and parasites
B. HIV test
C. Flexible sigmoidoscopy
D. 24 Hour fecal fat collection
E. Small-bowel biopsy
Primary Care Medicine:Question 21
An 82-year-old woman is evaluated in a nursing facility because of recurrent falls.
The staff reports that she has frequent slips/trips that have resulted in falls, but no
history of dizziness or syncope. The patient has mild-moderate cognitive impairment,
osteoarthritis involving her knees and low back (managed with acetaminophen and
ibuprofen), decreased hearing (she has declined a hearing aid), and osteoporosis
(treated with calcium and vitamin D supplements). Her son is concerned that she
may fall and fracture her hip and asks you to do what you can to prevent this
occurrence.
Which of the following will best reduce the risk of hip fracture in this patient?
A. Refer her for physical therapy
B. Add alendronate to her osteoporosis treatment
C. Use restraints to keep her from rising unattended
D. Have her wear hip protectors
E. Refer her for otologic evaluation
Primary Care Medicine:Question 22
A 34-year-old woman presents for advice on prevention of sexually transmitted
diseases (STDs). She is in a long-term relationship with a man who has casual sexual
encounters while traveling for business and who does not always use condoms for
these contacts. She had been using latex condoms and nonoxynol-9 gel for
contraception and STD prevention. However, she has developed a latex allergy over
the past year, manifest first by dermatitis to latex gloves in her work as a dental
hygienist, then as vaginal irritation to latex condoms, and more recently with two
episodes of wheezing when working around (but not touching) latex at her job. She
plans to continue in this sexual relationship and wonders how best to protect herself
against STDs.
Which of the following approaches would be most effective?
A. Continue using nonoxynol-9
B. Use polyurethane condoms.
C. Pretreat with a systemic antihistamine
D. Use natural membrane condoms
E. Begin hormonal contraception, and have the patient and partner get STD testing
periodically

Primary Care Medicine:Question 23


A 77-year-old man is evaluated in routine follow-up care for borderline hypertension
that has been successfully treated with diet and exercise. It has been 8 months since
his last visit, and his blood pressure is 150/110 mm Hg. The patient explains that he
has not been exercising regularly or sticking to his diet since his wife, who had
cooked all of his meals, died 6 months ago. The patient states that while he has
made a good emotional adjustment to losing his spouse, his energy level continues
to be low, leading him to sleep a lot and curtail most of his physical activities.
What is the most likely explanation of the patients symptoms?
A. Dependent personality type
B. Atypical depression
C. Uncontrolled hypertension
D. Post-traumatic stress
E. Anniversary grief response
Primary Care Medicine:Question 24
A 22-year-old male college student is evaluated for a papule on his finger. The
papule has been present for several months, over which time it has gradually
increased in size. It causes pain when he grips a pen. He is sexually active with one
female partner. They use oral contraceptives. He has never had a sexually
transmitted disease.
On examination, he has a 0.5-cm verrucous-appearing papule on the lateral aspect
of the second finger of his right hand, near the distal interphalangeal joint. He has
another, smooth, flat, 0.25-cm papule on the palm of his hand near the thenar
eminence. He has another small verrucous papule on the plantar surface of his left
foot. He has no genital lesions.
Which of the following is the most appropriate next step in the management of this
patient?
A. Imiquimod topical immunotherapy
B. Punch biopsy of the affected skin
C. Rapid plasmin reagin test
D. Triamcinolone ointment
E. Topical therapy with salicylic acid plaster
Primary Care Medicine:Question 25
A 39-year-old man is evaluated in the emergency room for bilateral red eye pain of 2
days duration. He has had no recent upper respiratory symptoms. His medical
history is significant for arthritis with chronic low-back pain and joint stiffness. He
had an episode of left-sided conjunctivitis and acute diarrheal illness 1 year ago
requiring treatment. He takes ibuprofen, and is taking additional doses for the eye
pain.
On examination, the patient is afebrile with normal vital signs, and he looks
uncomfortable. Visual acuity is 20/40 bilaterally. Pupils are 3 mm bilaterally and
reactive. Eyes are intensely injected, including prominent circumcorneal erythema
bilaterally.
What is the most appropriate next step in the management of this patient?
A. Treat with sulfacetamide, 10% ophthalmic drops, until redness clears
B. Treat with corticosteroid ophthalmic drops
C. Immediate referral to ophthalmology
D. Reassure the patient and prescribe over the counter Visine eye drops
E. Treat with ciprofloxacin, 0.3% ophthalmic

Primary Care Medicine:Question 26


A 65-year-old man is evaluated in the office before a planned vacation. His 3-week
itinerary includes visiting Mayan ruins in rural areas of Mexico.
He has coronary artery disease, but a recent stress test was negative for ischemia.
Electrocardiography shows sinus bradycardia and a left bundle branch block, findings
that are unchanged from a prior tracing. He has no known allergies, and his
medications include atenolol, simvastatin, aspirin, and lisinopril.
Which one of the following medications for malaria prophylaxis is contraindicated in
this patient?
A. Doxycycline
B. Atovaquone/proguanil
C. Chloroquine
D. Mefloquine
E. Primaquine
Primary Care Medicine:Question 27
A 36-year-old woman with type 1 diabetes mellitus removes her shoes for a foot
examination. She has a stage 2 deep ulcer under the first metatarsal of the left foot
without associated swelling, erythema, or exudate. She says that the ulcer has been
there for several weeks, but she has had no fever or pain. Filament testing shows
decreased sensation over the toes and metatarsal areas, but not the heel of both
feet. Her temperature is 36.9 C (98.4 F), pulse rate is 72/min, and blood pressure
is 125/80 mm Hg.
What is the most appropriate management of the patients foot ulcer?
A. Occlusive dressing
B. Intravenous antibiotic therapy
C. Topical antibiotic therapy
D. Total contact cast
E. Zinc paste-impregnated bandage (Unna boot)
Primary Care Medicine:Question 28
A 20-year-old male college athlete presents for advice about acne. He has numerous
open and closed comedones on his face, chest, and back. He has no cysts or scarring
at this time.
Which of the following would be the best therapy at this time?
A. 10% Benzoyl peroxide wash
B. Topical clindamycin gel
C. Topical erythromycin gel
D. Combination 5% benzoyl peroxide/1% clindamycin gel
E. Oral tetracycline, 500 mg orally twice a day
Primary Care Medicine:Question 29
A 46-year-old man develops a sudden, severe headache while mowing his lawn. The
headache involves the occipital area and the back of his neck. He has some nausea
and vomits twice. He reports no weakness or parasthesias. He has no visual
disturbances. He has no history of severe headaches. Hs medications include
pantoprazole, ibuprofen, and sertraline.
On examination, his temperature is 37.1 C (98.8 F), pulse rate 100/min, and blood
pressure 150/92 mm Hg. Funduscopic examination shows sharp discs without retinal
hemorrhage; the neurologic examination is normal.
What is the best next step in the evaluation of this patient?

A. No testing
B. CT scan of the head without contrast
C. CT scan of the head with contrast
D. MRI of the head
E. Lumbar puncture
Primary Care Medicine:Question 30
A 28-year-old man presents with 4 days of upper respiratory congestion and sinus
pain. The patient has had no previous significant medical history. He notes that
initially he may have had a mild fever but has not been febrile in the past 48 hours.
He describes some yellowish nasal discharge. On examination, he has fluid behind
his tympanic membranes and moderate tenderness over his maxillary sinuses.
What is the most appropriate initial management?
A. Oral prednisone taper
B. Amoxicillin
C. Oral decongestants
D. Nasal ipratropium
E. Azithromycin
Primary Care Medicine:Question 31
A 55-year-old gardener presents with an itchy rash that has persisted for 1 to 2
weeks. He describes it as red splotches that come and go, and that are intensely
pruritic. He has not previously had a similar rash. It does not seem to be worsened
by cold, sunlight, or touch. The individual lesions persist for less than a day; he has
no pain, shortness of breath, or lip swelling. He cannot recall any new detergents,
soaps, or chemical contacts that were associated with the onset of the rash. His diet
has not changed, he has not traveled, and he feels otherwise well. His medical
history is significant for type 2 diabetes mellitus, well-controlled with metformin
therapy, which he has been taking for 4 years at a stable dose. He takes no other
regular medications. For the rash, the patient has been taking diphenhydramine, 25
to 50 mg every 6 to 8 hours, in an attempt to stop the itch. On physical examination,
he has several red, round, indurated dime-sized lesions over his arms and trunk.
What is the most appropriate next step in the management of this patient?
A. Biopsy of a skin lesion
B. Patch testing for contact irritants
C. Adding ranitidine to the diphenhydramine
D. Adding dapsone to the diphenhydramine
E. Prednisone taper
Primary Care Medicine:Question 32
A 34-year-man calls the office with the acute onset of a nosebleed. He has been
using a tissue to dab his nose for the past 5 minutes but cannot stop the bleeding.
He has not had dizziness, nausea, or vomiting. He has no history of nosebleeds,
recent trauma, or hypertension. Last week he suffered a knee-twisting injury during
a soccer game and has been using ibuprofen 400 mg twice a day for knee
discomfort. What is the most appropriate advice to give this patient as a first step in
stopping his nosebleed?
A. Pinch the nose together between the thumb and index finger
B. Lie down with the head on pillows
C. Apply a hot pack to the bridge of the nose
D. Blow the nose
E. Pack the nose with tissue

Primary Care Medicine:Question 33


A 78-year-old woman is evaluated in an emergency department after falling and
sustaining soft tissue injuries. She fell after getting up to urinate in the middle of the
night, stating that she felt dizzy upon standing and fell en route to the bathroom.
She has had two other falls in the past year, once when she tripped over a curb, and
another time when she lost her balance when she turned quickly. Her medical history
includes hypertension well controlled on hydrochlorothiazide, 25 mg/d; depression
and anxiety managed with sertraline, 100mg at bedtime, and lorazepam, 1 mg upto
twice a day as needed; and gastroesophageal reflux disease treated with
omeprazole. She lives alone and does not smoke or drink.
On physical examination her heart rate is 64/mm and blood pressure 128/72 mm Hg
supine, and 72/mm and 118/68 mm Hg upon standing. Her vision is mildly impaired
(20/40), lower extremity strength is 5/5 except hip flexors are 5- symmetrically, and
her gait is fairly stable. She successfully performed a Get Up and Go test in 10
seconds.
Which of the following is the most appropriate next step in managing this patients
recurrent falls?
A. Refer her to ophthalmology
B. Discontinue lorazepam
C. Discontinue hydrochlorothiazide
D. Refer her for physical therapy
E. Substitute buspirone for sertraline
Primary Care Medicine:Question 34
A 46-year-old woman with gradually progressive menorrhagia attributed to
perimenopause requests an effective nonsurgical approach to reduce menstrual
bleeding. Physical examination, endometrial biopsy, and pelvic ultrasonography
reveal no abnormalities. Laboratory testing shows iron deficiency anemia but is
otherwise normal, including serum thyroid-stimulating hormone level. She is in a
long-term monogamous relationship with her husband, using condoms, but would
like more reliable contraception. She smokes 1.5 packs of cigarettes per day, having
tried unsuccessfully to quit many times. She has put on weight with each attempt to
quit smoking and does not wish to use any hormonal method that might contribute
to weight gain. She has normal blood pressure, an excellent lipid profile, and no
personal or family history of coronary artery disease or venous thromboembolic
disease.
Which of the following approaches to contraception would be safe, effective, and
likely most acceptable to her?
A. Combination oral hormonal contraceptive (after quitting smoking)
B. Transdermal hormonal contraceptive patch
C. Depot med roxyprogesterone acetate
D. Levonorgestrel intrauterine device
E. Endometrial ablation
Primary Care Medicine:Question 35
A 42-year-old woman presents with fatigue, insomnia, headaches and generalized
musculoskeletal pain. The patient works as an advertising executive and associates
the onset of symptoms with a particularly stressful time 18 months ago. She
continues to work but finds it a struggle. Several previous work-ups by specialists
have been negative. The pain symptoms were initially localized to her head, neck,
and shoulders but are now generalized. She reports difficulty falling asleep and not
feeling refreshed upon awakening. Family and social history are noncontributory.

On physical examination, she has bilateral tenderness below the sternocleidomastoid


muscle, near the second costochondral junction, over both greater trochanters, at
the medial fat pads of the knees, at the intersection of the suboccipital muscle, at
the mid-upper trapezius muscle, and at the upper outer quadrant of the buttocks.
What is the likely prognosis for this patients condition?
A. Complete recovery
B. Remitting, relapsing with progressive disability
C. Slow progression to complete disability
D. Improvement with disease-specific treatment
E. Symptom relief with supportive care
Primary Care Medicine:Question 36
You are asked to evaluate a 54-year-old woman before a planned resection of colon
cancer. She has a history of acute bacterial endocarditis related to a history of
intravenous drug use. She is HIV negative. She reports an allergy to penicillin; she
developed an urticarial rash. Her serum creatinine level is 1.5 mg/d L.
Which of the following is the most appropriate endocarditis prophylaxis for this
patient?
A. Cefazolin, 1.0 g intravenously, within 30 minutes before the procedure
B. Vancomycin, 1.0 g intravenously, plus gentamicin, 1.5 mg/kg intravenously,
within 30 minutes before the procedure
C. Clindamycin, 600 mg orally, within 30 minutes before the procedure
D. Vancomycin, 1.0 g intravenously, within 30 minutes before the procedure
E. Ampicillin, 1.0 g intravenously, plus gentamicin, 1.5 mg/kg intravenously, within
30 minutes before the procedure
Primary Care Medicine:Question 37
A 55-year-old white man inquires about prostate-specific antigen (PSA) screening
during a routine office visit. He is generally healthy with no urinary symptoms and
has no family history of cancer. You are aware that the prevalence of prostate cancer
in his age group is 10% to 42%, that the positive predictive value of a PSA value
exceeding 4 ng/mL for carcinoma of the prostate is 28% to 35% and the negative
predictive value is approximately 75%.
What should you tell the patient?
A. His risk of prostate cancer is between 10% and 42% and screening is warranted.
B. Because an elevated PSA would likely be falsely positive, screening is unwarranted
C. A PSA less than 4 ng/mL would mean prostate cancer is highly unlikely.
D. False-positive PSA results are approximately twice as common as true positives.
Primary Care Medicine:Question 38
A 17-year-old male is evaluated in the emergency room with right eye discomfort
and watering of the eye after being struck in the eye while playing paint ball. He
previously had normal visual acuity. Examination shows normal pupillary reflexes,
tearing of the right eye, and mild injection. Slit lamp examination with flourescein
shows a linear abnormality.
What is the most appropriate management of this patients eye condition?
A. The injured eye should be patched to reduce irritation and promote healing
B. No therapy; follow-up as needed
C. Topical anesthetic is indicated to minimize symptoms and irritation
D. Prednisone forte ophthalmic drops
E. Weekly examination of the cornea until re-epithelialization occurs

Primary Care Medicine:Question 39


A 19-year-old man in evaluated for an employment physical. His only specific health
concern is an impaired sense of smell. He has no recent nasal congestion or history
of sinus infections. He has no history of head trauma, and no headaches or visual
disturbances. On examination he has normal blood pressure. Examination of the
nose reveals no crusts or purulent drainage. Neurologic examination is normal.
Genital examination reveals only one testis and a small penis; the pubic hair has a
female pattern. Extremities are normal.
Which of the following statements about this patients underlying condition is true?
A. It is a rare autosomal recessive disorder characterized by mental retardation,
obesity, and hexadactyly, brachydactyly, or syndactyly.
B. It is caused by defects in theKAL gene that lead to impaired embryonic migration
of gonadotropin-releasing hormone neurons to the hypothalamus
C. It is associated with foul-smelling nasal mucosal crusting and Klebsiella infection
D. It is characterized by bradykinesia, pill-rolling tremor, and progressive dementia
Primary Care Medicine:Question 40
A 45-year-old woman presents with pruritus of her chest, back, arms and legs of 4
weeks duration. She has no significant medical history, takes no medications, and
has not used any new soaps, detergents, or cosmetics. Physical examination reveals
scattered excoriations but no rash. Skin examination is otherwise normal. The
patient follows your recommendation regarding bathing and limited use of soaps and
cosmetics and to use an emollient twice daily. On follow-up in 2 weeks, her pruritus
is minimally improved. What would you do next in the management of this patient?
A. Skin biopsy of an excoriation
B. Topical medium-potency corticosteroid cream
C. Hydroxyzine hydrochloride
D. Laboratory evaluation for systemic illness
E. Patch testing for contact allergens
Primary Care Medicine:Question 41
An 80-year-old woman is brought in by her family because she is becoming
increasingly hard of hearing and more socially withdrawn. She denies any discomfort
or other symptoms. On examination, she has thick plugs of cerumen completely
occluding both external auditory canals.
Which of the following would be contraindicated in this situation?
A. Prescribing a ceruminolytic
B. Prescribing olive oil
C. Removing the cerumen with a curette
D. Prescribing hydrogen peroxide drops
Primary Care Medicine:Question 42
An 18-year-old woman is evaluated for pustular acne. Her acne is not associated
with her menstrual periods, which are irregular, occurring every 3 to 4 months. The
patient has significant hair growth on her chin and around her nipples, but she does
not have clitoromegaly or other abnormal hair growth. Her body mass index is 29.
What is the most appropriate therapy for this patient?
A. Oral tetracycline, 500 mg orally twice a day
B. Isotretinoin, 10 mg/d for4 months
C. Spironolactone, 50 mg orally twice a day
D. Combined estrogen and progestin contraceptive
E. Metformin, 500 mg orally twice a day

Primary Care Medicine:Question 43


A 39-year-old woman presents for evaluation of headaches. She had the onset of
headaches at age 17 years, with migraines occurring two or three times a month.
This pattern continued until about 3 years ago when the headaches became more
frequent, and a bothersome headache occurring on a daily basis involving both the
right and left temporal areas and her forehead. These headaches are now present
every day, are present when she wakes up in the morning, and do not respond to
therapy with a selective serotonin antagonist (a triptan). Her current medications
include zolmitriptan, 1 tablet 8 to 10 times a month; naproxen, 500 mg orally twice
a day; acetaminophen with codeine, two or three tablets daily; and amitriptyline, 75
mg orally at bedtime. On funduscopic examination, the discs are sharp; neurologic
examination is normal.
What is the best next step in the evaluation of this patient?
A. CT scan of the head without contrast
B. CT scan of the head with contrast
C. MRI of the head
D. Magnetic resonance angiography
E. No imaging
Primary Care Medicine:Question 44
A 56-year-old woman presents to discuss the results of her recent lipid panel. She
has no personal or family history of premature coronary artery disease. She has
hypertension, which is well controlled with atenolol, 50 mg/d. She is a nonsmoker
and has no history of diabetes mellitus or vascular disease. Her serum total
cholesterol is 240 mg/dL with an HDL cholesterol of 68 mg/dL and triglycerides of
148 mg/dL. Her calculated LDL cholesterol is 142 mg/dL. Her blood pressure is
132/82 mm Hg.
What is the appropriate initial management?
A. Lifestyle changes
B. Atorvastatin
C. Cholestyramine
D. Gemfibrozil
E. Niacin
Primary Care Medicine:Question 45
A 72-year-old man is evaluated in follow-up of a red, velvety lesion detected in his
mouth 1 month ago. He has a history of moderate hypertension. He lives alone, eats
a healthy diet, but continues to smoke one half pack of cigarettes per day and
occasionally uses chewing tobacco. He has a history of heavy alcohol use in his past,
but now abstains. Examination of the oropharynx shows a small velvety red patch
along the buccal mucosa; the lesion is unchanged from the examination 1 month
ago.
What is the most appropriate next step in the evaluation of this patient?
A. Order a CT scan of the head and neck.
B. Culture the lesion for herpes simplex virus.
C. Begin isotretinoin and re-evaluate the patient in 8 weeks.
D. Refer the patient to dentistry for bite evaluation.
E. Refer the patient for a biopsy of the lesion.

Primary Care Medicine:Question 46


A 64-year-old man is evaluated for intractable hiccups that have persisted for 48
hours and disrupted his sleep. Attempts to stop the hiccups, including cold water
gargling, holding his breath, and the Valsalva maneuver, have failed. He is in good
health and has not had recent upper respiratory infection, cough, indigestion,
bloating, focal neurologic deficits, weight loss, or dyspnea. He has a 25-pack-year
smoking history but quit smoking 8 years ago. He takes no medications.
On physical examination, pulse rate is 86/min, blood pressure is 128/85 mm Hg; his
skin is normal and there is no lymphadenopathy. Sinuses are nontender, nasal
turbinates are without hyperemia or discharge. Tympanic membranes are clear. The
neck is supple and without masses. The chest is clear to auscultation. There are no
focal neurologic findings. Chest radiograph is normal.
What is the most appropriate next step in the management of this patient?
A. Chiorpromazine therapy
B. Ranitidine therapy
C. Cetirizine therapy
D. MRI of the head
E. CT scan of the abdomen
Primary Care Medicine:Question 47
A 77-year-old man screens positive for urinary incontinence at a routine clinic visit.
He relates 6 months of small-volume urine loss if he bends over and at times when
he sneezes. He notes some urinary hesitancy and occasionally has a sense of
incomplete emptying. He awakens to urinate one or two times a night but has no
dysuria or constipation. His medical problems are hypertension treated with
hydrochlorothiazide, large-joint osteoarthritis managed with celecoxib and
acetaminophen with codeine, and depression treated with nortriptyline.
On examination, vital signs (taken supine and standing) and heart and lungs are
normal. Lower extremity strength and sensation are intact. Sphincter tone is normal,
and the prostate gland is soft and normal in size. Urinalysis shows 1 + glucose and 1
+ protein but no cellular elements; serum electrolytes and creatinine are normal;
random serum glucose level is 138 mg/dL, and a post-void residual volume is 100
mL. Which of the following are the most likely cause and most appropriate
intervention for this patients urinary incontinence?
A. Overflow incontinence; stop nortriptyline and codeine
B. Stress incontinence; stop hydrochlorothiazide
C. Overflow incontinence; stop nortriptyline and start finasteride
D. Stress incontinence; teach timed voids and Kegel exercises
E. Overflow incontinence; treat diabetes
Primary Care Medicine:Question 48
A 32-year-old woman presents with right pelvic pain since stopping oral
contraceptive pills 12 months ago in an attempt to become pregnant. She recalls
having had similar but less troublesome pain for 1 year or more before she started
taking oral contraceptive pills for contraception and painful periods 5 years ago.
The pain worsens before and during her menses, and she has dyspareunia with deep
penetration. She has never been pregnant. Her husband is healthy and has a 7-yearold child from a prior marriage. The patients menses are regular, 28 to 30 days. She
and her husband are having intercourse twice at midcycle each month. She had an
appendectomy at age 5 years. She has had no bowel or bladder symptoms.

On examination, she is a well-appearing white woman who is tearful when discussing


her troubles with pelvic pain and difficulty conceiving. She reports pain in the right
pelvis on bimanual examination, which is otherwise normal.
Abdominal and rectovaginal examinations are normal. Pelvic ultrasound is normal.
Which is the best next step in the management of this patient?
A. Cyclic progesterone
B. Leuprolide acetate for 3 months, then clomiphene citrate
C. Hysterosalpingography
D. Laparoscopy
E. Psychiatric evaluation
Primary Care Medicine:Question 49
A 54-year-old woman is evaluated for fatigue, insomnia, headaches, and generalized
musculoskeletal pain. She reports she has been evaluated many times in the past for
these symptoms but without ever having received a definitive diagnosis. After
several visits, a complete physical examination, history, and laboratory studies, her
physician diagnoses the patient with fibromyalgia.
All of the following are appropriate pharmacologic treatment considerations EXCEPT?
A. Amitriptyline
B. Ibuprofen
C. Tramadol
D. Fluoxetine
E. Cyclobenzaprine
Primary Care Medicine:Question 50
You are asked to evaluate a 54-year-old woman before a planned resection of colon
cancer. She had been previously healthy and takes only occasional NSAIDs as
needed for pain. She reports an allergy to penicillin; she developed a mild rash
without oral involvement or wheezing. She has normal renal function.
Which of the following is the most appropriate antibiotic prophylaxis for this patient?
A. No antibiotic prophylaxis needed
B. Clindamycin, 600 mg intravenously, plus gentamicin, 1.5 mg/kg intravenously,
within 30 minutes before the procedure
C. Cefotetan, 2.0 g intravenously, within 30 minutes before the procedure
D. Vancomycin, 1 .0 g intravenously, within 30 minutes before the procedure
E. Ciprofloxacin, 500 mg intravenously, before the procedure
Primary Care Medicine:Question 51
A 68-year-old female retired teacher reports worsening shoulder pain over several
weeks that now severely limits use of her right arm, making simple task such as
brushing her hair difficult and awakening her from sleep. She has taken occasional
over-the-counter ibuprofen, with minimal improvement. She recalls no injury. Her
other medical problems are stable and include osteoporosis, diabetes, and
hypertension. She has no other arthritic symptoms but similar symptoms in the
contralateral shoulder improved after several weeks of nonsteroidal agents and
physical therapy. Her current medications are alendronate, glyburide,
hydrochlorothiazide, and lisinopril. On examination, the shoulder is normal to
inspection and there is subacromial tenderness. External rotation is painless. Pain is
experienced beyond 40 degrees of abduction and prevents the patient from
abducting the arm beyond 90 degrees. The arm can be abducted passively to 120
degrees. Laboratory tests include a serum creatinine level of 1.8 mg/dL.

What is the most appropriate next step in the management of this patient?
A. Obtain anteroposterior, lateral, and axillary radiographs of the shoulder
B. Order an MRI of the shoulder
C. Inject the subacromial bursa with aqueous methylprednisolone, 40 mg
D. Prescribe a regular regimen of ibuprofen, 300 mg three times daily
E. Measure serum uric acid
Primary Care Medicine:Question 52
A 74-year-old man is being evaluated before cataract replacement and intraocular
lens implant. The patient has type 2 diabetes mellitus, coronary artery disease with
history of congestive heart failure after bypass surgery 4 years ago, and
hypertension. His most recent hemoglobin Al c was 7.2%. Medications include
glyburide, amlodipine, lisinopril, furosemide, and aspirin.
Which of the following laboratory evaluations is indicated in pre-operative evaluation
of this patient?
A. No laboratory tests are needed
B. Chest radiograph
C. Prothrombin time
D. Electrocardiography
E. Complete blood count with platelet count
Primary Care Medicine:Question 53
A 33-year old male carpenter presents with an intensely pruritic rash involving both
elbows, wrists, and hands. He denies any prior rash. He has no systemic symptoms,
or dyspnea or wheezing. Examination reveals an erythematous papulosquamous
eruption in both antecubital fossae, wrists, and the interdigital web spaces of both
hands, with lichenification.
Which of the following would be most appropriate initial therapy?
A. Lindane
B. Nasal swabs, with antibiotic treatment aimed at eradication of Staphylococcus
colonization
C. Systemic oral methyprednisolone
D. Elimination diet to identify food allergy
E. Topical medium potency corticosteroids
Primary Care Medicine:Question 54
A 27-year-old woman presents for evaluation. She demands thyroid medication to
make her stronger because she is now Senator for the State of California. She
reports she has been sleeping 2 to 3 hours a night for the past 2 weeks and feels
well. She has not been hungry. She tells you she left her last provider because he
was reporting information to the election canvassing board that was damaging her
election hopes.
What is the most likely diagnosis?
A. Paranoid schizophrenia
B. Paranoid personality disorder
C. Histrionic personality disorder
D. Bipolar disorder
E. Narcissistic personality disorder

Primary Care Medicine:Question 55


A 24-year-old female graduate student is seen at the Student Health Clinic for
pretravel advice. Her research will involve fieldwork for 10 weeks at the ThailandCambodia border. She has no significant medical problems and takes no
medications. Her immunizations were updated for a previous trip 6 months ago.
What would you recommend for malaria prophylaxis for this patient?
A. Doxycycline
B. Atovaquone/proguanil (MalaroneTM)
C. Chloroquine
D. Mefloquine
E. Primaquine
Primary Care Medicine:Question 56
A 35-year-old female surfer reports right ear pain for the past 2 days. On
examination, there is pain with palpation of her tragus and with traction on the pinna
of the right ear. On inspection of the external auditory canal, there is a large amount
of white moist exudate occluding the view of the tympanic membrane, with
erythema of the visible underlying epidermis. There is no tenderness on percussing
the mastoid bone just behind the right ear.
Which of the following would be appropriate management of the exudate in the
external auditory canal?
A. Prescribe oral penicillin
B. Prescribe an oral quinolone antibiotic
C. Prescribe a topical antibiotic/corticosteroid
D. Culture the exudate and treat according to culture results
E. Prescribe hydrogen peroxide drops
Primary Care Medicine:Question 57
A 60-year-old woman with a history of type 2 diabetes mellitus and hypertension
reports a dry cough over the last 6 weeks. Her usual medications are lisinopril and
glyburide, both of which she has taken for more than 5 years. She had a viral upper
respiratory infection after her granddaughter visited 2 months ago, and though the
other symptoms have resolved, she has not been able to get rid of the cough. She
sleeps well, but her cough is worst when she arises in the morning.
On examination, her pulse rate is 80/min and blood pressure is 120/60 mm Hg. The
tympanic membranes are clearly visualized and normal, nasal mucosae are pink and
moist with white exudate present; cobblestoning is present on the posterior pharynx
and lung examination shows no wheezes, ronchi, or crackles.
What is the most appropriate management of the patients cough?
A. Prescribe an antihistamine/decongestant combination
B. Prescribe ranitidine
C. Prescribe azithromycin
D. Prescribe a codeine-containing cough syrup
E. Prescribe amoxicillin
Primary Care Medicine:Question 58
A 35-year-old man returns for follow-up of a blood pressure reading of 148/96 mm
Hg at his previous visit. He brings several outside readings of 145-154/85-98 mm
Hg, with his pulse ranging from 86 to 94/min. He does not smoke, take any
medications, or have diabetes mellitus. His body mass index is
25. The physical examination is otherwise unremarkable.

Laboratory studies:
Serum sodium 140 meq/L
Serum potassium 3.2 meq/L
Serum chloride 103 meq/L
Serum bicarbonate 24 meq/L
Plasma glucose 119 mg/dL
Blood urea nitrogen 15 mgldL
Serum creatininel.1 mg/dL
What is the most appropriate next step in the evaluation of this patient?
A. Measurement of serum cortisol
B. Renal artery duplex ultrasonography
C. Exercise treadmill testing
D. Measurement of plasma metanephrines
E. Determination of plasma renin/aldosterone ratio
Primary Care Medicine:Question 59
A 39-year-old woman presents for evaluation of headaches. She had the onset of
headaches at age 17 years, with migraines occurring two or three times a month.
This pattern continued until about 3 years ago when the headaches became more
frequent, and a bothersome headache occurring on a daily basis involving both the
right and left temporal areas and her forehead. These headaches are now present
every day, are present when she wakes up in the morning, and do not respond to
therapy with a selective serotonin antagonist (a triptan). Her current medications
include zolmitriptan, 1 tablet 8 to 10 times a month; naproxen, 500 mg orally twice
a day; acetaminophen with codeine, two or three tablets daily; and amitriptyline, 75
mg orally at bedtime. On funduscopic examination, the discs are sharp; neurologic
examination is normal.
What would be the most appropriate therapeutic approach to the patient?
A. Taper all current medications
B. Stop acetaminophen with codeine
C. Begin gabapentin therapy
D. Begin riboflavin therapy
E. Begin fluoxetine therapy
Primary Care Medicine:Question 60
A 36-year-old man with type 1 diabetes mellitus presents with 3 days of right ear
pain. The pain has gradually progressed and he has noted a small amount of
drainage from the ear. He is unaware of any trauma. On examination, his
temperature is 38.1 C (100.6 F). The pinna and mastoid are not tender; the ear
canal is moderately erythematous with some thin white discharge. The tympanic
membrane is partially visualized and is unremarkable. A few small lymph nodes are
palpable. The cranial nerves are intact.
What is the most appropriate initial management?
A. Hospitalization for intravenous ticarcillin/clavulunate
B. Ciprofloxacin, 750 mg orally twice a day
C. Antibiotic and corticosteroid otic drops (Cortisporin-TC Otic Suspension)
D. Acetic acid drops
E. Amoxicillin, 500 mg orally twice a day

Primary Care Medicine:Question 61


A 24-year-old woman is evaluated for oral ulcers that occur frequently, usually at
times of stress, and last about 1 week. The ulcers are unrelated to rash, joint
symptoms, or fever. She notes that they recur more frequently recently, and she
attributes a recent 4.4-kg (1 0-Ib) unintentional weight loss to pain with eating
during outbreaks of the ulcers. Her medical history is significant for irritable bowel
syndrome diagnosed by her previous provider after she reported occasional
abdominal pain and diarrhea. She has no history of an eating disorder, illicit drug
use, blood transfusion, or any previous sexual activity. Her family history is
unrevealing. On examination, she has two small, mildly tender oral ulcerations that
she notes have been present for about 6 days.
What is the best next step in the evaluation of this patient?
A. Order a colonoscopy.
B. Order an HIV test.
C. Order radiographs of the spine.
D. Prescribe acyclovir.
E. Refer her for a biopsy of the lesions.
Primary Care Medicine:Question 62
A 28-year-old woman presents for a pre-employment physical examination. She
plans to work in a long-term care facility for children. Her medical history is
unremarkable. She does not recall ever having had chicken pox. She received a
tetanus booster at age 15 years but has not had any other immunizations since.
Which of the following immunization regimens would be appropriate for this patient?
A. Tetanus-diphtheria
B. Varicella
C. Varicella (only if seronegative)
D. Tetanus-diphtheria, hepatitis A and B, varicella
E. Tetanus-diphtheria, hepatitis A and B, varicella (only if seronegative)
Primary Care Medicine:Question 63
An 82-year-old woman returns to your office because of urinary incontinence and
recurrent urinary tract infections that she first related a few months ago. She notes
urinary frequency and daily low-volume urine loss, usually when she coughs or
sneezes, that have been present for over a year. She has also had three episodes of
bladder infections over the past 15 months that resolved with antibiotic therapy, but
seem to recur every few months despite her drinking cranberry juice. She does not
now have symptoms of urinary urgency or dysuria. Her only medical problem is a
history of superventricular tachycardia that has been well controlled with low-dose
atenolol. Prior pelvic examination was positive for a small cystocele and moderate to
severe atrophic changes. Urinalysis showed 5 to 10 leukocytes per high-power field
and was nitrite-positive; culture grew normal gram-positive flora. At her initial visit
you recommended Kegel exercises and timed voids, but they have not helped her
incontinence as much as desired.
What is the most appropriate pharmacologic intervention at this time?
A. A trial of pseudoephredrine
B. A course of antibiotics
C. A trial of imipramine
D. A trial of oxybutinin
E. A trial of topical estrogen

Primary Care Medicine:Question 64


A 57-year-old man is evaluated for headache. He reports 1 year of awakening with
headaches most mornings. The headaches are frontal, bilateral, and improve after he
has gotten up. His only medication is acetaminophen one or two mornings each
week. He has only an occasional headache at other times of day, typically in the
afternoon during a stressful day at work, but those headaches feel different. He has
no nausea, vomiting, neurologic symptoms, or visual changes, and has no history of
malignancy, or neurologic disease. He feels exhausted when he wakes up despite
sleeping soundly for 7 hours each night. He drinks two cups of coffee in the morning,
a stable pattern. He has gained 8.8 kg (20 Ib) in the past year after a foot injury
decreased his ability to exercise. On examination, he is obese (BMI 34) with blood
pressure 155/95 mm Hg. Extraocular movements are intact; optic discs are flat
without papilledema. Neck is obese with full range of motion and no masses,
adenopathy, or thyromegaly. Detailed neurologic examination shows only mild facial
asymmetry, attributed to past Bells palsy.
What additional history is most likely to be helpful in confirming the diagnosis?
A. Alcohol use pattern
B. Herbal medication use
C. Beck depression score
D. History of head trauma
E. Additional sleep history
Primary Care Medicine:Question 65
You are asked to evaluate a previously healthy 26-year-old man before an elective
repair of an inguinal hernia. He has no personal or family history of easy bruising,
bleeding disorders, or thromboembolic disease. He takes occasional acetaminophen
or NSAIDs for pain. On physical examination, he is a well-developed young man. His
vital signs are normal. His physical examination is notable only for a reducible
indirect right inguinal hernia. He has no petechiae, ecchymoses, or telangiectasia.
What is the most appropriate laboratory evaluation before the planned surgery?
A. Prothrombin time, activated partial thromboplastin time, and platelet count
B. No laboratory testing is necessary
C. Prothrombin time, activated partial thromboplastin time, platelet count, and
bleeding time
D. Bleeding time
E. Complete blood count with platelet count
Primary Care Medicine:Question 66
A 49-year man is admitted for dyspnea after he ran out of his inhalers one week ago.
He has severe obstructive pulmonary disease and vasculitis, and is taking
prednisone, 40 mg/d. One day after admission, the patient developed right eye
discomfort. Eye examination at that time was negative except for trace injection
laterally of the right eye, with visual acuity 20/30. Now two days after admission, the
patient reports, I cant see out of my right eye. On examination, he has diffuse
erythema of the right eye. The lower anterior eye is opaque with a fluid level. Visual
acuity testing shows only light and dark perception.
Which of the following is the most likely diagnosis?
A. Open-angle glaucoma
B. Iritis
C. Keratitis
D. Keratoconjunctivitis sicca
E. Endophthalmitis

Primary Care Medicine:Question 67


A 70-year-old man returns for a routine follow-up visit. He has hypertension,
gastroesophageal reflux disease, and hyperlipidemia. His medications include
metoprolol, lisinopril, omeprazole, and atorvastatin. He is active and enjoys working
on projects around his home. You note that he has had a 5.4-kg (12-lb) weight loss
since his visit 6 months ago. His weight had been 176to 184 lb for the previous 4
years. He does not use dietary supplements or tobacco. He drinks alcohol
infrequently. He states that he is fatigued but his wife states he has been doing less
and has been more moody. He admits to difficulty sleeping. He denies constitutional
symptoms but feels his appetite has decreased. He denies dysphagia, abdominal
pain, or change in stool, but feels full and nauseated when he eats. His physical
examination is unremarkable, and stool testing for occult blood is negative.
Which of the following is the most appropriate step in the management of this
patient?
A. Begin fluoxetine
B. Colonoscopy
C. Upper endoscopy
D. Encourage caloric supplements
E. Begin metoclopramide
Primary Care Medicine:Question 68
A 29-year-old female snowboarder is evaluated for the sudden onset 2 hours ago of
severe vertigo, nausea, and vomiting with any head movement. She has no hearing
loss or diplopia. Neurologic examination is normal.
What is the best next step in the management of this patient?
A. Perform otolith repositioning maneuver
B. Order MRI of the head
C. Order electronystagmography
D. Prescribe meclizine
E. Lumbar puncture
Primary Care Medicine:Question 69
Two weeks ago, a migrant farm worker presented to a rural clinic with cough, and
active tuberculosis was diagnosed. The clinics nurse spent considerable time with the
patient while the patient was coughing. The nurse had previously been PPDnegative; a 5-tuberculin unit (Mantoux) PPD test is placed and, the nurse has no
reaction at 48 hours.
Which of the following would be the best strategy to evaluate the nurse at this time?
A. Placement of a 10-TU Mantoux PPD test
B. Placement of another 5-TU test in 3 months
C. A chest radiograph
D. No further testing
E. Placement of another 5-TU test in 1 week
Primary Care Medicine:Question 70
A 51-year-old man is evaluated for migraine headache. He reports a history of
migraines with onset at age 20 years. He had one or two headaches a month until
about age 30, and since that time has only had headaches once every 3 to 4 years.
When he was in his 20s he was usually treated with ergotamines with success. He
has not used any treatment for the past 8 years because his headaches have been
infrequent and not severe. Todays headache is on the left side of the head, maximal
behind the left eye, with a pulsating quality. It is associated with a scotoma, and he
feels nauseated.

His medical history includes gastroesophageal reflux disease, coronary artery


disease, depression, and plantar fasciitis; he had a myocardial infarction 2 years ago.
His medications include rabeprazole, atenolol, enteric coated aspirin, bupropion, and
simvastatin.
What treatment would you recommend for this patients headache?
A. Sumatriptan intramuscularly
B. Sumatriptan orally
C. Caffeine/ergotamine orally
D. Naproxen and metoclopramide
E. Dihydroergotamine nasal spray
Primary Care Medicine:Question 71
A 36-year-old man with long history of heavy alcohol use presents within 24 hours of
his last drink. He states that he needs help with the withdrawal. He denies any
other drug use. On examination, his pulse rate is 120/min and blood pressure is
172/96 mm Hg. He is moderately tremulous and is having visual hallucinations.
What is the most appropriate management?
A. Admit the patient and give clonidine, 1 mg
B. Admit the patient and give diazepam, 20 mg
C. Prescribe outpatient chlordiazepoxide, 50 mg
D. Prescribe outpatient lorazepam, 2 mg
E. Prescribe outpatient atenolol, 50 mg
Primary Care Medicine:Question 72
You make a home visit to one of your long-term patients, an 84-year-old woman
with moderate Alzheimers dementia, for which she takes donepazil, and urinary
incontinence. The patient lives with her daughter, who cares for her. Her daughter
notes that that your patient has had severe halitosis recently, and has been
complaining of dry eyes. On examination, the patient has bad breath, dry mouth,
and angular cheilitis. The daughter reports also that her mother has been having
trouble swallowing dry food. Her medications have been stable, and she is not taking
any over-the-counter medications.
What is the most appropriate next step in the management of this patient?
A. Order anti-Ro and anti-La antibody titers.
B. Order a speech and swallowing evaluation.
C. Discontinue donepezil therapy.
D. Add water at mealtime and sugar-free lemon candy during the day.
E. Begin therapy with amoxicillin/clavulanate.
Primary Care Medicine:Question 73
An asymptomatic 48-year-old woman undergoes her annual physical examination.
She had a vaginal hysterectomy for uterine leiomyoma 4 years ago. She has had
vaginal Pap smears every 3 years, all of which have been normal.
Which of the following is the most appropriate course of action for this patient?
A. Pelvic examination and vaginal Pap smear now and every year until age 65
B. Pelvic examination and vaginal Pap smear now and every 3 years until age 65
C. Pelvic examination but no vaginal Pap smear now and every year until age 65
D. Pelvic examination but no vaginal Pap smear now and every 3 years until age 64
E. Discontinue pelvic examination and vaginal Pap smear

Primary Care Medicine:Question 74


A 76-year-old retired accountant comes in to your clinic accompanied by his spouse.
His wife has noted his being more forgetful over the past year, and he agrees that
his memory is not as good as it used to be. He has more difficulty recalling names
and is more forgetful about current events and things he has recently heard. He
continues to drive and has not had accidents or gotten lost, and he continues to
handle the household finances without gross errors (although he has misplaced bills
a few times). He also continues to enjoy his usual hobbies that include playing cards
and golfing. His medical diagnoses are limited to gastroesophageal reflux disease,
treated with omeprazole, and atrial fibrillation that has been treated for 5 years with
diltiazem and warfarin.
Physical examination is remarkable for an irregular heart beat and a Mini-Mental
State Examination score of 27 out of 30 (he missed which day of the week it was by
one day, and could recall only 1 of 3 items on short-term memory testing). There are
no focal neurologic findings. Complete blood count and comprehensive serum
metabolic panel are normal; serum vitamin B 12 level is 542 pg/mL and serum
thyroid-stimulating hormone level is 4.9 UImL. A noncontrast CT scan of the head
shows mild periventricular white matter disease, consistent with ischemic changes.
Which of the following is the most appropriate diagnosis for this patients memory
problems?
A. Multi-infarct dementia
B. Mild cognitive impairment
C. Alzheimers disease
D. Impaired memory due to subtle hypothyroidism
E. Drug-induced memory problems
Primary Care Medicine:Question 75
A 48-year-old man with difficult-to-treat hypertension returns for follow up. He has a
7-year history of hypertension and a family history of essential hypertension and
obesity. His blood pressure readings are approximately 160/95 mm Hg despite
compliance with a regimen of atenolol, 100 mg/d; lisinopril, 40 mg/d; and
hydrochlorothiazide, 25 mg/d. He tries to follow a low-sodium diet. He takes no
over-the-counter medications, no herbal medications, and does not eat licorice. He
finds it hard to exercise, because he is fatigued after a day at work as an insurance
agent. He usually naps when he gets home, eats dinner, then goes to bed. He falls
asleep as soon as he puts his head on the pillow, and sleeps soundly all night. His
wife reports that he snores loudly.
On physical examination, he is well-appearing but obese (BMI 30), his pulse rate is
56/min and regular, and blood pressure 154/96 mm Hg. Facies are normal, but
posterior oropharynx is crowded from a large tongue and long soft palate. Neck is
obese without bruits or jugular venous distention. Cardiac and lung examinations are
normal. The abdomen is soft, without bruits or striae. Pulses are full and symmetric
in the upper and lower extremities. His electrolytes, renal function, and urinalysis are
normal; 24-hour urine sample is normal for metanephrines, catecholamines, and
vanillylmandelic acid.
Which of the following would be the best next step in the evaluation of this patient?
A. Renal artery duplex ultrasonography
B. Measurement of plasma metanephrines
C. Polysomnography
D. Overnight dexamethasone suppression test
E. Determination of blood lead level

Primary Care Medicine:Question 76


You are asked to evaluate a 71-year-old man before a scheduled transurethral
resection of the prostate gland for benign prostatic hyperplasia. His medical history is
notable for a bileaflet mechanical prosthetic aortic valve replacement for aortic
stenosis 6 years ago. He is receiving long-term anticoagulation with warfarin to a
target INR of 2.5 (range 2.0 to 3.0). He has no history of embolic complications
related to his valve or bleeding complications related to his anticoagulation. Recent
echocardiography demonstrated moderate concentric left ventricular hypertrophy,
but a normal left atrium and normal left ventricular ejection fraction. The patient did
not take his warfarin for 2 days before admission to the hospital. On laboratory
evaluation today, his INR is 1 .9. The surgery is scheduled for tomorrow.
What is the most appropriate management of this patients long-term anticoagulation
before surgery?
A. Unfractionated heparin to a target activated partial thromboplastin time of 1 .5
times normal, to be
discontinued 1 hour before surgery
B. Vitamin K, 5 mg orally today, and recheck INR tomorrow before surgery
C. No warfarin today and recheck INR tomorrow before surgery
D. Four units of fresh frozen plasma and recheck INR tomorrow before surgery
Primary Care Medicine:Question 77
A 53-year-old woman has been noticing progressive, dull pain over her lateral hip for
several weeks. She also describes a snapping sensation when she walks. She
recently joined a group of women who walk 1 to 2 miles several mornings per week,
and her pain is limiting this activity. There is no pain at rest.
On physical examination, her BMI is 22. There is full range of motion of the hip and
mild tenderness of over the lateral thigh. The pain is reproduced by adduction of the
hip. There are no sensory deficits.
What is the most appropriate next step in the management of this patient?
A. Obtain radiographs of the hip
B. Refer to an orthopedic surgeon
C. Inject the trochanteric bursa with aqueous methylprednisolone, 40 mg
D. Recommend stretching, heat, and NSAID therapy as needed
E. Refer for nerve conduction velocity study
Primary Care Medicine:Question 78
A 68-year-old black woman presents for transfer of care. She has hypertension,
hypothyroidism, and sleep apnea; she is taking replacement levothyroxine and CPAP
at night. Family history is positive for a parent who had glaucoma and legal
blindness. On examination, the patient is in no apparent distress; she is obese (BMI
32); her blood pressure is 155/95 mm Hg. Funduscopic examination shows a cup
disk ratio of 0.5 without disk hemorrhage.
In addition to the patients age, which of the following factors in this patient poses
the highest risk for developing glaucoma?
A. Family history
B. Hypertension
C. Sleep apnea
D. Hypothyroidism
E. Black American ethnicity

Primary Care Medicine:Question 79


A 63-year-old woman returns for follow-up. She has a history of Graves disease,
mild osteoarthritis, and osteoporosis. Her medications include alendronate, calcium
supplements, a multivitamin, vitamin E, and glucosamine/chondroitin.
She states she has had trouble concentrating at work and feels run down. She sleeps
poorly and has stopped walking for exercise. She has had a 6-kg (14-Ib) decrease in
weight from her baseline. Physical examination shows only Heberdens nodes.
She had a normal pelvic examination with Pap smear 8 months ago, a negative
mammogram 7 months ago, negative stool testing for occult blood 7 months ago,
and negative sigmoidoscopy 2 years ago.
On laboratory testing, hematocrit is 37%, serum thyroid-stimulating hormone
concentration is 0.7 i U/mL, and serum electrolytes, creatinine, and liver tests are
normal.
Which of the following is most appropriate next step in the management of this
patient?
A. Start megace therapy
B. Order a colonoscopy
C. Order an abdominal CT scan
D. Start mirtazapine therapy
E. Start propylthiouracil therapy
Primary Care Medicine:Question 80
A 46-year-old woman presents because of several days of pain in her wrist. The pain
worsened today to the point where she had to interrupt her work and called for an
urgent appointment. She is a faculty member at the local university and is grading
final examinations for a large class. She is having trouble writing due to the pain.
Her only prescription medication is levothyroxine for hypothyroidism. She has been
taking over-the-counter naproxen without relief.
Examination reveals no synovitis, intact motor strength, and positive Finkelsteins
test (percussion of the median nerve produces minimal discomfort and apposition of
the backs of the hands with the wrists at 90 degrees flexion produced no symptoms).
The patients pain localizes to the lateral wrist over the distal radius. There is no pain
with downward pressure on the first carpometacarpal joint.
What do you recommend next?
A. Radiograph of the wrist and hand
B. Corticosteroid injection into the first carpometacarpal joint
C. Corticosteroid injection into the tendon sheath at the radial styloid
D. Nerve conduction study of the median nerve
E. Therapy with a cyclooxygenase 2 inhibitor
Primary Care Medicine:Question 81
A 55-year-old man describes brief episodes of vertigo with movement of his head,
along with delayed-onset, fatigable horizontal nystagmus on Hallpike-Dix testing
when the head is turned to the left.
What is the most appropriate next step in the management of this patient?
A. Perform otolith repositioning maneuver
B. Order MRI of the head
C. Perform electrocardiography
D. Prescribe meclizine

Primary Care Medicine:Question 82


A previously healthy, athletic 40-year-old man presents after a second syncopal
episode. He was swimming in a pool when he climbed out to rest and was bitten by a
horsefly on the arm and bled from the bite. A minute later, he suddenly felt unwell
and fainted. Two or three tonic-clonic jerks were noted, but there was no
incontinence. The patient was unconscious for 1 minute and then woke up
completely. He was observed to be pale and diaphoretic with a pulse rate of 48/min.
He has no cardiac risk factors. He had a previous syncopal episode immediately after
vasectomy.
Physical examination shows a healthy-appearing man with a pulse rate of 60/min
and a blood pressure of 100/62 mm Hg. The rest of the physical examination is
normal, as is electrocardiography.
Which of the following is the best next step?
A. Electroencephalography
B. Complete blood count, serum chemistry panel, and measurement of serum
thyroid-stimulating hormone
C. Echocardiography
D. Event monitor for 2 weeks
E. No additional testing
Primary Care Medicine:Question 83
A 57-year-old man develops herpes zoster on the left side of his forehead and eyelid.
He is evaluated 14 hours after the onset of the blisters and is treated with
valacyclovir, 1 g three times a day, and acetaminophen with codeine. He returns the
next morning reporting the pain was so severe he could not sleep. A slit lamp
examination shows no corneal involvement.
What is the most appropriate recommendation to help with pain relief?
A. Switch from valacyclovir to acyclovir
B. Switch from valacyclovir to famcyclovir
C. Add prednisone
D. Begin capsaicin cream
E. Begin methadone
Primary Care Medicine:Question 84
A 34-year-old woman presents for routine care. On questioning, she admits to
drinking one to two alcohol-containing beverages each evening. On weekends, she
may drink two to three each evening.
She has not had any legal or social problems related to alcohol use; she has not had
any significant withdrawal symptoms and has been consistent in this pattern of use
for several years. On CAGE questioning, she denies any effort or interest in cutting
back and does not feel guilty about her alcohol use. She denies any history of
morning alcohol use, but does note that her husband intermittently encourages her
to decrease her alcohol use.
What is the most appropriate intervention at this point?
A. Reassure the patient that there is no evidence of alcohol abuse or dependence.
B. Intervene briefly to identify drinking patterns and to set common goals on future
use.
C. Begin inpatient detoxification.
D. Prescribe benzodiazepines for supporting cessation.
E. Prescribe disulfiram.

Primary Care Medicine:Question 85


A 35-year-old man is evaluated for decreased hearing in his right ear. He noticed the
hearing change 24 hours ago while talking on the telephone. He is recovering from
an upper respiratory infection and is taking a decongestant, but has no fever, ear
pain, vertigo, tinnitus, or trauma.
On examination, he is afebrile and hears speech well in the office. Although there is
some cerumen in each canal, the ear canals and tympanic membranes can be seen
and appear to be normal. He can hear finger-rubbing in his left but not in his right
ear. A Weber test lateralizes to the left.
What is the most likely diagnosis?
A. Unilateral conductive hearing loss
B. Serous otitis
C. Cholesteatoma
D. Unilateral sensorineural hearing loss
E. Otosclerosis
Primary Care Medicine:Question 86
A 28-year-old woman presents with a 2-year history of intermittent tinnitus. With
these episodes she usually experiences vertigo and fullness in her ears. Sometimes
she misses a day of work when these episodes occur. She now believes she has lost
hearing. She works as a store manager and describes her profession as stressful.
She has no history of exposure to excessive noise. Her medical history is significant
for occasional sinusitis. She takes no medications.
Physical examination shows no clear tympanic membranes and mild rotatory
nystagmus with rapid movement to the supine position (Barany or Dix-Hallpike
maneuver). Audiometry confirms low-frequency sensorineural hearing loss.
What is the most likely diagnosis?
A. Labyrinth itis
B. Otosclerosis
C. Menires disease
D. Stress-induced tinnitus
Primary Care Medicine:Question 87
A 77-year-old women with diabetes mellitus, hypothyroidism, and hypertension is
hospitalized after spilling hot coffee on herself and suffering second degree burns on
her leg and foot. Her usual medications are metformin, 500 mg twice a day;
glyburide, 10 mg/d; levothyroxine, 150 g/d; and lisinopril, 40 mg/d. Recent
laboratory studies included a hemoglobin A1C of 8.5% and serum thyroid-stimulating
hormone level of 5 U/mL. After admission, her usual medications were continued
and her wounds were treated with local care; pain was controlled with combined
hydrocodone and acetaminophen, 2 to 3 tablets per day, and zolpidem, 2.5 mg at
bedtime was prescribed for insomnia. On hospital day 2, she developed confusion,
and on evaluation her serum glucose level was 52 mg/dL. Despite initial treatment
with D5OW, lowering of her glyburide dosage to 5 mg/d, and consistent food intake,
hypoglycemia recurred twice over the next 48 hours.
On hospital day 3, hydroxyzine, 10 mg every 6 hours as needed, was added for
itching and on day 4 she developed atrial fibrillation with a heart rate of 130/mm. On
examination, her weight is 120 pounds, temperature is 37 C (98.6 F), blood
pressure 108/70 mm Hg. She appears anxious and is oriented to place but
incorrectly states the month and year and recalls only 1 of 3 items on short-term
memory testing. Neurologic examination is nonfocal, and the rest of the examination
shows only rapid irregular heart rate. Complete blood count is normal, as is a
comprehensive metabolic panel, except for a plasma glucose level of 68 mg/dL.

What is the most likely cause of this patients hospital complications?


A. Polypharmacy with adverse effects from the hydrocodore/acetaminophen and
zolpidem
B. Enforced adherence with adverse effects from glyburide and levothyroxine
C. Adverse drug event from hydroxyzine
D. Surreptitious alcohol use and withdrawal following hospitalization
Primary Care Medicine:Question 88
A 42-year-old woman being evaluated for her annual examination reports feeling
very fatigued. This problem has been getting worse for several years.
She runs her own housecleaning business and has no energy left at the end of the
day.
She has not been pursuing hobbies, going out, or doing anything she enjoys. She is
the divorced single parent of 10- and 13-year-old children and finds it difficult to do
recreational things with them, but has no feelings of guilt. She has no cardiac or
pulmonary symptoms but cannot exercise as she could a few years ago.
She has occasional right knee pain but no other arthralgias or myalgias and no rash.
She awakens in the morning with a dry throat and mouth. She has tried citalopram,
paroxetine, bupropion, and venlafaxine each for 4 to 12 weeks over the past year
without improvement.
She reports that she sleeps soundly for 7 hours per night, but feels tired on
awakening. Her children complain that she snores.
On physical examination, she is fatigued but otherwise healthy-appearing. Her
temperature is 36.5 C (98 F), pulse rate is 84/min and regular, and her blood
pressure is 130/82 mm Hg; her BMI is 25. She has no skin lesions.
Mucous membranes are pink and moist. Oropharyngeal opening is narrowed from
large noninflamed tonsils and long soft palate, but no erythema. There is no
adenopathy or erythema or swelling at joints, and trigger points are nontender to
palpation.
Remainder of examination is normal. Laboratory results, including complete blood
count, metabolic profile, and measurement of serum thyroid-stimulating hormone,
are normal.
What is the most likely diagnosis?
A. Chronic fatigue syndrome
B. Lyme disease
C. Hepatitis C
D. Obstructive sleep apnea
E. Major depression
Primary Care Medicine:Question 89
You are asked to perform a preoperative evaluation for a 56-year-old man with
rheumatoid arthritis before a scheduled total knee replacement. His only other
medical problem is hypertension.
His medications include prednisone, 5 mg/d; naproxen, 500 mg twice daily; atenolol,
50 mg/d; hydrochlorothiazide, 25 mg/d; and omeprazole, 20 mg/d. On physical
examination, his pulse rate is 70/min and blood pressure is 142/80 mm Hg.
He has facial plethora and central obesity. His heart and lungs are normal on
examination.
What is the most appropriate regimen for perioperative management of his
corticosteroid therapy?

A. Hydrocortisone, 50 mg intravenously every 8 hours for 3 doses; followed by


hydrocortisone, 25 mg intravenously every 8 hours for 3 doses; then resume usual
outpatient regimen
B. Hydrocortisone, 100 mg intravenously every 8 hours for 3 doses; then
hydrocortisone, 50 mg intravenously every 8 hours for 3 doses; then hydrocortisone,
25 mg intravenously every 8 hours for 3 doses; then resume usual outpatient
regimen.
C. Prednisone, 10 mg on the day of surgery and the first postoperative day; then
resume his usual dose of prednisone
D. Prednisone, 5mg on the day of surgery; prednisone, 10mg on the first
postoperative day; then resume usual outpatient regimen.
Primary Care Medicine:Question 90
A 52-year-old presents for evaluation of an enlarging lesion on the trunk. He is a
sailor and has had extensive sun exposure since childhood. He has no history of skin
cancer or suspicious moles. On examination, there is a 2-cm raised lesion with blueblack and reddish pigmentation and irregularly defined border.
Which of the following statements is correct about metastatic patterns of this lesion?
A. Early treatment of this lesion with topical 5-fluorouracil could have cured it.
B. Metastases spread predominantly via hematogenous spread.
C. Metastatic sites include the pericardium.
D. This lesion rarely metastasizes.
E. This lesion may metastasize locally.
Primary Care Medicine:Question 91
A 55-year-old black man presents for an annual examination. His medical history
includes sickle cell trait and hypertension. His medications include
hydrochlorothiazide, diltiazem, and atenolol. He has stable nocturia, urinating once
each night. Digital rectal examination discloses an asymmetric prostate gland. The
right lobe is slightly larger, and the anterior portion feels firmer than the rest of the
gland but no discrete nodule is noted. Serum prostate-specific antigen (PSA)
concentration is 3.2 ng/mL and has been 2.8 to 3.2 ng/mL for the previous 4 years.
What do you recommend for this patient?
A. Continued annual PSA monitoring
B. Follow-up PSA measurement in 3 to 6 months
C. Determination of free/total PSA ratio
D. Biopsy of the prostate gland
E. Transrectal ultrasonography of the prostate gland
Primary Care Medicine:Question 92
A 38-year-old man reports 3 months of worsening dyspepsia without acid reflux,
nausea, vomiting, or difficulty swallowing. He has been using an over-the-counter
antacid with some relief. He has not been traveling or camping in the last year.
Physical examination is normal, and his hematocrit is 33% with hemoglobin level of
11 g/dL.
Which of the following is the best next step in the management of this patient?
A. Prescribe eradication therapy for Helicobacter pylon infection
B. Refer for upper endoscopy
C. Prescribe a proton pump inhibitor
D. Prescribe a promotility agent
E. Order abdominal CT scan

Primary Care Medicine:Question 93


A 72-year-old woman is brought into the urgent care clinic after an episode of
syncope that occurred while she was reaching to get some dishes out of a high
cupboard and turned her head to answer a question from her son. She reports
feeling dizzy for a few seconds before she fainted. She has no known cardiac disease
and no other known risk factors, but she has never had her serum cholesterol
measured. She has not had chest pain. She takes hydrochiorothiazide for
hypertension and alendronate for osteoporosis.
Physical examination shows a thin woman, with a blood pressure of 150/94 mm Hg
and a pulse rate of 84/min and regular supine and blood pressure of 146/96 mm Hg
and a pulse rate of 90/min standing. Carotid pulses are normal with no bruits. A 1/6
systolic murmur is audible at the right sternal border without radiation. Pulses are
present in the feet. Electrocardiography shows normal sinus rhythm with borderline
left ventricular hypertrophy by voltage in aVL (11 mm) but is otherwise normal.
Nonfasting serum cholesterol level is 179 mg/dL with an HDL cholesterol of 50
mg/dL.
Which of the following is the best next step in the evaluation of this patient?
A. Carotid duplex ultrasonography
B. Exercise tolerance test
C. Event monitor
D. Carotid sinus massage
E. Echocardiography
Primary Care Medicine:Question 94
A 37-year-old man presents with a 3-month history of hoarseness. He has not
noticed any other symptoms. He is not a voice professional and has not overused his
voice. He is a nonsmoker and does not chew tobacco. He drinks one or two alcoholic
beverages a week. He does not recall any recent upper respiratory tract infections.
On examination, his pulse rate is 80/min; his blood pressure is 130/70 mm Hg; his
weight is 118 kg (260 Ib). Examination of the head, eyes, ears, nose, and throat is
normal.
What is the most appropriate approach to management of this patient?
A. Referral to an otolaryngologist to evaluate vocal cords
B. Chest radiography
C. 2-week course of erythromycin therapy
D. Intranasal corticosteroid and an antihistamine
E. 2-month course of proton pump inhibitor therapy
Primary Care Medicine:Question 95
A76-year-old woman with type 2 diabetes mellitus, hypertension, and elevated
serum cholesterol presents with intermittent calf pain with walking. Her medications
include metformin, glyburide, lisinopril, and atorvastatin. On examination, she has
reduced dorsalis pedis pulses.
What medication is most likely to improve her symptoms?
A. Pentoxifylline
B. Amlodipine
C. Clopidogrel
D. Aspirin
E. Cilostazol

Primary Care Medicine:Question 96


A 72-year-old man, a former construction worker, is evaluated as a new patient. His
wife accompanies him to his visit and notes that his hearing is not as acute as it used
to be. She says that he cannot participate in conversations in restaurants and has
trouble talking on the telephone. He needs to turn the television up so loudly that
she fears for her own hearing. You need to speak up in the office to get an
appropriate response from him. He has trouble hearing finger-rubbing bilaterally.
You refer him for an audiogram. The left ear demonstrates mild decreased hearing in
the upper frequencies consistent with aging. The right ear shows significant downsloping sensorineural loss in the upper frequencies, up to 40 dB lower than the left
ear at 8000 Hz.
What is the most appropriate next step in the management of this patient?
A. Refer the patient to an otologist for stapedectomy.
B. Refer the patient for a hearing aid fitting.
C. Order MRI of the cerebellopontine angle.
D. Order a contrast-enhanced CT scan of the head.
Primary Care Med icine:Question 97
A 30-year-old woman is evaluated in clinic for dizziness of 6 weeks duration. The
symptoms are worse when she gets out of bed in the morning but never go away
entirely. She does not have vertigo. She has also had abdominal pain and nausea.
She recently had a normal abdominal ultrasound and upper endoscopy. Since her
upper endoscopy 3 weeks ago, she has had right-sided chest pain, which is
persistent but not pleuritic. On physical examination, she has mild abdominal
discomfort in all four quadrants with deep palpation.
What is the most likely diagnosis?
A. Pulmonary embolus
B. Astrocytoma
C. Hypochondriasis
D. Somatization disorder
E. Abridged somatization
Primary Care Medicine:Question 98
An 82-year-old women with diabetes mellitus taking glipizide, 5 mg/d, hypertension
well-controlled with lisinopril, 20 mg/d, atrial fibrillation managed with warfarin, 2.5
mg/d, and digoxin, 0.25mg/d, and depression treated with mirtazepine, 15 mg at
bedtime, presents with gradual weight loss of 4.4 kg (10 Ib) over the past few
months. She relates diminished appetite and intermittent nausea for the past 6 to 12
months. She indicates her mood has been good and that the mirtazapine has
improved her spirits and sleep. She has no other specific physical complaints and
denies chest or abdominal pain, heartburn or reflux symptoms, or change in her
bowel habits.
On examination, she weighs 53 kg (116 Ib), down from 57.6 kg (126 Ib) 8 months
ago, heart rate is 60/mm and irregular, and blood pressure 126/78 mm Hg. Her
examination is unremarkable with pertinent negatives including no
lymphadenopathy, normal cardiac (except for irregular heart rate) and abdominal
examinations. Hematocrit is 42%, serum electrolyte levels are normal; serum
creatinine is 1.3 mg/dL, glucose 110 mg/dL. INR is 3.0, and hemoglobin A1C level is
7.4%.

What is the most appropriate medication adjustment to try to improve this patients
appetite and weight loss?
A. Decrease digoxin dose from 0.25 to 0.125 mg/d
B. Decrease glipizide dose from 5 to 2.5 mg/d
C. Decrease mirtazapine dose from 15 to 7.5 mg at bedtime
D. Decrease warfarin dose from 2.5 to 2 mg/d
E. Decrease lisinopril dose from 20 to 10 mg/d
Primary Care MediCine:Question 99
A 38-year-old man is seen in follow-up for clavicular fracture sustained in a motor
vehicle accident 6 weeks ago, requesting clearance to return to work. He had been
driving alone in daylight when his car left the road at a curve and struck a tree. He
does not think he lost consciousness or struck his head, but he does not remember
leaving the road. He recalls no prodrome and remembers clearly the sound of
crunching metal from the impact. The patient does not smoke, drink, or use other
drugs. He is obese and was diagnosed with type 2 diabetes mellitus and
hypertension 3 years ago and takes metformin and enalapril as his only medications.
He has never had hypoglycemia and has no history of cardiac or neurologic diseases
including seizures.
On evaluation by medics at the site of the accident, his pulse rate was 90/min and
regular, blood pressure was 150/1 00 mm Hg, and blood glucose was 180 mg/dL. He
was alert, oriented, and complained only of anterior shoulder pain at the site of the
fracture. He was evaluated in the emergency department and has been treated
conservatively with a shoulder sling, pain medication, and physical therapy. He feels
ready to return to work.
On specific questioning, he reports three near-miss car accidents in the past 2 years;
in none of these cases is he quite sure what happened.
What is the next information you would obtain to look for an underlying explanation
for his driving history?
A. Neuropsychiatric testing
B. Polysomnography
C. 3-day blood glucose monitoring
D. Electroencephalography
E. Arrhythmia monitor
Primary Care Medicine:Question 100
You are asked to evaluate a 67-year-old man who is hospitalized for an
esophagectomy for early stage adenocarcinoma of the esophagus. The diagnosis was
made on routine biopsy of a patch of Barretts esophagus that had been followed for
6 years with serial endoscopic examination.
He does not have dysphagia. His medical history is notable for type 2 diabetes
mellitus that has been poorly controlled over the past several months.
His most recent hemoglobin A1c was 9.2% and fasting morning blood glucose levels
have often been in excess of 250 mg/dL. His diabetic medications are glyburide, 10
mg/d; and metformin, 500 mg twice daily.
He has continued on his outpatient medications since admission, but has just been
changed to an American Diabetes Association 2000 kcal/d diet.

What is the most appropriate management for this patients diabetes mellitus during
his hospital stay?
A. Increase glyburide to 10 mg twice daily and metformin to 850 mg with each meal
B. Discontinue oral medications and initiate long-acting basal glargine insulin at night
and short-acting prandial lispro insulin with each meal
C. Discontinue oral medications and initiate an insulin drip with a target blood
glucose level of 120 to 180 mg/dL
D. Discontinue his oral medications and initiate a multiple-dose regimen of
intermediate-acting NPH insulin in the morning and night combined with prandial
injections of short-acting lispro insulin with a target blood glucose level of 120 to 180
mg/dL
E. Discontinue metformin during hospitalization and add sliding-scale regular insulin
to cover excursions from target blood glucose measurements in excess of 150 mg/dL
Primary Care Medicine:Question 101
A healthy 35-year-old woman calls the office to report that her daughter was sent
home from school with head lice. The school nurse has recommended that the girl be
treated with permethrin. There is no prior history of head lice in the household. The
mother has no itching of the scalp but reports that she and her daughter have
shared combs. There are no other children at home. The mother asks for advice. You
tell the mother to launder the clothes and bedding.
What other advice should be given?
A. Disinfect the house
B. Treat herself with permethrin
C. Ask another adult to thoroughly comb her hair with a nit comb
D. Treat herself with synergized pyrethrin
E. Use lindane shampoo
Primary Care Medicine:Question 102
A 34-year-old woman presents for routine gynecologic screening. She also asks you
to examine her skin as she occasionally uses tanning salons and is concerned about
cancer, with a lesion on her back that is slowly enlarging. She burns easily and
recalls several severe burns in childhood while fishing that caused blistering. She
notes that she has more moles than most people. Family history is negative for
melanoma, but her mother also has many moles. To the patients knowledge, her
mother has never had a mole biopsied. On examination, the patient is fair-skinned
and has blue eyes. Scattered on the trunk and extremities are numerous small welldemarcated and uniformly pigmented moles, each <5 mm. There is one 8-mm
darkly pigmented lesion on the upper back that has an irregular indistinct border and
nonuniform dark brown-black hues.
What is the most important next step in the evaluation of this patient?
A. Re-evaluate in 3 months
B. Punch biopsy of the lesion
C. Refer for total body photography
D. Excisional biopsy of the lesion
E. Topical 5-fluorouracil

Primary Care Medicine:Question 103


A 65-year-old man comes to the emergency department with right knee pain. He has
recently retired and is traveling with his wife. They have been walking more than
usual while sightseeing, but he has not fallen or injured the knee. His knee felt a bit
stiff last night at bedtime. When he awoke his right knee was painful, but he was
able to walk without difficulty. He has not had any previous joint problems. He has
no fever, chills, rigors, rash, or other pain. The patient has a history of hypertension
and recently started taking daily aspirin.
His temperature is 37.5 C (99.5 F), pulse rate is 66/min, and blood pressure is
148/90mm Hg. On examination, his right knee is warm and erythematous. There is a
mild effusion and pain with passive flexion. The hematocrit is 39%, leukocyte count
is 12,500/L, and serum creatinine is 1.0 mg/dL. Six months ago, the serum uric
acid was 7.2 mg/dL.
Which of the following is the next best step in the management of this patient?
A. Rest, elevation, and celecoxib therapy
B. Radiography of the knee
C. Aspiration of the knee joint
D. Blood cultures
E. Allopurinol therapy
Primary Care Medicine:Question 104
A 22-year-old male college senior is planning a trip through India. He is departing in
4 weeks and will be traveling for 8 weeks. He has no medical problems. His
immunizations were updated the summer before entering college 3.5 years ago. He
received diphteria-tetanus; measles, mumps, and rubella; and a meningococcal
immunization. His parents suggested he get his vaccinations for his trip before the
school year ends when his health insurance will end. He denies plans for any sexual
contact while traveling or other high risk behaviors but does plan on staying at
remote hostels.
What immunizations would you recommend at this time?
A. Hepatitis A and typhoid
B. Hepatitis A, typhoid, and polio booster
C. Combined hepatitis A and hepatitis B, and typhoid
D. Combined hepatitis A and hepatitis B, typhoid, and polio booster
Primary Care Medicine:Question 105
A 42-year-old female contractor reports 4 months of medial right elbow and forearm
pain. She also reports associated numbness and tingling in her right 4th and 5th
digits. On examination, there is no swelling or redness at the elbow, forearm, or
hand, and full range of motion is intact at the elbow. There is no exacerbation of
symptoms with shoulder or neck movements, or with resisted wrist flexion or
extension. There is slight wasting of the hypothenar muscles and tenderness
medially at the elbow. Percussion over that same area causes tingling that radiates
to the 4th and 5th digits but does not produce symptoms when repeated.
What is the most likely cause of her hand and elbow pain?
A. Carpal tunnel syndrome
B. Cubital tunnel syndrome
C. Medial epicondylitis
D. Saturday night palsy
E. C7 radiculopathy

Primary Care Medicine:Question 106


A 57-year-old man with a long history of intermittent back pain related to his work
as a truck driver presents with severe back pain radiating down his left leg that
began 2 days ago when he was helping a friend move. He says that his left leg feels
weak. He has to urinate one or two times per night and has slight urinary hesitancy.
Physical examination shows a heavyset man who has difficult moving; his pulse rate
is 92/min and his blood pressure is 150/92 mm Hg; body mass index is 28. Left
straight-leg raise causes pain at 45 degrees, his great toe dorsiflexion is weak, and
his ankle jerk is diminished. Anal wink is present, the prostate gland is enlarged, and
sphincter tone is normal. No sensory level is detectable. He says that he has never
had pain like this before, and he asks for pain pills and to be able to go lie down.
Lumbosacral spine films are normal and erythrocyte sedimentation rate is 10 mm/h.
In addition to analgesics and clinical follow-up, what is the best management at this
time?
A. Lumbosacral traction therapy
B. Chiropractic adjustments
C. Physical therapy back school and exercise program
D. Referral to an orthopedic surgeon
E. Bed rest with activity as tolerated
Primary Care Medicine:Question 107
A 49-year-old woman who has had a hysterectomy for leiomyoma seeks care for hot
flushes that are disturbing her sleep several times a night. She is very uncomfortable
but has recently read that hormones are not safe. She has no family history of
breast cancer. She is otherwise healthy and does not smoke. She drinks two glasses
of milk per day and takes a multivitamin.
Which of the following would be the best therapy for this patients hot flushes?
A. Black cohosh
B. Raloxifene
C. Soy supplements
D. Estrogen
E. Sertraline
Primary Care Medicine:Question 108
A 35-year-old man presents for evaluation of a rash that has been present for
several months. It is not pruritic but has enlarged and now covers his chest as well
as his back. He has had a similar rash in the past that became hypopigmented when
he became tan.
What would be the most appropriate therapy?
A. Amoxicillin
B. Azithromycin
C. Triamcinolone, 0.1% cream
D. Clotrimazole, 1% cream
E. Oral ketoconazole
Primary Care Medicine:Question 109
A 53-year-old woman returns for follow-up of nail disease. She has had thickened
yellow toenails for the past 3 years. She initially tried topical antifungal creams
without benefit. Six months ago, she was treated with three cycles of itraconazole,
400 mg/d for 1 week each month. She has not noticed any improvement in her nails.
On examination, both great toenails show thickening with debris beneath the nail.

What is the most appropriate management at this time?


A. Terbinafine, 250 mg/d
B. Itraconazole, 400 mg/d 1 wk/month for 2 months
C. Itraconazole, 400 mg/d 1 wk/month for 3 months
D. Ketoconazole, 2% cream to the nail bed daily
E. Culture of a nail sample
Primary Care Medicine:Question 110
A 68-year-old woman presents with acute onset of pleuritic chest pain. She describes
a several-day history of upper respiratory symptoms including mild cough, sinus
congestion, and low-grade fevers. Symptoms began earlier in the day while she was
at rest. She denies any significant cardiopulmonary medical history. Her medications
include combination estrogen and progestin and omeprazole. On examination, her
pulse rate is 78/min and blood pressure is 176/74 mm Hg. Her lungs are clear, S1
and S2 are regular and without murmur or rub.There is no edema of the extremities.
Chest radiograph and electrocardiogram are normal. Serum D-dimer is 1 .0 g/mL
(normal <0.5 glmL).
What is the most appropriate next step in the management of this patient?
A. Spiral CT scan
B. Transesophageal echocardiography
C. Ventilation/perfusion scan
D. Lower extremity duplex ultrasonography
E. No further testing
Primary Care Medicine:Question 111
A 24-year-old female graduate student presents for a routine examination. She notes
that she has been experiencing an increase in heart fluttering recently. The
palpitations come on without precipitant, last 2 to 20 minutes, and are sometimes
associated with shortness of breath. She gets the symptoms at least once a week, at
various times of the day. She sometimes experiences a feeling of doom along with
the palpitations, and at times feels numbness in her fingertips. Her graduate
program is stressful but is going well. S
he is in a stable relationship; does not smoke, drink, or use recreational drugs; and
drinks one cup of coffee daily. She has no symptoms of hyperthyroidism; she
exercises infrequently but does not note exercise intolerance. Her heart rate is
76/mm and regular, and cardiac examination is normal. Electrocardiography is
normal.
What is the most appropriate next step in the evaluation of this patient?
A. Refer her for cognitive behavioral therapy.
B. Begin therapy with paroxetine.
C. Order a continuous loop monitor (Holter monitor).
D. Refer her for an electrophysiology study.
E. Order a transtelephonic monitor (King of Hearts monitor).
Primary Care Med icine:Question 112
An 18-year-old woman is evaluated for a pre-college physical. Examination of the
mouth reveals yellow discoloration on the inner enamel surfaces of her teeth; the
exterior surfaces appear white and normal. She has no history of dental problems
and brushes regularly, flosses occasionally, and gets regular dental care. She has no
oral complaints.

What is the next step in the evaluation of this patient?


A. Refer her to a dentist.
B. Ask her about self-induced vomiting.
C. Ask if she grinds her teeth at night.
D. Prescribe a proton pump inhibitor.
E. Ask about childhood fluoride exposure.
Primary Care Medicine:Question 113
A 68-year-old woman is being started on long-term anticoagulation for chronic atrial
fibrillation. Her baseline INR is 1.0, and after 2 days of taking warfarin, 5 mg/d, it
rises to 1.3.
What is the most appropriate next step in the management of this patient?
A. Reduce the daily dose of warfarin by approximately half and check the INR.
B. Maintain the current dose and recheck the INR in another 2 days.
C. Maintain the current dose and recheck the INR in 1 week.
D. Stop warfarin for 3 days and restart at 2 mg/d.
E. Increase the dose from 5 mg/d to 7.5 mg/d.
Primary Care Medicine:Question 114
You are assessing a patient with a suspicious 1.2-cm papular skin lesion on the leg.
The lesion has an indiscrete edge and is pigmented with bluish-purple and orangered hues. A total thickness excisional biopsy of the entire lesion is performed, and
pathology confirms early superficial spreading melanoma with a clear margin and
oculometer depth of 0.88 mm. Which of the following is the best management plan?
A. Re-excision with a 2-cm margin
B. Re-excision with a 10-cm margin
C. Re-excision with a 2-cm margin and elective lymph node dissection
D. Re-excision with a 2-cm margin and sentinel lymph node biopsy
E. No further excision is necessary
Primary Care Medicine:Question 115
A 44-year-old male fish-packer reports intermittent bilateral volar wrist and finger
pain in his thumbs and index and middle fingers, radiating up the forearms and
aggravated by wrist movements at work. He remembers no trauma, and has no grip
weakness, morning stiffness, or night-time symptoms. On examination, there is no
muscle atrophy in the hand or forearm, full range of motion is intact in the wrist and
elbow, joints are normal without synovial swelling, but thumb abduction is weakened
bilaterally. Phalens test is negative, but Tinels test is positive bilaterally.
What is the next step in managing this patients symptoms?
A. Electromyography
B. Referral for surgical release of the radial nerve
C. MRI of the wrist
D. Wrist splints
E. Vitamin B 6 supplements

Primary Care Medicine:Question 116


In the previous patient, which of the following findings would be most suggestive of
a possible associated systemic disease?
A. Negative Phalens test
B. Positive Tinels test
C. Bilateral wrist symptoms
D. Absence of muscle atrophy
E. Weak thumb abduction
Primary Care Medicine:Question 117
A 62-year-old Filapina is evaluated for vaginal dryness and pain with intercourse.
She was diagnosed with breast cancer (T1N2M0) 3 years ago and stopped her
hormone replacement therapy. She is taking tamoxifen. Her husband is very upset
that she has pain during intercourse. She also notes nocturia twice per night and
irritative symptoms during the day with urinary urgency and frequency but no
incontinence. Vaginal examination shows pale mucosa with visible vessels and a
urethral caruncle. Urinalysis is normal except for 1 erythrocyte per high-power field.
The plasma glucose level is 93 mg/dL.
Which of the following is the best therapy at this time?
A. Increase the dosage of tamoxifen
B. Begin raloxifene therapy
C. Recommend vaginal lubricants
D. Prescribe a vaginal estrogen ring
E. Prescribe nightly vaginal conjugated estrogen cream
Primary Care Medicine:Question 118
A 58-year-old man presents because he noticed in the shower that his scrotum has
suddenly gotten larger. He has not had fevers or penile discharge. Lifting or straining
does not make it markedly worse.
Examination shows a nontender enlarged scrotal sac on the left, which
transilluminates. Palpation of the scrotum reveals a worm-like mass posterior to a
normal testicle. Insertion of a finger into the inguinal canal on the left is
unremarkable, but the scrotal enlargement limits the examination. The mass does
not decrease with the patient lying down.
Which of the following is the best next step in the management of this patient?
A. Reassurance without further testing
B. Nuclear scan of the testes
C. Urine culture
D. Abdominal CT scan
E. Surgical consultation for possible hernia repair
Primary Care Medicine:Question 119
A 45-year-old man with a mechanical aortic valve has been taking warfarin, 5 mg/d,
for many years with a stable INR around 3.0. He has a recent history of bleeding
from peptic ulcer disease that was treated with triple antibiotic therapy. His only
other medication is rabeprazole. For chronic intermittent pain in his knee, he is given
a prescription for celecoxib. Two weeks later his INR is 12. There is no evidence of
acute bleeding.

What is the most appropriate next step in the management of this patient?
A. Stop the warfarin and recheck the INR in 2 days.
B. Administer vitamin K, 2 to 2.5 mg orally, and recheck the INR the following day.
C. Admit the patient for observation and administration of fresh frozen plasma
D. Administer vitamin K, 5 mg orally, and recheck the INR the following day.
E. Administer vitamin K, 5 mg intravenously, and recheck the INR the following day.
Primary Care Med icine:Question 120
A 72-year-old man presents because he has seen blood in his urine. He has
paroxysmal atrial fibrillation and mild hypertension; his medications include warfarin
and hydrochlorothiazide. He smokes about 5 cigarettes a week. His pulse rate is
85/min and irregular, blood pressure is 138/86 mm Hg, and he has no
lymphadenopathy or abdominal masses. Examination of the prostate gland shows an
enlarged, nontender gland without nodules. Urinalysis in the office shows 2+ blood
and 10 erythrocytes per high-power field on microscopic examination, without casts
or crystals. Hematocrit is 35%; serum creatinine is 1.1 mg/dL. A second urinalysis 1
week later confirms 18 erythrocytes per high-power field. Cystoscopy, CTintravenous pyelography, and ultrasonography and biopsy of the prostate gland are
normal.
Which of the following is the best treatment for this patient?
A. Discontinue warfarin therapy
B. Begin finasteride therapy
C. Begin doxazosin therapy
D. Begin saw palmetto therapy
E. Begin ciprofloxacin therapy
Primary Care Med icine:Question 121
A 52-year-old woman is evaluated in the clinic for headache and sore throat that
have persisted for 48 hours. She has had a dry cough, worse at night, but no visual
changes, myalgia, or arthralgia. On examination, her temperature is 37 C (98.6 F),
and there is a mildly erythematous pharynx and tender anterior cervical
lymphadenopathy. The sinuses are nontender, the neck is supple, and the chest is
clear to auscultation. The patient cares for her two preschool grandchildren in her
home 5 days per week. The patient has no allergies.
What is best next step in the management of this patient?
A. A 10-day course of penicillin therapy
B. A 5-day course of azithromycin therapy
C. Rapid streptococcal antigen testing
D. Reassurance
E. Throat culture

ANSWERS
Primary Care Medicine:Question 1
The correct answer is A
Educational Objectives
Manage chronic warfarin therapy.
Critique
According to the recently validated CHADS-2 index, this patient has a absolute 4%
annual risk of stroke. This risk can be reduced by approximately 68% by long-term
oral anticoagulation. Warfarin therapy should be initiated at a dose of 5 mg/d. Higher
loading doses have been not been shown to hasten attainment of a therapeutic INR
and frequently produce excessively high values. Aspirin alone would reduce the risk
by approximately 15%. Recent data indicate that rate control plus long-term oral
anticoagulation improved quality of life and outcome in older patients with atrial
fibrillation more than cardioversion and treatment with antiarrhythmic agents (Van
Gelder et al. and Wyse et al). Because warfarin suppresses the levels of
anticoagulant factors protein C and protein S earlier than the procoagulant factors II,
VII, IX, and X, it can rarely produce skin necrosis. This is a serious phenomenon to
which patients with protein C deficiency are more susceptible, but its rarity makes
initial therapy with heparin unnecessary.
Primary Care Medicine:Question 2
The correct answer is D
Educational Objectives
Recognize the optimal initial medical management of obesity.
Critique
This patient is obese, with a BMI of 35, which confers high risk for complications of
obesity, including hypertension, type 2 diabetes mellitus, and cardiovascular disease,
independent of the presence or absence of other risk factors. The presence of other
risk factors, including smoking, existing hypertension, dyslipidemia, or diabetes,
would certainly further increase her risk, but she is already at high risk; it would be
appropriate to initiate treatment of her obesity prior to obtaining additional
laboratory testing.
The optimal initial management of obesity includes changes in both diet and
exercise. Typical physical activity accounts for only 15% of daily caloric expenditure.
Exercise may increase this caloric expenditure somewhat, but is not sufficient to
achieve sustainable weight loss. Current dietary recommendations include a balanced
diet with a negative caloric balance of 500 to 1000 kcal/d. A negative caloric intake
of this scale will result in a 0.5 to 1.0 kg (1 to 2 Ib) per week weight loss over the
initial 6 months of the diet.
Diets rich in fat tend to be high calorie, and saturated fats, in particular, are
implicated in cardiovascular risk. Recommended diets generally include less than
30% of caloric intake from fat. Convenience and snack foods tend to be rich in fats
and refined sugars, and are very calorie-dense. Minimizing these foods is essential
for weight loss. Excessive alcohol consumption is incompatible with weight loss.
Very-low-calorie diets and rapid weight loss are potentially harmful. Diets rich in
protein and fat, and very low in carbohydrates (the Atkins Diet) are effective for
weight loss, largely by limiting caloric intake, but are of uncertain safety in terms of
their effect on lipids.
Exercise is an essential component of sustainable weight loss. Patients who succeed
with weight loss achieve lasting changes in habits of diet and daily exercise. Exercise
has additional salutary effects on cardiovascular risk independent of weight loss.
Pharmacologic therapy with sibutramine or orlistat, although successful in short-term

weight loss, is generally reserved for patients who have not achieved weight loss
goals after trials of diet and exercise alone. These agents are most successful when
coupled with diet and exercise. Weight lost is typically regained following cessation of
pharmacologic therapy.
There is considerable interest in the potential of bariatric surgery to achieve
sustainable weight loss. Concomitant strides are being made in the endocrinology of
weight and appetite control. Leptin appears to be a hormone responsive to stores of
body fat, while the recently discovered hormone ghrelin appears to signal
preprandial hunger and satiation.
Primary Care Med icine:Question 3
The correct answer is D
Educational Objectives
Recognize the medical complications of bulimia.
Critique
This patient presents with an acute upper gastrointestinal bleed. Endoscopy reveals
erosive esophagitis and a Mallory-Weiss tear. Her laboratory findings are not
consistent with her stated history. She has hypochloremia, hypokalemia, and a high
serum bicarbonate concentration, findings that suggest prolonged vomiting not the
1-hour history given. Endoscopy results show significant reflux disease in addition to
the Mallory-Weiss tear. This combination would be most consistent with bulimia.
Bulimic patients can develop severe reflux disease and the repeated vomiting can
cause Mallory-Weiss tears. The electrolyte disturbances that occur with bulimia are
an important clue to suspecting the diagnosis. Hypokalemia and hypochloremia are
relatively common electrolyte abnormalities in bulimic patients.
Bacterial toxin-induced food poisoning would not explain the electrolyte
abnormalities or the erosive esophagitis. NSAID-induced pill esophagitis would not
cause the electrolyte disturbances seen in this patient. Although scleroderma can
lead to severe reflux esophagitis, there are no other features in this case to suggest
scleroderma.
Primary Care Medicine:Question 4
The correct answer is B
Educational Objectives
Recognize esophageal dysphagia and its most common cause, esophageal reflux
disease; recognize esophageal dysmotility and begin the workup.
Critique
Difficulty swallowing that is delayed by several seconds suggests an esophageal
rather than oropharyngeal cause of dysphagia. The most common cause of
esophageal dysphagia is esophageal reflux disease, by way of either dysmotility or
scarring and stricture formation in the lower esophagus.
Localization of symptoms to the substernal notch is common, and does not
accurately represent the site of esophageal obstruction. Esophageal malignancy
could cause the same complex of symptoms seen in this patient but is a much less
common cause of esophageal dysphasia than is reflux disease.
Achalasia usually causes unpredictable, intermittent dysphagia for both solids and
liquids, rather than the predictable symptoms with solids seen in this patient.
Cricopharyngeal spasm causes oropharyngeal obstruction and therefore should cause
immediate difficulty swallowing, rather than the delay of several seconds seen in this
patient. Plummer-Vinson syndrome occurs when esophageal webs are present in the
setting of iron deficiency anemia and is unlikely in this patient with a normal
hematocrit. The patients symptoms of dysphagia for solids, delayed by several
seconds suggest an obstruction in the esophagus. Though esophageal reflux is the

most likely cause of an underlying stricture, malignancy must be excluded by upper


endoscopy. If her symptoms suggested oropharyngeal dysphagia, with immediate
onset and nasal regurgitation or choking, a barium swallow would be the first step in
her workup. Esophageal manometry is the test of choice for confirming a diagnosis of
achalasia, and can be performed in patients with esophageal dysmotility and a
normal upper endoscopy. Surgical myotomy is a treatment for cricopharyngeal
spasm, a cause of oropharyngeal obstruction. Myotomy is also a treatment option for
patients with achalasia.
Calcium channel blockers are used to prevent esophageal spasm and improve
esophageal emptying in patients with achalasia when dilation or myotomy are
contraindicated or unsuccessful.
Primary Care Medicine:Question 5
The correct answer is A
Educational Objectives
Understand the effect of lifestyle interventions for hypertension.
Critique
A recent meta-analysis has reconfirmed that aerobic exercise (30 minutes of exercise
vigorous enough to cause a sweat) significantly reduces blood pressure. Each of the
other interventions lowers blood pressure modestly, but not as much as aerobic
exercise, which lowers blood pressure in both hypertensive and normotensive
patients. Weight loss lowers blood pressure by about 1 point systolic and diastolic for
every kg lost. Other lifestyle interventions of importance include sodium restriction
(the average American consumes 10 g of salt per day; 4 grams per day is a noadded-salt diet), and adequate dietary potassium and calcium intake. Limiting
alcohol to 1 drink per day is also recommended, as is smoking cessation.
Primary Care Medicine:Question 6
The correct answer is A
Educational Objectives
Recognize the association between HIV infection and severe seborrheic dermatitis.
Critique
This patient presents with extensive seborrheic dermatitis, which has been minimally
responsive to low-potency corticosteroid therapy. Treatment with an antifungal
shampoo or with combination of topical antifungal and a low-potency corticosteroid
cream are appropriate treatment options. However, it is important to recognize the
possibility that the seborrheic dermatitis may be a manifestation of underlying HIV
disease and to assess for HIV risk factors. Seborrheic dermatitis is a common
cutaneous finding in patients with HIV disease. In one study 23.8% of HIV-infected
patients were found to have seborrheic dermatitis at baseline. The severity of
seborrheic dermatitis has been correlated with increasing progression of disease.
Recognition of refractory seborrheic dermatitis or extensive disease should prompt
clinical evaluation for the possibility of underlying HIV disease.
Primary Care Medicine:Question 7
The correct answer is B
Educational Objectives
Perform appropriate preventive interventions for women patients.
Critique
This patient is a healthy woman whose main purpose for her office visit is to get a
check-up. Because she is between 40 and 50 years old, she should have a
mammogram every 1 to 2 years. Although one recent study called the value of such
screening into question, the majority of the evidence suggests that there is a benefit

to screening women in their 40s, and the updated US Preventive Services Task Force
report recommends such screening.
Because the patient has never been sexually active, she does not need cervical
cancer screening. Some women choose to have such screening nonetheless; if she
desired cervical cancer screening, the appropriate interval given her negligible risk
(assuming her sexual history is true) would be every 3 years or more.
Colon cancer screening should begin at age 50 years unless there is a strong family
history of colon cancer developing in the 50s or earlier; this patients father
developed colon cancer at age 75 years, and so she does not need to be screened at
this time.
In addition, the patient is not at significantly elevated risk of ovarian cancer. Her
mother did not have ovarian cancer, but a maternal aunt did, in her 60s. Screening
for ovarian cancer with ovarian ultrasonography and measurement of serum CA-125
has not been shown to be beneficial.
Primary Care Med icine:Question 8
The correct answer is B
Educational Objectives
Recognize medication-induced constipation.
Critique
The patients history and findings are typical for mild to moderate constipation. Initial
measures should include an exercise program and increased fiber and fluid intake.
Bulk laxatives with adequate fluids are safe. Osmotic laxatives are safe for patients
who do not respond to these initial interventions; stimulant laxatives are generally
not used for first-line therapy or for long term. Many medications can cause or
exacerbate constipation. Among these, diuretics are notorious. Hydrochlorothiazide
may have precipitated constipation in this case, and her constipation is not
controlled. Therefore a switch to a nondiuretic antihypertensive medication is
indicated. Usually, colonoscopy and colonic transit studies are reserved for patients
who do not respond to initial measures, including treatment of diseases and
adjustment of medications that are associated with constipation.
Primary Care Medicine:Question 9
The correct answer is D
Educational Objectives
Treat erectile dysfunction in an older man with multiple medical problems.
Critique
This patient likely has an organic (vascular and/or neurogenic) cause of his erectile
dysfunction, and prostaglandin therapy (alprostadil is a synthetic form of
prostaglandin El), delivered either intra-urethrally or via intracavernous injection,
has been shown to be effective in men with erectile dysfunction from various organic
causes. lntra-urethral therapy is a reasonable choice for this patient, and the first
application should be undertaken in the physicians office because of the potential for
complications that include urethral bleeding, vasovagal response, hypotension, and
priapism.
Although diminished libido raises the possibility of hypogonadism, this patients
normal testes and normal testosterone level weigh against this diagnosis and against
a trial of testosterone replacement therapy. Sildenafil, usually a first-line agent to
treat organic erectile dysfunction, is absolutely contraindicated in men taking nitrate
therapy due to potentiation of blood pressure-lowering effects of nitroglycerin, and
therefore is not an option for this patient taking isosorbide. Although the patients
history raises the possibility of depression, he lacks vegetative symptoms or
anhedonia and he clearly has vascular disease and may also have a neurogenic

contribution to his erectile dysfunction (from his diabetes). Depression is associated


with erectile dysfunction, but without further signs or symptoms of depression a trial
of an antidepressant is not warranted (and SSRI agents might worsen sexual
dysfunction). Yohimbine has shown some efficacy in men with nonorganic erectile
dysfunction but is not recommended for men with organic erectile dysfunction
because its effect appears to be marginal in such cases.
Primary Care Medicine:Question 10
The correct answer is E
Educational Objectives
Evaluate a breast lump in the setting of a normal mammogram.
Critique
Every distinct breast lump should be followed to resolution or to definitive diagnosis.
It is not appropriate to reassure a woman that a breast lump is benign solely on the
basis of physical examination characteristics and/or a normal mammogram report.
This type of reassurance, along with failure to evaluate further or to arrange close
follow up, is a leading cause of physician-generated delay in diagnosis of breast
cancer. Younger women and women who present with pain or a self-identified lump
are less likely to receive evaluation to resolution or definitive diagnosis than are
older women and women whose abnormal findings were detected by a physician or
by mammography.
A lump without obvious features of malignancy may be followed through one
menstrual cycle. Findings that are of concern for cancer include overlying skin
changes or retraction; bloody nipple discharge; a lump that is rock-hard, fixed, or
has an irregular surface on examination; or spiculated calcifications on mammogram.
Resolution in one cycle suggests a cyst, whereas persistence or growth clearly
indicates the need for further evaluation. A cyst can be proven by two routes:
aspiration of nonbloody cyst fluid (with complete resolution of the palpable mass) or
visualization of a fluid-filled sac on breast ultrasonography. Distinguishing between
normal and abnormal solid breast tissue can be more problematic. Mammography
examines the breasts for certain focal radiographic characteristics suggestive of
malignancy but cannot diagnose cancer or prove its absence. Mammography has a
false-negative rate of 25% in premenopausal women and 10% in postmenopausal
women. A normal mammogram should never preclude further evaluation of a focal
breast finding. Special mammographic views are used if a visualized abnormality
needs further characterization, but are not useful if the standard views are normal.
Unless proven to be a cyst, a persistent palpable mass should be sampled by a
physician skilled in fine-needle aspiration (FNA) of the breast. FNA is more sensitive
and specific than clinical breast examination and breast imaging but is subject to
false-negative results from sampling error. Combining information from the three
modalitiesexamination, imaging, and FNAre-evaluating in light of these results,
and following periodically until resolution or definitive diagnosis are standard in
evaluating breast lumps that do not resolve in one menstrual cycle.
Primary Care Medicine:Question 11
The correct answer is D
Educational Objectives
Apply likelihood ratios in physical diagnosis.
Critique
The positive likelihood ratio of a clinical finding represents probability of that finding
in patients with the condition of interest (in this instance, severe aortic stenosis) in
relation to probability of the finding in patients without the condition. Although
finding with a likelihood ratio greater than 1 .0 argues in favor of a diagnosis, values

below 2.0 and 4.0 are relatively weak indicators. A likelihood ratio of 2.0, for
example, means that the probability of the condition is approximately 15% higher
than among patients without the finding. In this case, the initial suspicion of severe
aortic stenosis would be low in the absence of any other physical findings or clinical
symptoms. A 15% increase in probability makes severe aortic stenosis a possibility,
albeit relatively unlikely. An echocardiogram would be warranted to exclude severe
aortic stenosis as well as lesser degrees of stenosis.
Primary Care Med icine:Question 12
The correct answer is D
Educational Objectives
Interpret genetic susceptibility testing in an asymptomatic patient.
Critique
Testing for genetic susceptibility offers substantial promise for presymptomatic
identification of persons at high risk for cancer, potentially allowing intensified
surveillance or prophylactic interventions. Internists are often confronted with
questions of the appropriate use and interpretation of genetic susceptibility tests in
asymptomatic patients.
This patient has a high risk for breast cancer by virtue of history alone. The Gail
model is a research-based clinical prediction rule that allows estimation of individual
probabilities of breast cancer based on nongenetic risk factors obtained from
personal and family history. These factors include age, age at menarche, parity, age
at first live birth, race, and presence of a first-degree relative (mother or sister) with
known invasive breast cancer. Based on her history, this patient has a 0.8% 5-year
risk for invasive breast cancer, compared with 0.4% 5-year risk in a woman of the
same age and race but without other risk factors. Her lifetime risk for invasive breast
cancer (up to age 90 years) is 19%; for women of the same age and race without
risk factors it would be 12.5%.
Given her family history alone, her chance of testing positive forBRCA1 or BRCA2
mutations is higher than average, 15% to 30%, though some studies suggest the
risk may be lower in nonreferral populations. The presence of either mutation in
conjunction with a family history of BRCA1/BRCA2
-positive breast cancer confers a substantially higher cumulative lifetime risk of
breast cancer - 19% by age 40, 50% by age 50, and 85% by age 70. These
estimates are most reliable when the BRCA mutation status of the relative with
breast cancer is known. For this reason, most experts recommend testing the
affected individual before testing at-risk unaffected family members.
In the presence of BRCA1 or BRCA2 positive tests, the risk for breast cancer is so
much higher than baseline that screening mammography beginning at age 25 to 35
years, and yearly intervals has been recommended. However, there is no evidence of
a mortality benefit to this approach.
Primary Care Medicine:Question 13
The correct answer is C
Educational Objectives
Recognize categories of diabetic retinopathy and need for urgent referral in patients
with proliferative retinopathy.
Critique
This patient has findings of proliferative retinopathy with new vessels on the disk.
Urgent referral to ophthalmology for laser therapy is required to protect vision. Laser
photocoagulation of proliferative retinopathy has been shown to prevent blindness;
in one large trial, there was a 50% reduction in severe vision loss after 5 years.
Proliferative retinopathy refers to new vessel formation near the disk or elsewhere on

the retina; these fragile vessels become surrounded by fibrous tissue and adhere to
the posterior vitreous, and may cause vitreous hemorrhage or retinal detachment.
The patient also had hemorrhages and exudates, which are part of background, or
nonproliferative, diabetic retinopathy. However, routine referral to ophthalmology is
not a correct choice given the findings of proliferative retinopathy as noted above.
Guidelines for routine screening from the American Diabetes Association are as
follows: Ophthalmology referral for screening retinal examination is recommended
annually for patients with type 1 diabetes of more than 5 years duration, but not
before puberty. Recommendations for type 2 diabetes include annual ophthalmology
examination from the time of diagnosis. However, if the patient with type 2 diabetes
does not have proteinuria or severe hyperglycemia and has no evidence of
retinopathy by stereoscopic photographs, then the next screening examination may
be done in 4 years, with annual examinations thereafter even if no retinopathy is
identified.
Patients with macular edema also have been shown to benefit from laser therapy of
the macula to prevent vision loss, but the retinal examination described did not
include perimacular exudates as seen in macular edema. Rapid aggressive control of
glycemia in this patient could worsen the retinopathy.
Primary Care Med icine:Question 14
The correct answer is D
Educational Objectives
Manage abnormal PSA results.
Critique
Although not all experts recommend routinely checking serum PSA in men, the cutoff for abnormal values is 4.0 ng/mL. For those with an abnormal PSA, further
evaluation by a urologist is appropriate. Although his value of 5.0 ng/mL is just the
abnormal range, if the goal of screening is to find early prostate cancer then values
between 4 and 10 ng/mL deserve further testing.
When the PSA is greater then 10 ng/mL, the likelihood of locally or distally advanced
prostate cancer (stage C or D) is much greater when compared to patients with a
PSA from 4 to 10. The ability of the PSA test to discriminate between prostate cancer
and benign causes of elevated PSA values is only fair, but the specificity of the test is
better for men in their 50s when compared to men in their 70s. Tamsulosin is
prescribed for men with benign prostatic hyperplasia, but this patient has no
symptoms to suggest this disorder.
Population-based studies have suggested that a PSA increase of greater than 0.75
ng/mL per year is associated with prostate cancer. With a PSA level greater than 4
ng/mL, referral to a urologist would be preferred over further serial testing. Testing
for free PSA would provide further risk information. However, even if this patients
free PSA is greater than 25%, studies suggest his risk of prostate cancer is still 8%.
Most physicians would recommend urology evaluation. In areas of uncertainty such
as this, patient preference plays a large role in decision-making.
Primary Care Medicine:Question 15
The correct answer is B
Educational Objectives
Treat venous stasis ulcers.
Critique
Although the type of bandage does not seem to affect healing efficacy, patients
report significantly less pain when occlusive dressings (containing hydrogels or
hydrocolloid) are used compared with zinc paste-impregnated (Unna boot) or
nonadherent dressings, although this has not been studied as a primary end point.

Topical antibiotics are ineffective in this setting and complications include antibiotic
resistance and contact dermatitis. If pain is worsening as a sign of local infection,
system antibiotics are warranted. The comparative effect of Vaseline gauze has not
been studied. Of note, eutetic mixture of a local anesthetic (EMLA), a topical
anesthetic used during debridement does decrease ulcer pain but may impair ulcer
healing, and may also cause excess local skin reactions.
Primary Care Medicine:Question 16
The correct answer is B
Educational Objectives
Recognize the indications for and side effects of isotretinoin therapy for acne.
Critique
Scarring acne with cysts in a patient taking oral contraceptives is an indication for
isotretinoin therapy. Isotretinoin is highly teratogenic, and therefore, her intentions
about pregnancy and her willingness to use two forms of contraception and have
monthly pregnancy tests during treatment are key to whether she can take this
drug. Pancreatitis is a risk during therapy, and plasma triglyceride levels must be
monitored. Although alcohol is not contraindicated during therapy, it should be used
with caution. Finally, patients should be cautioned to avoid sun-exposure during
therapy because isotretinoin is sun-sensitizing.
Primary Care Medicine:Question 17
The correct answer is B
Educational Objectives
Recognize when it is appropriate to work-up an anal fissure.
Critique
This patient should undergo colonoscopy. The fissure is not in the posterior midline,
the usual location for an anal fissure. Fissures that do not occur in the posterior
midline are more likely to be due to a secondary cause. In this patient with weight
loss in addition to anal pain and bleeding, the possibility of Crohns disease should be
considered. Anal fissures in patients with Crohns disease also frequently occur in the
posterior midline (66%), but multiple fissures or fissures away from the posterior
midline (32%) are not uncommon. Early detection of underlying Crohns disease is
important because unhealed fissures in Crohns disease frequently progress to more
complicated and serious problems including anal abscess or fistula.
Primary Care Medicine:Question 18
The correct answer is A
Educational Objectives
Recognize appropriate pharmacologic management of alcohol dependence.
Critique
This patient presents with a history of alcohol dependence. Brief interventions work
for patients with at-risk alcohol use, but more aggressive therapy, potentially
including pharamcotherapy, is indicated in both alcohol abuse and dependence.
Benzodiazepines such as diazepam would be used in the acute detoxification setting.
Antidepressants and anxiolytics may play a role if an underlying psychiatric disorder
is present. Disulfiram has been used for years by leading to an accumulation of
aldehyde if alcohol is consumed. However, studies have been inconclusive on its
efficacy in enhancing abstinence. Naltrexone, an opioid receptor antagonist, has
been shown to be effective in short-term treatment as well as decreasing the
frequency of relapse.

Primary Care Medicine:Question 19


The correct answer is D
Educational Objectives
Understand the appropriate screening interval between Pap smears.
Critique
In this patient at low risk for cervical cancer who is in a long-term monogamous
relationship and has had many normal smears it is appropriate to lengthen the
screening interval for Pap tests to every 3 years. Her last Pap test was 1 year ago, so
her next test is due in 2 more years. Because cervical cancer is caused by a sexually
transmitted disease (human papillomavirus infection), women can be categorized
into cervical cancer risk groups in large part based on their sexual histories. In
addition, cervical cancer is slow-growing and has known and recognizable
precursors. Annual Pap tests do not identify more invasive cancer than tests
performed every 2 or 3 years in low-risk women who have had a number of normal
tests.
Women who have HIV infection should be screened more frequently, because human
papillomavirus can grow faster and lesions can progress more quickly in significantly
immunosuppressed patients.
Women who have multiple sex partners, a history of abnormal Pap tests, or have
recently been diagnosed with a sexually transmitted disease are at higher risk of
cervical cancer, and current guidelines suggest screening such women annually.
Primary Care Medicine:Question 20
The correct answer is E
Educational Objectives
Diagnose celiac sprue.
Critique
Celiac sprue is a T cell-mediated inflammatory reaction to gluten, a protein in wheat,
barley, rye, and oats, and results in small-bowel villous cell loss and subsequent
malabsorption. Affected children and young adults present with abdominal cramps,
weight loss, diarrhea, and lassitude. The diagnosis is a consideration in adults who
have unexplained folate deficiency, iron deficiency, low albumin, or elevated results
of liver function tests. Testing for IgA antiendomysial antibodies (EMA) to tissue
transglutaminase is a diagnostically useful test for patients presenting with diarrhea,
weight loss, and nutritional deficiencies. False-negative EMA testing occurs in
patients with celiac sprue eating a low-gluten diet. Small-bowel biopsy is mandatory
to confirm the diagnosis.
Celiac sprue may be associated with dermatitis herpetiformis, Howell-Jolly bodies,
human leukocyte antigen (HLA)-DR3, HLA-DQw2, and a small increased risk of
intestinal lymphoma and adenocarcinoma of the small bowel. Dermatitis
herpetiformis is a pruritic, papulovesicular eruption on the extensor surfaces, back,
and intergluteal folds. Skin biopsy reveals IgA deposits.
Treatment of celiac sprue is a gluten-free diet. Rapid symptomatic improvement
occurs in most patients within of weeks of the exclusion of dietary gluten. Intestinal
biopsies and EMA concentrations return to normal. Dermatitis herpetiformis responds
to dapsone and a gluten-free diet.

Primary Care Medicine:Question 21


The correct answer is D
Educational Objectives
Reduce the risk of fall-related injuries among older adults at increased risk for falls
and fractures.
Critique
Although the above interventions have not been compared head to head, available
evidence indicates that impact-absorbing external hip protectors can reduce the risk
of hip fracture by 60% (and by 80% or more when worn consistently by the user)a
risk reduction that exceeds that reported for the other options. A recent editorial
(Rubenstein) concluded that hip protectors offer a powerful new method to reduce
the risk of hip fractures and that their use should be strongly encouraged for persons
at increased risk (for example, those with osteoporosis and high fall risk). Although a
recent randomized trial raised questions about the efficacy of hip protectors, the key
limiting factor in their efficacy appears to be patient compliance (van Schoor et al).
Efficacy in preventing hip fracture appears to be high when hip protectors are in
place at the time of a fall.
Physical therapy is an important component of multifactorial assessments and
targeted interventions to reduce the risk of recurrent falls and fall-related injuries.
However, while such interventions can significantly reduce the incidence of falls (by
10% to 30%) they do not prevent the majority of falls and provide only partial
protection against hip fracture. Treatment of osteoporosis is a key intervention to
reduce the risk of hip fracture and also confers protection for other sites of fracture
besides the hip. Although integral to reducing fall-related fractures, the relative risk
reduction in hip fractures ascribed to osteoporosis treatment is less than that noted
above for hip protectors (clinically both should be strongly considered). Use of
restraints is generally contraindicated as there is little evidence demonstrating their
efficacy in reducing falls and substantial evidence of injuries resulting from
restraints. Addressing sensory deficits is an important component of multifactorial
interventions to reduce fall risk, but evidence indicating improved hearing can reduce
fall risks is lacking.
Primary Care Medicine:Question 22
The correct answer is B
Educational Objectives
Review barrier contraceptive methods available to a person with latex allergy.
Critique
This woman intends to continue this relationship and needs effective STD protection,
but is at risk for serious manifestations of latex allergy. She should not go without
STD protection, given the very real risks to her health long term. Abstinence would
be most effective but is not acceptable to her. A test-and-treat strategy would only
be reasonable if STDs were all harmless and curable. Despite past reports of efficacy,
nonoxynol-9 does not prevent transmission of HIV and other STDs. Polyurethane
male and female condoms are available over the counter, and use of either
represents the most reasonable approach to reduce her risk for STDs. However,
some studies show a higher rate of breakage or slippage of the polyurethane
condoms compared with latex. She should be counseled that no barrier method is
foolproof for preventing transmission of all STDs.
For patients who need or want alternatives to latex diaphragms and cervical caps,
the intravaginal contraceptive sponge is again available over the counter, and
silicone cervical caps are available in many countries but not yet in the United
States.

Primary Care Medicine:Question 23


The correct answer is B
Educational Objectives
Recognize the risk of depression associated with age and death of a spouse.
Critique
Depression among the elderly, especially after the loss of a spouse, is common (15%
to 35% in the first year). The presentation of depression in ambulatory care is often
atypical and consist of physical rather than psychosocial symptoms. This often
contributes to the difficulty of assessing patients for depression and getting them to
accept the diagnosis when it is made. Although the patient may have been
dependent on his wife to cook for him, there is not enough evidence to assess
whether the patients personality type is dependent. His poorly controlled blood
pressure may reflect poor compliance related to underlying depression but is
probably unrelated to his symptoms. There is no mention made by the patient of
recurrent re-living of the loss; thus post-traumatic stress is not a likely explanation.
Finally, anniversary grief reaction is generally understood to mark a specific
milestone in the grieving process (6 months, 1 year) and is characterized by a
recognizable shift in understanding or symptoms. No such marker event is present in
this scenario.
Primary Care Medicine:Question 24
The correct answer is E
Educational Objectives
Recognize and treat common warts.
Critique
Punch biopsy may be useful if the diagnosis is uncertain, but the location and
appearance of the lesions described are typical of verruca vulgaris, or common
cutaneous warts. Warts, caused by infection with subtypes of the human papilloma
virus, are common (occurring in up to 24% of teenagers) and frequently occur on
the hands and feet. They are typically verrucous in appearance, but may be flat and
smooth. A rapid plasmin reagin test would be a useful test to diagnosis secondary
syphilis in the appropriate clinical context. Although the macular rash of secondary
syphilis may occur on the palms and soles, this rash is not typically verrucous in
appearance. Condyloma lata, another skin lesion seen in secondary syphilis, is
typically smooth and velvety in appearance and most commonly seen in the
anogenital region.
The treatment of cutaneous warts remains poorly studied. Warts commonly resolve
spontaneously, although this patient experiences discomfort in his usual activities
related to the wart on his index finger. Topical application of salicylic acid, in the
form of liquid, plaster, cream, or ointment, eradicates cutaneous warts in 60% to
80% of patients. Cryotherapy with liquid nitrogen is of similar efficacy, and may be
preferable for facial warts where scarring is of concern, but should be used with
caution on warts on the digits because of a risk of painful neuralgia. Imiquimod, a
recently released topical immunotherapy, is effective for genital warts, but is
expensive and unproven for use in common cutaneous warts.
Primary Care Medicine:Question 25
The correct answer is C
Educational Objectives
Distinguish features of iritis from benign conjunctivitis.
Critique
This patient has acute iritis, or anterior uveitis, which is often mistaken for benign
bacterial or viral conjunctivitis and inappropriately treated with topical antibiotics.

The clues to diagnosis of iritis in this patient include eye pain rather than irritation,
lack of discharge, decrease in visual acuity, and ciliary flush described by
circumcorneal erythema on examination. About half of patients with iritis have an
underlying inflammatory arthritis or chronic infection. The patient has features
suggestive of Reiters syndrome. Acute iritis also occurs in association with HLA-B27
without systemic disease. Urgent ophthalmology referral is required for diagnostic
examination and management of iritis, usually treated with a topical corticosteroid
such as prednisone forte. Corticosteroids should not be given empirically in the
primary care setting without establishing a definitive diagnosis. Complications of iritis
include cataract, vision loss, chronic uveitis, and synechiae (adhesions between the
iris and lens capsule).
Keratitis may be associated with mild visual impairment and eye pain, except for
herpes keratitis in which there is hypoesthesia of the cornea. Characteristic skin
lesions of herpes simplex or herpes zoster are usually present with herpetic keratitis.
Keratitis in the setting of extended wear contact lenses is usually due to
Pseudomonas infection and presents with a copious purulent discharge. Copious
purulent discharge is an indication for emergent ophthalmology referral.
Primary Care Medicine:Question 26
The correct answer is D
Educational Objectives
Select the appropriate malaria chemoprophylaxis regimen for a traveler.
Critique
Mefloquine is contraindicated in patients with cardiac conduction disease, and
therefore, should not be prescribed for this traveler. The US Centers for Disease
Control and Prevention (CDC) recommends chloroquine for areas with chloroquinesensitive malaria, such as Mexico. Chloroquine prophylaxis should be started 1 week
before departure, taken 500 mg weekly while traveling, and continued for 4 weeks
after return. Chloroquine has a low rate of adverse effects and is inexpensive.
Doxycycline would provide protection but requires daily administration, causes sun
sensitivity, and nausea. Atovaquone/proguanil (MalaroneTM) is recommended for
travel to areas with chloroquine-resistant malaria. It is relatively expensive.
Primaquine is recommended after prolonged exposure to decrease the risk of
Plasmodium ovale and P. vivax
Primary Care Medicine:Question 27
The correct answer is D
Educational Objectives
Recognize that for uninfected grade 1 and 2 foot ulcers in patients with diabetes total
contact casting improves healing.
Critique
The main components of treating diabetic foot ulcers are to alleviate weight-bearing,
debride exudate and infected tissue, and treat associated infection. In this patient,
there are no signs of infection so relief of pressure on the foot is the mainstay of
therapy. For patients who find complete bed-rest difficult, a total contact cast keeps
pressure off the ulcer and thereby significantly increases the proportion of ulcers that
heal and shortens healing time. Total contact casts are contraindicated if there are
any signs of infection in the ulcer, or if osteomyelitis is present; therefore, the
presence of warmth, erythema, or exudate or exposed or infected bone precludes
their use. Neither topical antibiotics nor occlusive dressings improve healing of
diabetic foot ulcers. Intravenous antibiotics are not indicated in grade 1 and 2 ulcers
without signs of infection; such ulcers if infected may not require hospitalization
because they are likely to respond to outpatient antibiotic therapy, nonadhesive

dressings, and twice-daily debridement. Zinc paste-impregnated bandages (Unna


boots) are placed on patients with venous ulcers to reduce edema and encourage
ulcer healing.
Primary Care Medicine:Question 28
The correct answer is D
Educational Objectives
Choose topical therapy for a patient with comedonal acne.
Critique
The combination of 5% benzoyl peroxide and 1% clindamycin gel is the first
therapeutic step in this young man. Benzoyl peroxide and clindamycin would likely
be the most effective therapy and should be tried for 2 to 3 months before adding
topical tretinoin or initiating oral antibiotics because he will likely develop resistant
bacteria with prolonged oral therapy. Oral tetracycline will likely be more sunsensitizing but might be appropriate for someone with extensive involvement of the
body surface. He should use noncomedogenic sunscreen.
Primary Care Medicine:Question 29
The correct answer is B
Educational Objectives
Recognize the typical presentation of subarachnoid hemorrhage and select the
appropriate early
diagnostic test.
Critique
This patient has a clinical picture suggestive of subarachnoid hemorrhage. He is at
the mean age for aneurysm rupture and has sudden onset of a severe headache with
neck involvement. An important historical feature is that he has no previous
headache history, which makes an alternative diagnosis much less likely. The
appropriate first test for suspected subarachnoid hemorrhage is a noncontrast CT
scan of the head. If CT scanning takes place within 24 hours of onset of symptoms,
95% of subarachnoid hemorrhages are detected with the noncontrast scan. The
chance of a subarachnoid hemorrhage being present in a patient with a negative
noncontrast scan is very low, 1 .9% in a recent study of patients who presented with
worst headache of their life. Because of the catastrophic possibilities if a
subarachnoid hemorrhage is missed, lumbar puncture should be done if noncontrast
CT is negative in patients with high pre-CT scan probability. MRI scans are not as
effective as noncontrast CT in diagnosing acute subarachnoid hemorrhage.
Primary Care Medicine:Question 30
The correct answer is C
Educational Objectives
Treat acute rhinosinusitis.
Critique
This patient presents with a classic case of viral rhinosinusitis. In 87% of viral upper
respiratory infections, there is radiologic evidence of sinusitis. However, less than
2% to 3% of these infections progress to a bacterial infection. Therefore, the most
appropriate therapy includes decongestants, analgesics, and, potentially,
antihistamines. Factors that make a bacterial infection more likely include duration
longer than 7 days, purulent nasal discharge, worsening of symptoms after initial
improvement, tooth pain, and failure to improve with decongestants. Systemic
corticosteroid therapy would be over-aggressive, and nasal ipratropium would be
appropriate to reduce rhinorrhea.

Primary Care Medicine:Question 31


The correct answer is C
Educational Objectives
Diagnose and treat urticaria.
Critique
This history and rash are consistent with a diagnosis of chronic urticaria; the absence
of symptoms of angioedema is reassuring. In more than half of cases, the cause of
chronic urticaria cannot be found. First-line treatment of chronic urticaria is maximaldose antihistamines, which this patient has used without much success. Because
15% of histamine receptors in the vasculature are H2 receptors, adding an H2
receptor antagonist, such as ranitidine, to the Hi antagonist can be very effective.
Skin biopsy (to rule out vasculitis) is necessary only if the lesions are atypical in
appearance or if they last more than 24 hours in a single location. Dapsone has been
suggested for the treatment of chronic urticaria refractory to usual therapy, but this
patient has not been maximally managed yet. Prednisone should be reserved for
patients who have angioedema, and although it is effective in chronic urticaria, it is
not a first-line agent because of its side effects, especially in patients with diabetes.
Primary Care Medicine:Question 32
The correct answer is A
Educational Objectives
Provide patients at home with maneuvers that stop a nosebleed.
Critique
Epistaxis is often a frightening sight for patients. Maneuvers used at home stop
bleeding in most cases. Patients are instructed to sit up and lean forward at the waist
and breathe with the mouth open. This allows blood to flow out the nostrils rather
than down the throat. Patients should pinch the soft parts of the nose together for at
least 5 minutes. An ice pack (crushed ice in a plastic bag or washcloth) applied to the
nose augments vasoconstriction. Oxymetazoline can treat nosebleeds in 65% to 75%
of cases, thus precluding the need for nasal packing, but it should not be used daily.
Nasal packs with tissue or cotton plugs are used when these initial measures fail.
After the acute event, patients should take NSAIDs or aspirin products and should
consult their physicians. Nose ointments are effective agents to counter the mucosal
dryness that can lead to a nosebleed. Laboratory evaluation is not indicated in the
absence or major bleeding, recurrent bleeding, or suspected coagulopathy.
Primary Care Medicine:Question 33
The correct answer is B
Educational Objectives
Consider the most appropriate intervention(s) to decrease fall rates among patients
with recurrent falls.
Critique
Observational studies suggest that medications are among the most readily
modifiable risk factors for falls, with psychotropic agents (benzodiazepines, tricyclic
antidepressants, and neuroleptics) leading the list of commonly implicated drugs.
Specific single interventions that have been shown to reduce falls in randomized
controlled trials include withdrawal of psychotropic medications; home hazard
assessment and modification for patients with history of falling; and exercise
programs. Discontinuing this patients lorazepam therapy is the most appropriate
single intervention.
Falls are usually multifactorial, and therefore addressing all potential contributing
factors is important. However this patients vision is not markedly impaired (20/40),
and the lorazepam therapy is a more obvious and likely more important contributor

to her recurrent falls. Orthostatic hypotension may have contributed to her most
recent fall. However, on examination she did not have significant changes in blood
pressure or heart rate upon standing, and her two other falls were not related to
orthostatic positional changes. Thus the evidence to suggest that discontinuing her
hydrochlorothiazide would reduce her risk of recurrent falls is less compelling than
that for stopping the lorazepam. Referral for physical therapy is an important
consideration for all patients with recurrent falls but this patient did not have
unstable gait and she completed a timed Get Up and Go test in less than 20 seconds,
again making this choice less compelling than stopping her lorazepam. Use of
antidepressants has been associated with increased fall risk but much of this
association may be due to confounding by indication, and there is no existing
evidence to suggest that switching sertraline to buspirone would decrease her fall
risk.
Primary Care Medicine:Question 34
The correct answer is D
Educational Objectives
Review methods of contraception which lessen menstrual flow.
Critique
This perimenopausal woman who smokes desires reduction in menstrual flow and
more effective contraception, without chance of weight gain. Combination hormonal
contraceptives would most likely be effective, but are contraindicated in her because
heavy smoking significantly increases the risk of myocardial infarction, especially in
women taking oral contraceptive pills. (The new transdermal formulation should be
assumed to have this same potential, in the absence of data showing otherwise.)
Though quitting smoking is clearly advisable, she sounds unlikely to make that
change at this time. Depot medroxyprogesterone acetate would also likely be
effective but commonly causes weight gain, the possibility of which would be
unacceptable to her. The levonorgestrel intrauterine device would be the best of
these options in this womans situation. It reduces menstrual blood loss in idiopathic
menorrhagia, provides effective contraception, has no or few systemic side effects,
and can be left in for 5 years. Endometrial ablation would be another option but has
a lower success rate at reducing menstrual blood loss than does levonorgestrel
intrauterine device, and ablation may not prevent pregnancy.
Primary Care Medicine:Question 35
The correct answer is E
Educational Objectives
Recognize fibromyalgia and understand its prognosis.
Critique
This case focuses on a patient with classic symptoms of fibromyalgia (widespread
musculoskeletal pain for more than 6 months and excess tenderness in at least 11 or
18 predefined anatomic sites). In addition to her symptoms, which have been
present for 18 months, she has also undergone several specialty work-ups, all with
negative results. The literature suggests that this in not an uncommon pattern for
patients with fibromyalgia and that further work-ups have a low likelihood of
producing positive findings. The diagnostic criteria mentioned above have been
validated in randomized controlled trials and are a basis on which to base a positive
diagnosis. There is currently no known cause for fibromyalgia, nor is there any cure.
Patients with the symptom pattern presented by this patient do not often improve by
themselves but rather continue to worsen over time. Fibromyalgia is not simply a
psychological syndrome but a biopsychosocial one and supportive care aimed at
symptom relief and coping strategies is important. Behavioral therapy and/or

pharmacotherapy have had demonstrated efficacy in treating fibromyalgia.


Additionally, the literature suggests that a strong physician-patient relationship with
a realistic recognition that treatment may be long term and involve an integration of
biological, psychological, and social factors is an important dimension of successful
treatment.
Primary Care Medicine:Question 36
The correct answer is B
Educational Objectives
Recognize the appropriate endocarditis prophylaxis for high-risk patients undergoing
high-risk
procedures.
Critique
This patient is considered high-risk for developing perioperative endocarditis because
of her prior episode of endocarditis. Other high-risk patient groups include those with
prosthetic heart valves, complex cyanotic congenital heart disease, and surgically
reconstructed systemic pulmonary shunts. She is undergoing a procedure that puts
her at increased risk for bacteremia (genitourinary or nonesophageal gastrointestinal
surgeries). Typically, such patients are treated with ampicillin and gentamicin before
surgery. In patients with penicillin allergies, vancomycin is substituted for ampicillin.
Aminoglycosides could be omitted before high-risk surgeries in patients at moderate
risk for endocarditis, such as those with hypertrophic cardiomyopathy, mitral valve
prolapse with valvular regurgitation and/or thickened leaflets, or less severe
congenital cardiac states. Oral regimens such as amoxicillin, cephalexin, clindamycin,
or azithromycin may be used before low-risk procedures, such as dental, oral,
respiratory tract, orthopedic, or esophageal surgeries. Oral regimens are typically
given 1 hour before surgery, whereas intravenous medications are administered to
complete within 30 minutes before surgery.
Primary Care Medicine:Question 37
The correct answer is D
Educational Objectives
Apply predictive value in screening.
Critique
Positive predictive value represents that probability of disease given a positive
screening test. In this case, the likelihood of carcinoma is approximately 30%, that
is, 70% of men who do not have cancer would undergo further work-up
unnecessarily. Moreover, a negative test does not exclude the presence of cancer.
These uncertainties about interpreting the results of PSA screening, combined with
information about the risks and benefits of the evaluation of elevated PSA values and
of treatment for prostate cancer should be conveyed to the patient so that he can
make an informed choice about whether to undergo screening.
Primary Care Medicine:Question 38
The correct answer is B
Educational Objectives
Recognize and manage corneal trauma.
Critique
This patient has a corneal abrasion from ocular trauma that occurred with a paintball
air gun, an increasingly common source of eye injuries. Corneal abrasions usually
have an associated history of trauma, industrial or occupational etiology, and are
also common in the perioperative setting. A controlled trial of corneal abrasions in
patients with trauma without infection or contact lens wear demonstrated that eye

patch is not beneficial. In fact, patients who wore an eye patch had more eye
discomfort and slower wound healing. Topical anesthetic is not indicated as it delays
healing, masks further damage, and can cause corneal ulceration. Topical antibiotics
are often prescribed in corneal abrasion, depending on the type of trauma and extent
of injury, but oral antibiotics are not routinely indicated. Frequency of follow-up
depends on the severity of the corneal abrasion. Severe abrasions with large area of
epithelial damage should be examined daily, and a mild abrasion may not require
repeat examination if symptoms resolve and there is no residual visual impairment.
Topical corticosteroids are not indicated.
Primary Care Medicine:Question 39
The correct answer is B
Educational Objectives
Recognize the significance of anosmia in a man with delayed pubertal development.
Critique
Primary care physicians should be able to differentiate common causes of anosmia
from more serious neuroendocrine conditions. This young man most likely has
Kallmans syndrome, a disorder characterized by hypothalamic hypogonadism and
anosmia. The anosmia is due to olfactory bulb agenesis or hypoplasia. The syndrome
may also be associated with color blindness, optic atrophy, nerve deafness, cleft
palate, renal abnormalities, cryptorchidism, and neurologic abnormalities such as
mirror movements. Defects in the KAL gene, which maps to chromosome Xp22.3,
prevent embryonic migration of gonadotropin releasing hormone (GnRH) neurons to
the hypothalamus. GnRH deficiency prevents progression through puberty. Males
present with delayed puberty and pronounced hypogonadal features, including a
small penis. Female patients present with primary amenorrhea and delayed pubertal
development.
Laurence-Moon-Bardet-Biedl syndrome is a rare autosomal recessive disorder is
characterized by mental retardation, obesity, and hexadactyly, brachydactyly, or
syndactyly. Central diabetes insipidus may or may not be associated. GnRH
deficiency is common. This patient does not have these features.
Ozena is a chronic nasal condition, associated with colonization or infection
withKlebsiella ozenae. It is characterized by nasal mucosal atrophy and foul-smelling
crusts in the nasal passages, typically seen on close nasal examination. Some
patients with Parkinsons disease will develop anosmia; this patient has no
parkinsonian features.
Primary Care Medicine:Question 40
The correct answer is D
Educational Objectives
Evaluate a patient with pruritus for a potential systemic disorder.
Critique
Although most patients with generalized pruritus have xerosis, this patient did not
improve with empiric therapy and scabies is unlikely without a characteristic rash. A
minority of patients with generalized pruritus have an underlying disorder, which is
more likely in this patient with no skin lesions other than excoriations. Laboratory
testing for hematologic causes, renal failure, cholestasis, endocrine disorders, and
chronic infection is indicated. Skin biopsy of an excoriation is unlikely to provide
useful information. Treatment with topical corticosteroids without a working
diagnosis is not appropriate. Symptomatic treatment with hydroxyzine hydrochloride
may help but does not identify the cause. Although referral to a dermatologist would
be helpful, the patient should first be evaluated for systemic causes of the pruritus.

Primary Care Medicine:Question 41


The correct answer is C
Educational Objectives
Recognize methods for removing cerumen from an occluded external ear canal.
Critique
Cerumen plugs that completely occlude the external ear canal are best approached
first with a ceruminolytic to soften the wax and loosen adherence to the ear canal.
Curettes are most effectively used when the occlusion is partial, allowing passage of
the curette behind the plug. Irrigation could be attempted, though it is also more
effective after use of a ceruminolytic and in the setting of a partial blockage where
the irrigation solution can reach past the plug to push it out. Additionally, irrigation
should be done with a 20-mL or smaller syringe to decrease the risk of rupturing the
tympanic membrane. Audiogram should be ordered only if the hearing loss persists
after removal of the cerumen. The patients social withdrawal may be a sign of
depression. Screening for depression is warranted, and if persistent after hearing
issues are addressed, then an antidepressant like fluoxetine may be warranted.
Primary Care Medicine:Question 42
The correct answer is D
Educational Objectives
Diagnose and treat androgenic acne.
Critique
This woman has polycystic ovary syndrome with mild hirsutism and acne. Thus, her
acne is androgen-based and topical therapies are unlikely to be ineffective. Similarly,
antibiotics may help, but she would likely develop resistance. As a first step, lowprogesterone oral contraceptives and weight loss should be recommended. A
combined estrogen and progestin contraceptive has been approved for use for acne,
although other oral contraceptives with low progesterone should work similarly.
Although metformin may help her polycystic ovary syndrome including normalizing
her ovulation and possibly reducing her acne, it is not first-line therapy.
Spironolactone may also be effective for the acne but would be less effective for her
menstrual irregularity.
Primary Care Medicine:Question 43
The correct answer is E
Educational Objectives
Recognize chronic daily headache and know the low yield of head imaging in affected
patients.
Critique
This patient has chronic daily headache (transform migraine, rebound analgesic
headache). It commonly occurs in patients with a migraine history and may be
partially due to analgesic overuse. This patient is using a large amount of analgesics,
including NSAIDs, triptans, and narcotics. Narcotics and barbiturates are strongly
associated with the development of analgesic rebound headache. Recently triptans
have been implicated as a cause of analgesic rebound headache. Imaging of affected
patients with a normal neurologic examination has a very low yield. In a recent
retrospective study of 402 patients with chronic daily headache and a normal
neurologic examination, 6% of patients with migraine, 1.4% of patients with tensionheadache, and no one with a mixed tension-migraine pattern had abnormalities. A
number of other studies have confirmed the extremely low yield (0-1%) of clinically
significant neuroimaging abnormalities in patients with chronic headaches who have
a normal neurologic exam.

Primary Care Medicine:Question 44


The correct answer is A
Educational Objectives
Manage a lipid disorder in a patient without significantly increased risk for coronary
artery disease.
Critique
This woman has no coronary artery disease equivalent (diabetes mellitus, abdominal
aortic aneurysm, symptomatic peripheral artery disease or coronary artery disease).
Her hypertension is her only risk factor, and the HDL cholesterol level greater than
60 mg/dL is a negative risk factor. Therefore, for the purposes of determining
whether she is a candidate for lipid-lowering therapy, she has 0-1 risks (these risks
are independent of LDL cholesterol level). Intervention should be pursued for a
serum LDL cholesterol greater than 160 mg/dL, with medication recommended if the
level is greater than 190 mg/dL and optional between 160 and 189 mg/dL. There are
no data on the benefit of lipid-lowering with medication in this low-risk population.
Primary Care Medicine:Question 45
The correct answer is E
Educational Objectives
Recognize and evaluate erythroplakia.
Critique
A velvety red mucosal abnormality is called erythroplakia. In someone with strong
risk factors for oral cancer, such as this patient, there is a high rate of carcinoma in
situ or frank cancer detected in a biopsy specimen of erythroplakia. If the
abnormality persists for longer than 1 month, it should be biopsied. In this case, the
appropriate next step after the 1-month follow-up is biopsy. Chewing tobacco is a
strong risk factor for erythroplakia, but there is no evidence that the lesion will
regress if the patients refrains from tobacco use.
Isotretinoin therapy may help with regression of lesions, but it is associated with
various side effects and should be prescribed only after a biopsy is negative. CT scan
is not an adequate study to assess small mucosal lesions of the mouth for malignant
features.
Primary Care Medicine:Question 46
The correct answer is A
Educational Objectives
Recognize the use of chiorpromazine to treat hiccups.
Critique
Many cases of hiccups begin by a combination of talking, eating, drinking, laughing,
or a mild digestive disturbance. Nearly all episodes of hiccups resolve spontaneously.
Intractable hiccups can be an indication of a serious underlying disease process and
should be investigated. Considerations include diseases of the central nervous
system (stroke, tumor), irritation of the phrenic nerve (foreign body, pharyngitis,
gastric distension or reflux), psychogenic stress, and toxins such as alcohol, and
rarely, medications. Initial investigation includes history, physical examination, and
chest radiograph. Laboratory evaluation is usually not helpful.
Chlorpromazine is approved for the treatment of hiccups. Because hiccups are
disturbing this patients quality of life, initial therapy with chlorpromazine is
indicated. Should symptomatic treatment fail, the clinician must consider atypical
symptoms of gastroesophageal reflux or other common problems such as asthma as
the source of hiccups. Without abnormal neurologic findings on the physical
examination, head imaging is unnecessary.

Primary Care Medicine:Question 47


The correct answer is A
Educational Objectives
Determine the cause and management for an older male patient with urinary
incontinence.
Critique
This patient has overflow incontinence based on his elevated post-void residual
volume of 100 mL. He is taking two medications that can decrease bladder
emptying: nortriptyline via its anticholinergic effects and sympathomimetic activity,
and codeine which can decrease bladder smooth-muscle contractility. The most
appropriate first step in managing this patient is to stop both medications and
reassess.
Even though he has a history of leakage with bending and sneezing these symptoms
can also occur with overflow incontinence and the elevated post-void residual volume
implicates overflow rather than stress incontinence. He may well have some degree
of prostatism contributing to his symptoms and stopping nortriptyline is appropriate;
however, codeine should also be stopped, and it is more appropriate to reassess
after stopping medications than to simultaneously proceed with a new medication.
Also, if a new treatment for prostatism were to be added, an -blocker would be
more effective and have a faster onset than finasteride.
Although this patient has glucose in his urine and a mildly elevated random serum
glucose, a diagnosis of diabetes cannot be established based on the laboratory data
provided. Furthermore, overflow incontinence due to diabetes-related neuropathy is
highly unlikely in this patient lacking other signs of autonomic or sacral nerve
dysfunction.
Primary Care Medicine:Question 48
The correct answer is D
Educational Objectives
Recognize and evaluate endometriosis.
Critique
Cyclic pelvic pain associated with dyspareunia suggests endometriosis. Pelvic
examination is usually normal, although nodularity is sometimes palpable in pelvic
structures (such as the uterine ligaments or the pouch of Douglas). Less commonly,
an endometrioma may be apparent as a mass, usually adnexal, on examination or
pelvic ultrasound. Imaging is otherwise normal. Definitive diagnosis is made
surgically, though the presence of endometriosis does not prove this is the cause of
pain, and microscopic disease can be missed unless biopsies are taken.
Endometriosis is found in at least 20% of women undergoing laparoscopy for pelvic
pain and in one third to one half of women undergoing laparoscopy for otherwise
unexplained infertility. Empiric hormonal therapy is reasonable when history
suggests endometriosis and the goal is symptom control. Leuprolide acetate induces
menopause temporarily, can be used for 3 to 6 months to quiet the endometrial
tissue, and is usually followed by another suppressive hormonal approach such as
continuous, systemic progesterone or combination estrogen/progestin. Hormonal
therapy and surgical therapy are probably equivalent in treatment of pain, though no
large comparison trials have been done.
In this case, fertility is impaired, with history suggesting her cycles are ovulatory and
with the male partners fertility proven in the past. A postcoital test and an
hysterosalpingogram would be advised as the next diagnostic tests in most cases,
but since this woman has cyclic pelvic pain suggestive of endometriosis and desires
pregnancy, laparoscopy is the best next step. In endometriosis, tubal anatomy is
typically normal on hysterosalpingogram unless the disease or subsequent adhesions

are extensive. Inflammatory substances present in peritoneal fluid of women with


endometriosis may be one route by which fertility is impaired. For infertility
associated with endometriosis, hormonal therapy to suppress the endometrial tissue
is ineffective, and laparoscopy to remove visible disease is indicated. If conception
does not then occur, treatment with clomiphene citrate (or gonadotropins) to
augment ovulation, followed by intrauterine insemination in the same cycle, can be
tried. The hormonal milieu of pregnancy, including high progesterone levels, is
beneficial for endometriosis.
Cyclic progesterone is used to protect the endometrium against the effects of
unopposed estrogen and induce withdrawal bleeding in women with anovulatory
cycles. Less commonly, it is used to prevent recurrent early miscarriage from
inadequate luteal production of progesterone. Neither of these situations is
suggested in this case. Hysterosalpingogram is a fluoroscopic study using contrast to
outline the uterine cavity and fallopian tubes and is used to examine for tubal causes
of infertility. Conception rates in infertile couples increase in the cycle in which
hysterosalpingogram is done, perhaps by clearing material or microscopic adhesions
from the tubes. While woman with advanced endometriosis may have tubal
abnormalities, in most cases the study is normal, and an hysterosalpingogram will
neither diagnose the cause of pelvic pain nor help (significantly) to treat the
infertility. Psychiatric evaluation and treatment may help address psychological
aspects of chronic pain and infertility, but is not diagnostic and will not improve
fertility. While a history of sexual abuse is reported to be common among women
with chronic pain, most of the studies with this finding used referral populations and
thus may be biased.
Primary Care Medicine:Question 49
The correct answer is B
Educational Objectives
Recommend appropriate treatment for fibromyalgia.
Critique
Treatment with anti-inflammatory, analgesic, and central nervous system-active
medications have all been tried with varying success in the management of
fibromyalgia. In a controlled clinical trial, ibuprofen had no beneficial effects over
placebo, and of the above pharmacologic options nonsteroidal anti-inflammatory
drugs appear to have the least effect, consistent with the lack of evidence that tissue
inflammation is present in patients with fibromyalgia.
Both amitriptyline and cyclobenzaprine have proven to be of some efficacy in
controlled trials. Patients should be started on low doses of either of these
medications taken 1 to 2 hours before bedtime. Tramadol, which combines analgesic
mechanisms mediated via weak opioid and monoaminergic actions, is widely used
and anecdotally highly effective in fibromyalgia. Although further evaluation in
controlled trials is required, tramadol causes minimal respiratory depression,
dependence, and tolerance and is, therefore, more appropriate for long-term regular
administration than other narcotic analgesics. Data regarding the efficacy of selective
serotonin re-uptake inhibitors (SSRIs) in treating fibromyalgia have been largely
disappointing, but two randomized controlled trials using fluoxetine have indicated
beneficial effects of this SSRI. One of these studies suggested that the combination
of a low-moderate-dose fluoxetine in the morning with low-dose amitriptyline in the
evening was more effective than either medication alone.

Primary Care Medicine:Question 50


The correct answer is C
Educational Objectives
Recognize the appropriate antibiotic prophylaxis before a clean-contaminated
gastrointestinal surgery.
Critique
The principles of surgical antibiotic prophylaxis are (1) maintain adequate tissue
levels of an appropriate antibiotic from incision to closure of a surgical wound; (2)
antibiotics should be given no more than 30 to 90 minutes before incision; (3)
antibiotics may need to be readministered intraoperatively to maintain adequate
tissue levels; and (4) prophylaxis should not exceed 24 hours unless procedures are
complicated, contaminated, or dirty.
Uncomplicated colorectal surgery is considered clean-contaminated; a contaminated
viscous is entered in a controlled fashion without spillage of colonic contents.
Contaminated procedures occur in the presence of acute inflammation, infected bile
or urine, or gross spillage from the gastrointestinal tract. Dirty procedures are
those that occur in the presence of established infection.
Cefotetan, or a similar second-generation cephalosporin, is recommended
prophylaxis for uncomplicated colorectal surgery because it covers typical skin flora
as well as the majority of organisms found in the gastrointestinal tract. A recent
systematic review concluded that first-generation cephalosporins, such as cefazolin,
were equally effective as second-generation cephalosporins. A single preprocedure
dose of prophylactic antibiotics is as effective as prolonged prophylaxis, although
antibiotics should be readministered intraoperatively for prolonged procedures.
Clindamycin and an aminoglycoside are recommended in patients with documented
cephalosporin allergies; a fluoroquinolone may be substituted for the aminoglycoside
in patients with impaired renal function.
Primary Care Medicine:Question 51
The correct answer is C
Educational Objectives
Diagnose and treat shoulder pain.
Critique
A diagnosis of rotator subacromial bursitis (or cuff tendonitis) is strongly suggested
by tenderness over the supraspinatus tendon, nocturnal pain, and pain on abduction
past 40 degrees. The absence of occupational trauma or other symptoms of
osteoarthritis makes that diagnosis unlikely, whereas the subacute onset and the
lack of any other symptoms make systemic inflammatory disease or an acute
inflammatory arthritis unlikely. A complete rupture of the rotator cuff is unlikely
because the patient can abduct the arm and hold it in an outstretched position. A
partial tear is possible but is difficult to assess because pain limits fully testing
strength.
The most sensible intervention at this point would be instillation into the bursa of
corticosteroids admixed with 3 to 4 mL of 1% lidocaine. Although meta-analyses
based on a limited number of relatively small trials indicate little difference between
NSAIDs and instillation of corticosteroids into the subacromial bursa, this patient has
been using NSAIDs without success. Higher doses are relatively contraindicated by
the patients age and diabetes. Moreover, an aggressive approach to treatment is
warranted in view of the patients apparent reduction in range of motion that might
reflect early signs of adhesive capsulitis (frozen shoulder) and for which
corticosteroids are also often beneficial and provide relief more quickly than
physiotherapy. The patient should, however, also be referred promptly for physical
therapy to regain function in the shoulder. An MRI would not be immediately

warranted unless following injection and relief of pain, a re-examination


demonstrated weakness of shoulder, suggesting a rotator cuff tear. Although, some
surgical reviews caution against the use of intrabursal corticosteroids in rotator cuff
tears, there is little firm evidence that a single injection impairs healing. Moreover, a
partial rotator cuff tear in this patient would be treated conservatively with physical
therapy. Plain radiographs might show calcification of the supraspinatus tendon
confirming the diagnosis of rotator cuff tendonitis but absence of this finding does
not rule the diagnosis out. Radiographs would be indicated if the history were more
consistent with osteoarthritis. Reflex sympathetic dystrophy is characterized by
osteopenia of the humeral head on plain radiographs and increased activity on
radionuclide bone scanning, but these tests are unwarranted in the present case. In
uncontrolled trials, this condition has improved with high-dose oral corticosteroids.
Primary Care Medicine:Question 52
The correct answer is A
Educational Objectives
Review preoperative evaluation for cataract surgery in patients with underlying
medical conditions.
Critique
Various screening tests are often ordered in pre-operative evaluation for cataract
surgery. No routine screening studies have been proven helpful in predicting
complications or increasing the safety of cataract procedures, as shown in a large
prospective randomized trial that included complicated elderly medical patients.
Common medical complications of cataract surgery are bradyarrhythmias and
hypertension. HIV testing is not indicated in the pre-operative setting except in
patients with identified risk factors.
Primary Care Medicine:Question 53
The correct answer is E
Educational Objectives
Treatments mild atopic dermatitis.
Critique
This patient has a typical presentation of atopic dermatitis (AD), one of the most
common skin conditions presenting to primary care physicians. The mainstays of
initial therapy are identification and avoidance of potential allergens and use of
topical corticosteroids. Corticosteroids are underutilized in the treatment of AD, in
part because of unfounded fears of corticosteroid atrophy.
Lindane therapy would be indicated if the patient had scabies, but the sites of
involvement and the appearance of the lesions are not characteristic of scabies. The
role of Staphylococcus aureus in AD has gained increased attention.S. aureus can
cause superinfection in severe or uncontrolled cases of AD, but may also play a role
in pathogenesis through production of superantigens. This observation has led to use
of antistaphylococcal therapy to eradicate colonization and nasal carriage; evidence
to date of the effectiveness of this approach are conflicting. Food allergies may play
a role in childhood AD, but are usually not a causative factor in adults. Because of
the potential short- and long-term toxicities, systemic corticosteroids should be
reserved for severe cases refractory to other therapies. Advances in understanding
the immunopathogenesis of AD have led to use of immunomodulators, such as
tacrolimus. Although topical tacrolimus has been shown to be effective for severe AD
in clinical trials, its use as initial therapy is limited by local skin burning and systemic
symptoms.

Primary Care Medicine:Question 54


The correct answer is D
Educational Objectives
Recognize typical features of the manic phase of bipolar disorder.
Critique
This patient presents with demanding, egocentric behavior. She has a great deal of
paranoia, and an elevated self esteem. All these features are consistent with the
manic phase of bipolar disorder, as is her lack of need for sleep. Other common
clinical issues that may be a clue to the manic phase include excessive credit card
debt, multiple sexual partners, calling acquaintances in the middle of the night, and
excessive long-distance phone bills. Although this patient has paranoid behavior, she
does not have other features that fit with schizophrenia or paranoid personality
disorder. Paranoid schizophrenia does not usually include disorganized speech or
behavior. Paranoid personality disorder does not include delusions. Patients with
frontal lobe brain tumor can have personality changes but would be unlikely to have
excessive energy and less need for sleep.
Primary Care Medicine:Question 55
The correct answer is B
Educational Objectives
Prescribe malaria prophylaxis for a patient traveling to areas of mefloquine-resistant
malaria.
Critique
MalaroneTM (atovaquone 250 mg/proguanil hydrochloride 100 mg) is recommended
for travel to the Thailand-Cambodia border and the Thailand-Myanmar border
because of the prevalence of mefloquine-resistant malaria in these regions. Malarone
therapy should be started 1 to 2 days before travel, taken 1 tablet daily, and
continued for 7 days after returning.
Doxycycline would provide protection but frequently causes vaginal yeast infections,
photosensitivity, and nausea. Chloroquine is recommended for areas where
chloroquine-sensitive malaria still occurs, that is, Mexico, Haiti, Dominican Republic,
Central America west of the Canal Zone, and parts of the Middle East. Primaquine is
sometimes recommended when other drugs are contraindicated and G6PD deficiency
has been excluded.
Primary Care Medicine:Question 56
The correct answer is C
Educational Objectives
Recognize and treat external otitis.
Critique
External otitis causes tenderness on palpation of the tragus or traction of the pinna,
and accumulation of wet exudate on an erythematous base in the external ear canal.
Surfers are at greater risk of this infection, as are those who wear hearing aids.
External otitis is a contraindication to removal of cerumen by any method, whether
curette, irrigation, or ceruminolytic. Treatment is with eardrops containing
hydrocortisone, neomycin, and polymixin. The suspension formulation is considered
safe, even if there is a tympanic membrane perforation so it can be used if the
tympanic membrane cannot be visualized. Oral antibiotics are not indicated for
external otitis. Culture of the exudate is not necessary before treatment in this
setting.

Primary Care Medicine:Question 57


The correct answer is A
Educational Objectives
Recognize and treat post-nasal drip as a factor causing chronic cough.
Critique
This patients recent upper respiratory infection, nasal exudate, and cobblestoning in
the posterior pharynx all suggest post-nasal drip as the cause of her cough. A
decongestant is likely to improve her symptoms. Decongestants can be safely used
in patients with well-controlled hypertension. A chest radiograph is not required at
this time as it is likely that treatment for post-nasal drip will resolve her cough;
however, a chest radiograph is warranted if the cough persists or worsens after an
appropriate therapeutic trial. Though ACE-inhibitor therapy can cause cough, she has
tolerated this medication well for a number of years and is unlikely to develop cough
from her medication at this point. Codeine-containing cough syrup will suppress the
cough and so is useful for patients losing sleep due to coughing, though this is not
the case for this patient. In addition, codeine may cause such side effects as
constipation and mental slowing without treating the underlying cause. Antibiotic
therapy is not warranted for post-nasal drip. In a significant number of cases,
chronic cough is due to multiple concurrent precipitants. If her symptoms persist
after several weeks of decongestant, reactive airways or esophageal reflux should be
considered. Reactive airways can also occur in the setting of a recent upper
respiratory setting, even in the absence of wheezing.
Primary Care Medicine:Question 58
The correct answer is E
Educational Objectives
Recognize hypokalemia as a reason to evaluate a patient for primary
hyperaldosteronism.
Critique
This patient may have primary hyperaldosteronism: his initial serum potassium
measurement is low at 3.2 meq/dL and although the average age of onset of
hypertension is in the early 30s, he is still relatively young to have hypertension.
Although hyperaldosteronism was previously thought to account for only 1% to 2%
of cases of hypertension, recent studies (for example, Lim et al.) suggest that the
incidence may be significantly higher in primary care populations. Additional history
to obtain from this man would include potassium intake, use of herbal supplements
or black licorice (aldosterone-like agonist), and recent diarrhea, inhaler use for
asthma, exercise without potassium replenishment, and other causes of hypokalemia
should be considered. In women, and some men, one must also consider diuretic
abuse for weight loss.
Primary Care Medicine:Question 59
The correct answer is A
Educational Objectives
Recognize chronic daily headache and understand the role of analgesic overuse in
the persistence of
this condition.
Critique
This patient has chronic daily headache, and the large amount of analgesics from
several different classes play an important role in the persistence of her headaches.
The best approach is to stop her current therapy and see if this leads to
improvement or resolution of her headaches. If the headaches are severe, she may
need to be hospitalized to undergo medication withdrawal. The length of withdrawal

headaches is longest with previous analgesic use (9.5 days) compared to pure
triptan withdrawal (4 days). The severity of withdrawal headaches appears to be
worse with narcotics and barbiturates than with triptans. Once the withdrawal is
complete, if headaches persist, prophylactic therapy can be started. Starting
prophylactic therapy before removing the current analgesics decreases the
effectiveness of prophylaxis. Amitriptyline has been the best studied for chronic daily
headache with the most consistent success. If she develops true migraine headaches
after dry withdrawal, triptans could be reinstated, but in a limited amount (no more
than twice a week).
Primary Care Medicine:Question 60
The correct answer is B
Educational Objectives
Recognize and treat serious otitis externa.
Critique
This patient has a moderately severe case of otitis externa. Because the ear canal is
still open and the tympanic membrane is visible, placement of a wick is not
necessary. He does not appear to have malignant otitis in which the surrounding
cartilage and bone are involved, although he is certainly at risk because of the
diabetes mellitus. Most such cases can be managed with oral antibiotics.
Staphylococcus and Streptococcus sp. must be covered, and, especially in the
diabetic patient, Pseudomonas is important. Topical therapy would be inadequate
because of the degree of inflammation, the low-grade fever, and the patients risk for
progression.
Primary Care Medicine:Question 61
The correct answer is A
Educational Objectives
Recognize causes of recurrent aphthous ulcers.
Critique
This is a classic history for recurrent aphthous ulcers. The diagnosis of aphthous
ulcers is clinical, and does not need to be confirmed by any testing. However,
aphthous ulcers can be associated with several systemic illnesses. This patient has
several worrisome signs that her ulcers could be related to a systemic illness: the
ulcers are becoming more frequent, she has lost 4.4 kg (10 Ib), and she has
abdominal pain and diarrhea recently diagnosed as irritable bowel disease. The
patient may therefore actually have inflammatory bowel disease, which is a cause of
recurrent oral ulcers. The patient should have a colonoscopy with biopsies. While HIV
and the spondyloarthropathies can both be associated with aphthous ulcers, this
patient does not have risk factors for HIV or any symptoms of arthritis.
Oral ulcers such as these are not related to herpes simplex virus infection, and
acyclovir has not been shown to be of benefit. Because this patients current lesions
are only about a week old, they do not require biopsy. If the patients illness had
features of pemphigoid or pemphigus vulgaris such as blisters preceding the ulcers,
biopsy would be of benefit to rule out those rare diseases. If the ulcers persisted for
longer than a month in any one place, then a biopsy would be warranted to rule out
malignant causes.

Primary Care Medicine:Question 62


The correct answer is E
Educational Objectives
Review immunization status in an adult patient.
Critique
Although some groups recommend a single tetanus booster at age 50 years after a
patient completes the full five-dose pediatric tetanus-diphtheria series, most
recommendations include immunization every 10 years after completion of the
pediatric series. Day care center staff and those who work in long-term care facilities
should receive hepatitis A and B vaccination. Varicella infection in the adult is
associated with significant morbidity. It is cost effective to determine serologic status
on nonelderly patients with an unreliable history of varicella because 70% of them
are found to be immune.
Primary Care Medicine:Question 63
The correct answer is E
Educational Objectives
Manage an older woman with stress urinary incontinence.
Critique
This patients urinary frequency and stress incontinence have not fully responded to
initial nonpharmacologic interventions (Kegel exercises, timed voids). Pharmacologic
treatments should now be considered, including estrogens and -adrenergic
agonists. Estrogen deficiency is associated with a wide range of urogenital
complaints including frequency, incontinence, and urinary tract infections. Treatment
with topical estrogen or an estrogen-impregnated vaginal ring has been shown to
improve the thickness and quality of pen-urethral tissue and effectively treat
postmenopausal atrophic changes. Some investigations have indicated that
estrogens can also provide prophylaxis against recurrent urinary tract infections.
Data regarding the efficacy of estrogen on reducing incontinence are mixed, but as
this patient has moderate to severe atrophic changes on examination and both stress
incontinence and recurrent infections, topical estrogen is the best initial choice to
mitigate her symptoms.
-Adrenergic agonists (for example, phenylpropanolamine) increase internal
sphincter tone and bladder outflow resistance and are a reasonable treatment choice
for stress incontinence (and may be most effective when given in combination with
estrogens, which up-regulatect-adrenergic receptors). However, this patient has a
history of superventricular tachycardia, and these agents should be used cautiously,
if at all, in patients with a history of cardiac arrhythmia. Antibiotics are not indicated
in this patient without active symptoms of infection and gram-positive flora on
culture. Imipramine, a tricyclic antidepressant with -agonist and anticholinergic
activity, is not indicated in this patient with stress incontinence (it may have a
limited role in treating younger patients with urge and mixed urge/stress
incontinence, but side-effects generally prohibit its use in older adults). Oxybutinin is
indicated for urge incontinence, a condition not present in this patient.
Primary Care Medicine:Question 64
The correct answer is E
Educational Objectives
Recognize sleep apnea as a cause of nocturnal or early morning headaches.
Critique
The list of conditions causing recurring early morning or nocturnal headaches is fairly
short: migraines, cluster headaches, chronic paroxysmal hemicrania, obstructive
sleep apnea, and increased intrecranial pressure, especially from a mass lesion.

Hypnic headache describes headache coming on during sleep; some cases may
represent a sleep-related headache disorder, whereas other cases probably
represent one of the above diagnoses. Other conditions such as bruxism,
temporomandibular joint dysfunction, and various musculoskeletal conditions of the
cervical spine may cause headaches that are present on awakening but also occur at
other times. Nausea, vomiting, or neurologic symptoms suggest migraine or a
process causing increased intercranial pressure. Papilledema is the most important
sign to look for on physical examination. If papilledema is present, the brain should
be imaged for a possible mass lesion. Several other findings suggest that imaging of
the brain may be needed: vomiting, neurologic abnormalities, history of malignancy,
or progression in severity or frequency of headaches. Additional history from this
patients bed partner or cohabitants regarding loud snoring or gasping/choking during
sleep would be the next step to obtain information regarding possible sleep apnea. In
men shirt size greater than 17.5 inches is suggestive of sleep apnea. Herbal
medications, alcohol, and depression are not specifically associated with early
morning headaches. Past head trauma can contribute to insomnia, and a chronic
subdural hematoma might cause nocturnal headaches, but this is less likely given his
history suggestive of a sleep disorder.
Primary Care Medicine:Question 65
The correct answer is B
Educational Objectives
Recognize that laboratory testing is unnecessary in healthy patients without a history
suspicious for bleeding disorders before low-risk surgeries.
Critique
Hernia repair is generally considered to be low risk for bleeding complications:
nonvital organs are involved, the surgical site is exposed, there is limited surgical
dissection, and local hemostasis is likely to be effective. Furthermore, the patient
describes no personal or family history of bleeding or clotting disorders. No further
laboratory testing is required prior to low risk procedures (dental extraction, lymph
node biopsy, or herniorrhaphy) in patients without a history or physical examination
suggestive of bleeding disorders. Extensive preoperative testing in this setting is
expensive, rarely identifies an abnormality, and potentially delays indicated medical
care. Among patients with a history and physical examination that do not suggest a
bleeding disorder, laboratory abnormalities do not predict adverse surgical outcomes.
In one prospective study, prothrombin time, activated partial thromboplastin time,
and platelet counts were obtained in more than 3000 consecutive patients
undergoing general surgery, only 1 of 340 patients (0.3%) with a normal history but
abnormal tests required treatment of the abnormality. By contrast, 26 of 172 (15%)
of patients with an abnormal history and laboratory abnormality required treatment
of the abnormality.
In patients with a personal or family history of bleeding disorders (for example,
prolonged bleeding following dental extractions, hemarthrosis, easy bruising, or
petechiae) or prior to moderate- or high-risk procedures (for example, laparotomy,
thoracotomy, mastectomy, liver or kidney biopsy, neurosurgery, ophthalmic
surgery), it would be appropriate to obtain prothrombin time, activated partial
thromboplastin time, and a platelet count as an initial screen for bleeding disorders.
These screening tests identify patients with coagulation or platelet abnormalities and
would prompt additional testing as indicated to identify an underlying disorder.
Bleeding time is a reasonable measure of the interaction between platelets and
injured endothelium, and has been used historically as a surrogate of bleeding risk,
but does not accurately predict bleeding risk.

Primary Care Medicine:Question 66


The correct answer is E
Educational Objectives
Recognize endophthalmitis.
Critique
This patient has endophthalmitis, infection of the vitreous humor within the globe of
the eye. The hallmark of endophthalmitis is a hypopyon, or collection of pus in the
anterior chamber. When endophthalmitis is suspected, emergent aspiration and
culture of fluid in the anterior chamber and vitreous humor are required.
Endogenous endophthalmitis is the result of metastatic infection to the globe, and
requires a search for primary infection source such as meningitis, urinary tract
infection, endocarditis, skin, and puerperal, pulmonary, and gastrointestinal sites.
Infectious organisms include Staphylococcus aureus, streptococci, Neisseria
meningitidis, and other gram-negative rods. Bacillus cereus is a likely cause of
endophthalmitis in injection drug users. Diabetes is a risk factor for acute bacterial
endophthalmitis. This patients corticosteroid therapy puts him at risk.
Exogenous endophthalmitis is associated with trauma and eye surgery, occurring in
0.08% to 0.5% of patients after cataract surgery. The most common organisms in
postoperative endophthalmitis are Staphylococcus aureus, streptococci, and gramnegative rods; trauma-related endophthalmitis is most likely due to Staphylococcus
aureus, coagulase-negative staphylococci, streptococci, gram-negative rods, or
fungi.
Treatment of endogenous endophthalmitis includes intravenous antibiotics. Patients
with postoperative endophthalmitis are given intravitreal antibiotics. The use of
corticosteroids is controversial. Keratitis and iritis are inflammatory processes that
would present with red eye and vision change but not a fluid level. Acute angle
glaucoma presents as loss of vision and tonometry is diagnostic. Keratoconjuctivitis
sicca presents with dry irritative symptoms but does not affect vision. Despite
prompt therapy, outcome in endophthalmitis is poor, and most patients lose their
vision.
Primary Care Medicine:Question 67
The correct answer is C
Educational Objectives
Evaluate and manage unexplained weight loss.
Critique
This patient has unexplained weight loss of greater than 5%. The most common
causes of such weight
loss include psychiatric disorders, gastrointestinal disorders, and malignancies.
Complaints of early satiety and nausea, particularly when the patient has a history of
gastroesophageal reflux, should prompt evaluation of the upper gastrointestinal tract
in order to rule out esophageal or gastric cancer and gastrointestinal lymphoma, or
less likely, peptic ulcer disease. Major depression is a very common cause of
unexplained weight loss. This patient has several symptoms suggesting depression
but does not meet the criteria for major depression. Moreover, the anorexia in
depression usually does not involve nausea and early satiety. Also he reports an
active life and no anhedonia. Although colon cancer is more common than cancer in
the upper gastrointestinal tract, the patients symptoms suggest an upper
gastrointestinal disorder. Caloric supplements with short-term follow-up are
recommended for unexplained weight loss in patients with no localizing symptoms.
Metoclopramide is a reasonable treatment for gastroparesis but because of this
patients risk for cancer, evaluation is appropriate.

Primary Care Medicine:Question 68


The correct answer is D
Educational Objectives
Recognize the likely cause and manage a patient with dizziness.
Critique
This patient likely has labyrinthitis, also called vestibular neuronitis, usually a selflimited dysfunction of the inner ear causing severe symptoms of vertigo. Meclizine is
the most helpful in decreasing symptoms for this patient, and for those with vertigo
due to Menires disease.
The Epley otolith repositioning maneuver is used in benign positional vertigo; it
alleviates symptoms in the majority of patients, though recurrences are common.
Patients can be taught to perform the maneuver at home. Extinguishing exercises
are also helpful in reducing symptoms in patients with benign positional vertigo. MRI
is not required because her symptoms are neither persistent, nor progressive, as one
would expect with an acoustic neuroma.
Electronystagmography is helpful if history and examination fail to distinguish
peripheral from central vertigo. Vertigo without other brainstem deficits is highly
unlikely to be of central origin. Patients with unilateral sensorineural hearing deficit
confirmed by audiology should have brain MRI to rule out acoustic neuroma. Episodic
vertigo with associated brainstem deficits warrants MRI and angiography.
Primary Care Medicine:Question 69
The correct answer is B
Educational Objectives
Evaluate a recently exposed person for tuberculosis infection.
Critique
The most likely reason for a false-negative PPD is that the nurse has not converted
yet. Most exposed, infected persons convert in 4 to 7 weeks. Therefore, repeat
testing in 3 months is appropriate. Mantoux tests using strengths other than 5 TU
(0.1 mL) are no longer recommended. Other causes of false-negative results that
might apply here would include immunosuppression or anergy. Persons taking
corticosteroids, who have HIV infection, or who are on hemodialysis, as well as
elderly patients fall into this category. A 5-TU booster test 1 week later is
appropriate if enough time has elapsed for conversion and immunocompromise may
be an issue. A chest radiograph is not indicated in an otherwise healthy person with
a negative PPD because the sensitivity of a properly placed test is 95%.
Primary Care Medicine:Question 70
The correct answer is D
Educational Objectives
Understand the contraindications for the use of triptans and ergotamines in the
treatment of migraine
headache.
Critique
This patient should not receive triptans or ergotamines because of his history of
coronary artery disease, which is an absolute contraindication to triptan and
ergotamine use. The actual risk is very low for cardiac events with triptans, and
almost all serious events have occurred in patients with underlying coronary artery
disease. An alternative migraine treatment such as an NSAID with a promotility
agent (metoclopramide) would be an appropriate first line therapy in this patient.

Primary Care Medicine:Question 71


The correct answer is B
Educational Objectives
Recognize that, in acute alcohol withdrawal, symptom-triggered dosing of
benzodiazepines is more
effective.
Critique
This patient should be hospitalized because he is hallucinating and is hypertensive.
The risk of seizures in this setting is 3%, usually with 48 hours of withdrawal.
Benzodiazepines are the mainstay of therapy in the patient with acute alcohol
withdrawal. In general, the long-acting benzodiazepines such as diazepam are
preferred. Three dosing regimens have been used with these medications. A frontloaded approach uses a fairly large dose of a long-acting medication and lets it
slowly be metabolized. A fixed regimen uses a gradual, set taper and the symptomtriggered uses the Clinical Institute Withdrawal Assessment scale to determine level
of withdrawal and doses accordingly. This final option leads to less total
benzodiazepine use and faster discharge.
Primary Care Medicine:Question 72
The correct answer is D
Educational Objectives
Recognize and start nonpharmacologic treatment of xerostomia.
Critique
This patient has xerostomia. She has dry eyes; the symptoms of halitosis, angular
cheilitis, a dry mouth, and trouble chewing and swallowing her food all relate to
decreased oral secretions. Xerostomia increases in frequency with age, and can be
caused by many medications, previous irradiation, or systemic illnesses such as
Sjogrens syndrome. Donepazil does not cause xerostomia, however, and should not
be discontinued. For this patient, whose age at onset and comorbidities are the likely
causes of her xerostomia, the first step in treatment is nonpharmacologic. She
should drink an adequate amount of fluid during the day, especially at mealtime. In
addition, secretagogues, such as lemon candies, can help maintain a moist mouth
environment that will help alleviate the oral dryness and angular cheilitis.
Although this symptom complex could be caused by Sjogrens syndrome, this patient
has several more common reasons to have xerostomia. Sjogrens syndrome typically
presents in women in their 50s; if the clinical suspicion for Sjogrens syndrome was
higher, with no alternate possibilities, then the antibody titers would be appropriate
diagnostic tests.
A speech and swallowing evaluation is not necessary at this point, because she is not
choking, although it might be indicated if swallowing does not improve with
enhanced salivation. Although sinusitis can cause halitosis, it is not a cause of the
rest of her symptom complex, and so treating for sinusitis just based on this clinical
scenario is not necessary.
Primary Care Medicine:Question 73
The correct answer is E
Educational Objectives
Understand the approach to cervical cancer screening in patients who have had
hysterectomy for benign disease.
Critique
In asymptomatic women who have had a vaginal hysterectomy for benign disease,
there is no proven benefit to routine pelvic examinations or Pap smear to detect
cancer. Vaginal hysterectomy for benign disease is not associated with an increased

incidence of vaginal malignancy. Estimates of the positive predictive value of an


abnormal vaginal smear in this setting approach zero. In addition, there is no
evidence that routine pelvic exams are effective measures to screen for ovarian
cancer. Masses discovered by pelvic examinations are commonly benign adnexal
masses or advanced ovarian tumors. Pelvic examinations miss small, early-stage
ovarian tumors. Pap smear is not used to detect ovarian cancer due to unacceptable
low sensitivity rates of approximately 10% to 30%.
Primary Care Medicine:Question 74
The correct answer is B
Educational Objectives
Diagnose mild cognitive impairment.
Critique
This patients symptoms are most consistent with isolated memory problems and do
not meet criteria for a diagnosis of dementia because his deficits are limited to one
area of cognitive function (memory) and there is no history that his impairments are
sufficient to interfere with his usual activities. Although his current diagnosis is most
appropriately referred to as mild cognitive impairment (MCI) this may represent the
earliest stages of a dementing illness, with studies suggesting that patients with MCI
have a high risk of progressing to dementia, with an annual conversion rate of 10%
to 15%.
Multi-infarct dementia and Alzheimers dementia are not appropriate diagnoses
because the patient does not meet criteria for having dementia. Furthermore, mildmoderate white matter disease is frequently noted on head CT scans, and this
finding alone cannot establish a diagnosis of vascular dementia. Hypothyroidism is
not implicated because, although the patients TSH level is at the upper limit of
normal, it is still within the normal range. Finally, although drugs are a key factor
that need to be considered as potential causes of cognitive impairment, the history
does not indicate recent medication changes and none of the three medications he is
taking have been implicated as directly affecting cognition.
Primary Care Medicine:Question 75
The correct answer is C
Educational Objectives
Recognize obstructive sleep cause of difficult-to-treat hypertension.
Critique
This patients history of snoring and daytime sleepiness, in conjunction with his
crowded oropharynx and large neck, suggests that he may have obstructive sleep
apnea. In large epidemiologic studies, this disorder has been clearly associated with
hypertension, independent of obesity and with a dose-dependent relationship
between severity of sleep-disordered breathing and likelihood of hypertension. The
Wisconsin Sleep Cohort Study found a higher rate of hypertension developing in
normotensive persons who had sleep-disordered breathing than in those without,
independent of weight. Sleep studies on patients with difficult-to-treat hypertension
evaluated in a specialty clinic found previously undiagnosed obstructive sleep apnea
in 83% of the patients (96% in men, 65% in women). Although treatment of
obstructive sleep apnea with nocturnal CPAP does not reduce blood pressure
significantly in established hypertension, CPAP can reduce daytime sleepiness and
improve quality of life.
The other causes of secondary hypertension listed are less supported by the findings
in this case. Although normal results on one round of urinary catecholamine testing
do not rule out pheochromocytoma, the pattern tends be a paroxysmal elevation of
blood pressure rather than persistent elevation. The absence of physical findings of

hypercortisolism disease (moon facies, buffalo hump, and purple abdominal striae)
and normal plasma glucose testing all suggest Cushings syndrome is not likely.
Normal renal function, absence of a renal artery bruit, and limited benefit from an
ACE inhibitor are all evidence against significant renal artery stenosis. Chronic lead
intoxication causes hypertension in the setting of chronic renal insufficiency, not
present in this case.
Primary Care Medicine:Question 76
The correct answer is C
Educational Objectives
Manage long-term anticoagulation in the perioperative setting.
Critique
Tilting disc or bileaflet mechanical prosthetic valves are associated with an estimated
8% annual risk of thromboembolic complications in the absence of anticoagulation.
Current guidelines recommend long-term anticoagulation to a target INR of 2.5 for
modern mechanical prosthetic valves at the aortic position in patients in sinus
rhythm with normal left atrial size and left ventricular function. Anticoagulation
reduces the annual thromboembolic risk to approximately 2%.
General surgeries may be safely performed if the INR is 1 .5 or less. Withholding
warfarin for four doses before scheduled surgery and then resuming warfarin therapy
on the first postoperative night may safely achieve this INR. This strategy results in a
subtherapeutic INR for approximately 2 days before and 2 days after surgery,
exposing patients to minimal thromboembolic risk. The patient in this example is
likely to be at an INR of 1 .5 the following day, without any specific intervention.
Vitamin K administration would be expected to normalize the patients INR, but
would unnecessarily prolong the time required to reinstitute long-term
anticoagulation with warfarin. Fresh frozen plasma would similarly normalize the
INR, but is expensive and not likely to significantly reduce bleeding complications
related to the planned surgery. The use of unfractionated or low-molecular-weight
heparins prior to surgery would be safe but unnecessary in a patient without specific
indication for acute anticoagulation (acute venous or arterial thromboembolism
within the past month, atrial fibrillation with prior embolic complication, or prior
recurrent venous thromboembolism).
Primary Care Medicine:Question 77
The correct answer is D
Educational Objectives
Diagnose and treat common causes of hip pain.
Critique
Pain with walking but not at rest suggests osteoarthritis of the hip, but the
snapping sensation is characteristic of iliotibial band syndrome, which is often
precipitated by initiation of exercise or an increase in intensity. This diagnosis is
further suggested by pain with adduction, which is performed by extending the
affected leg over the edge of the examination table while the patients lies on her
side. Meralgia paresthetica is caused by pressure on the lateral femoral cutaneous
nerve as it perforates the fascia of the lateral thigh and is most common in
overweight persons. It may cause pain but dysesthesia is typically more prominent
and these symptoms are unrelated to activity. The pain of trochanteric bursitis may
occur with walking but is usually most severe when lying on the affected hip. On
examination, tenderness is localized over the greater trochanter. lschial bursitis
causes pain over the ischial tuberosity when seated.
Initial therapy for iliotibial band syndrome is heat, stretching, and NSAIDs. If
symptoms persist, evaluation by a physical therapist is indicated. Although

osteoarthritis remains a possibility, radiographs can be deferred for a few weeks until
the results from conservative therapy have been evaluated. If radiographs
demonstrate degenerative changes of the hip and the pain becomes severe enough
to interfere with regular activities, the patient may wish to consider hip replacement,
at which point referral to an orthopedic surgeon would be in order. Injection of the
trochanteric bursa or a nerve conduction velocity study are not indicated
Primary Care Medicine:Question 78
The correct answer is E
Educational Objectives
Emphasize importance of screening for glaucoma and recognize major risk factors.
Critique
Primary open-angle glaucoma occurs in 2% of American adults, has an insidious
onset, and is initially asymptomatic. Therefore, screening for risk factors in primary
care is important in identifying those at high risk. Prevalence of glaucoma increases
steadily with age over 50 years. This patient has other risk factors, the most
important being black American ethnicity. The prevalence of glaucoma is 3% in black
Americans younger than 60 years, compared to 12% of those older than 70 years.
Hispanics also appear more likely to have glaucoma than white Americans. Family
history of glaucoma increases the risk somewhat if there is an affected parent, more
so if an affected sibling, but still less than black American ethnicity. Systemic
hypertension is not considered a significant risk factor. There is a weak association of
glaucoma with other medical conditions, such as sleep apnea.
Primary Care Medicine:Question 79
The correct answer is D
Educational Objectives
Recognize that major depression is a common cause of unexplained weight loss.
Critique
The patient has unexplained weight loss with symptoms typical for major depression:
fatigue, difficulty concentrating, sleep disturbance. Mirtazapine is a newer
antidepressant that is sedating. Weight gain is common with this medication.
In patients with unexplained weight loss, endoscopy of the upper and lower
gastrointestinal tracts and imaging beyond a chest radiograph are unlikely to disclose
a cause for the weight loss in the absence of more specific symptoms or laboratory
abnormalities. The patients serum thyroid-stimulating hormone concentration is
borderline low but not in a range to suggest a relapse of her Graves disease.
Primary Care Medicine:Question 80
The correct answer is C
Educational Objectives
Diagnose de Quervains tenosynovitis.
Critique
The patient has typical findings of De Quervains tenosynovitis, that is, pain localized
to the lateral wrist
over the distal radius and a positive Finkelsteins test. Corticosteroid injection into
the tendon sheath provides the most immediate relief. In the absence of trauma and
with an examination suggesting De Quervains tenosynovitis, plain radiographs are
not necessary. Osteoarthritis of the first carpometacarpal joint causes pain over the
lateral wrist but just distal to the wrist crease, nearer the base of the thumb, when
compared with De Quervains tenosynovitis. A nerve conduction study is used to
confirm carpel tunnel syndrome. Although the Tinels sign was equivocal, the clinical
scenario does not suggest carpel tunnel syndrome. Therapy with nonsteroidal anti-

inflammatory agents, either nonselective agents or COX-2 selective agents, may


improve De Quervains tenosynovitis, but local corticosteroid injection is more
effective therapy.
Primary Care Medicine:Question 81
The correct answer is A
Educational Objectives
Recognize the likely cause and manage patients with dizziness
Critique
This patient has positional episodes of vertigo, likely due to benign positional vertigo.
The Epley otolith repositioning maneuver alleviates symptoms in most patients,
although recurrences are common. Patients can be taught to perform the maneuver
at home. Extinguishing exercises are also helpful, though this was not given as an
option.
Meclizine therapy is not effective for patients with benign positional vertigo. There is
no indication that the patient has cardiac arrhythmias, and therefore
electrocardiography would not be indicated. MRI of the head would not be required
at this time because the symptoms are neither persistent nor progressive, as one
would expect with an acoustic neuroma.
Primary Care Medicine:Question 82
The correct answer is E
Educational Objectives
Recognize vasovagal syncope and understand that no further laboratory evaluation
testing is indicated.
Critique
This patient clearly has vasovagal syncope triggered by seeing blood on his arm. His
history suggests a previous vasovagal episode, and is otherwise benign. Importantly,
two or three tonic-clonic jerks in this situation do not indicate seizure activity, but
are the normal result of hypoxia in the brain. Vasovagal and situational syncope
account for 23% of cases of syncope. Laboratory tests in this situation yield a
diagnosis in only 2% to 3% of cases over all and would not be helpful in this case
when the diagnosis is clear. The do not miss diagnosis in this case is premature
heart disease, but he has no cardiac risk factors. Conversely, patients who have
exercise-induced syncope should have a careful examination for murmurs
(hypertrophic obstructive cardiomyopathy-systolic murmur that increases with
Valsalva) and a thorough cardiac evaluation.
Primary Care Medicine:Question 83
The correct answer is C
Educational Objectives
Understand the role of corticosteroids in the treatment of herpes zoster.
Critique
This patient has severe pain due to herpes zoster. His initial treatment and
evaluation were appropriate. He received a potent antiviral agent active against
varicella zoster (valacyclovir), and he had a slit lamp examination to look for subtle
corneal involvement. His pain was severe enough to disrupt sleep. Addition of
prednisone is the next appropriate step in treatment. Prednisone improves quality of
life scores in patients with severe pain from acute zoster. Methadone is a long-acting
pain medication and would not be the optimal short-term narcotic pain management
for this patient. Capsaicin cream would not be a good choice to use around the eye
because of the irritation it could cause to the conjunctiva if any contacted the eye.
Capsaicin is also not a good treatment option for acute pain due to herpes zoster; its

role is in the long-term management of post herpetic neuralgia.


Primary Care Medicine:Question 84
The correct answer is B
Educational Objectives
Recognize and treat at-risk drinking.
Critique
This patient meets the criteria for at-risk (heavy) alcohol use, which is defined as
greater than 14 drinks/week for men and greater than 7 drinks/wk for women
without evidence of abuse or dependence. She responds positively to one of four
CAGE questions (annoyance). The CAGE questions are best at identifying patients
with abuse or dependence. The appropriate treatment for those patients with at-risk
alcohol use is the brief intervention. In this intervention, drinking patterns are
discussed, goals are agreed upon, and support/follow-up is arranged. Medications
and inpatient programs are more appropriate for those with abuse or dependence
issues.
Primary Care Medicine:Question 85
The correct answer is D
Educational Objectives
Recognize acute sensorineural hearing loss.
Critique
A Weber test that lateralizes away from the ear in question makes the diagnosis of
sensorineural hearing loss in the affected ear. In this patient, who has symptoms in
only one ear, who has a normal examination, and who has a Weber that lateralizes
away from the affected ear, the only possible diagnosis is acute unilateral
sensorineural hearing loss. The Weber test in a patient with unilateral conductive
hearing loss lateralizes toward the side of the complaint, because the bone
conduction is greater than air conduction on that side. Cholesteatoma, serous otitis,
and otosclerosis are causes of conductive hearing loss.
Primary Care Medicine:Question 86
The correct answer is C
Educational Objectives
Distinguish features of Menires disease from other causes of tinnitus.
Critique
The most common cause of episodic tinnitus is age-related hearing loss. This young
patients tinnitus and hearing loss are classic findings of Menires disease. This
condition results from an idiopathic accumulation of fluid in the endolymphatic sacs
of the inner ear. Patients with Menires disease have episodes of tinnitus, hearing
loss, vertigo, and aural fullness. Diagnosis is based on these clinical symptoms, the
finding of sensorineural hearing loss, usually at low pitches, and the absence of
clinical findings to suggest other causes of tinnitus (for example, medications, otitis,
trauma, and less common central nervous system disorders such as acoustic
neuroma or multiple sclerosis). Treatment begins with dietary restrictions of salt and
caffeine, judicious use of diuretics, and symptomatic relief with vestibular
suppressants and antiemetics.
Labyrinthitis results from recent trauma or infection. Hallmark symptoms include
acute tinnitus and vertigo. Episodes are severe, but limited and do not lead to
recurrences or hearing loss. Otosclerosis involves bony overgrowth of the stapes
leading to conductive and not sensorineural hearing loss. Stress-induced tinnitus is
not usually associated with hearing loss.

Primary Care Medicine:Question 87


The correct answer is B
Educational Objectives
Recognize nonadherence and forced compliance as a cause of adverse drug
reactions in hospitalized
patients.
Critique
This patients hospital complications include recurrent hypoglycemia, new-onset
atrial fibrillation, and confusion. Although the latter could be caused by any and all of
the new medications she received, her constellation of symptoms is best explained
by enforced adherence to prior medications (glyburide, metformin, and
levothyroxine) that she likely had not been fully compliant with before admission.
Under- and noncompliance occur frequently in older adults taking multiple
medications and should be considered in any patient who is not responding well to
appropriate and/or increasing doses of medications. One sign that compliance was
problematic is that this patient was taking relatively high-dose levothyroxine and yet
her serum TSH was borderline high. When outpatient doses are suddenly enforced in
the hospital setting, adverse effects, such as hypoglycemia from diabetic medications
and atrial fibrillation from excessive thyroid replacement, may ensue.
Polypharmacy is an important consideration as this patient received three new
psychoactive medications after admission, placing her at very high risk for delirium,
which she did appear to develop. However these medications would not explain her
hypoglycemia or her atrial fibrillation as clearly as enforced adherence. Hydroxyzine,
a sedating antihistamine, is strongly associated with an increased risk of delirium,
and its anticholinergic activity could also increase the likelihood of rapid heart rate.
However, this medication should not induce hypoglycemia or atrial arrhythmia and
again does not explain her constellation of problems as well as forced compliance.
Lastly, withdrawal from agents that patients may be surreptitiously taking (for
example, alcohol or benzodiazepines) is an important and often overlooked
consideration of delirium in hospitalized patients. However, no evidence of
surreptitious alcohol use is provided in the history, physical examination, or
laboratory results, and the patients course and complications are more consistent
with drug-induced side effects from levothyroxine and hypoglycemic-inducing
medications than from alcohol withdrawal.
Primary Care Medicine:Question 88
The correct answer is D
Educational Objectives
Recognize sleep apnea as a cause of persistent fatigue.
Critique
Several historical features in this case suggest obstructive sleep apnea: loud habitual
snoring, dry mouth on awakening (suggesting mouth breathing), and feeling
unrefreshed on awakening. This patient has oropharyngeal crowding, which is
associated with obstructive sleep apnea. People with obstructive sleep apnea often
do not describe their daytime symptoms as sleepiness; instead, terms like
tiredness or lack of energy or fatigue may be used. Although obstructive sleep
apnea is more prevalent at higher BMI, it does occur in patients at normal weight.
She has three symptoms which could fit with chronic fatigue syndrome (unrefreshing
sleep, neuropsychological symptoms, and sore throat), but no findings of fever,
pharyngitis, or cervical or axillary adenopathy. Chronic hepatitis C has not been
convincingly linked to fatigue. Chronic Lyme disease can cause a large knee effusion
but less commonly pain. Chronic suicidality and complaints of emptiness are a major
feature of borderline personality disorder, a diagnosis to consider when depression

does not respond to antidepressants. Depression is commonly diagnosed in women


with obstructive sleep apnea before their sleep disorder is recognized.
Primary Care Medicine:Question 89
The correct answer is A
Educational Objectives
Manage perioperative corticosteroid administration for a moderate-risk surgery.
Critique
Adrenal insufficiency in the perioperative setting is associated with increased
morbidity and mortality. The most common cause of adrenal insufficiency in the
perioperative setting comes from the long-term exogenous administration of
corticosteroids. Long-term administration of corticosteroids suppresses pituitary
corticotropin synthesis. Chronic suppression of corticotropin results in atrophy of the
adrenal cortex and decreased responsiveness to corticotropin. Persons treated
chronically with daily prednisone, 5 mg or more (or an equivalent corticosteroid),
and persons treated with prednisone, 20 to 30 mg/d for 1 or more weeks in the
preceding 3 months, are at risk for adrenal insufficiency in the perioperative period.
Other causes of adrenal insufficiency include pre-existing autoimmune adrenal
insufficiency or perioperative hemorrhage or sepsis.
Current recommendations for corticosteroid replacement are based on the usual
physiologic response of the hypothalamic-pituitary-adrenal axis to surgical stress.
Major surgeries (cardiothoracic, oncologic, or major abdominal surgeries) are treated
initially with hydrocortisone, 100 mg intravenously every 8 hours for three doses,
then 50 mg for three doses, then 25 mg for three doses, followed, in uncomplicated
patients, by resumption of the usual outpatient dose. Moderate surgeries
(orthopedic, urologic, otolaryngologic) are treated initially with hydrocortisone, 50
mg intravenously every 8 hours for three doses, then 25 mg for three doses,
followed by resumption of the usual outpatient steroid dose. Patients scheduled for
minor procedures may simply take their usual dose on the day of surgery and may
optionally double their first postoperative dose.
Primary Care Medicine:Question 90
The correct answer is C
Educational Objectives
Recognize melanoma and how it metastasizes.
Critique
The lesion described is a nodular 2-cm lesion that by size and features is highly
suspicious for a cutaneous melanoma. Therefore, it has high potential for
metastases, which usually occur via regional lymphatic vessels. Melanoma can
metastasize to almost every major organ and tissue, with lung and liver most
common distant sites; the heart and pericardium are rare classic distant sites. The
presence or absence of melanoma in regional lymph nodes is the single most
important factor for predicting survival. Melanoma may occur in the anterior or
posterior eye. Most cases of ocular melanoma are from hematogenous spread, but
may be a primary site with ultraviolet light as a risk factor. A history of melanoma is
always relevant in evaluation of new lesions or tumor mass, as melanoma
metastases may occur greater than ten years after primary melanoma excision.
Metastatic skin lesions may or may not be pigmented like the primary lesion.

Primary Care Medicine:Question 91


The correct answer is D
Educational Objectives
Recognize the limits of serum PSA testing for prostate cancer screening.
Critique
Although no nodule is felt on digital rectal exam, the presence of a discrete localized
firm region should prompt suspicion for further evaluation by a urologist, including
biopsy. Biopsy of this area will likely be recommended. Research reports have
defined an abnormal digital rectal examination differently but most include even an
asymmetric enlargement of one side of the gland.
Continued PSA monitoring at any frequency is inappropriate. The false negative rate
of the PSA is noted by the fact that 25% of men undergoing surgery for prostate
cancer have a PSA in the normal range. Ultrasonography of the prostate is only 60%
sensitive among men referred for evaluation of abnormal PSA or digital rectal
examination. However even if the ultrasonography was normal, further evaluation of
the abnormal physical finding would be recommended.
Primary Care Medicine:Question 92
The correct answer is B
Educational Objectives
Refer patients with dyspepsia and alarm findings for upper endoscopy.
Critique
Patients younger than 45 years with dyspepsia and no alarm symptoms can be
tested for Helicobacter pylon infection and treated empirically for either peptic ulcer
disease or H. pylon without endoscopy. However, this patient has an alarm symptom
of anemia, requiring upper endoscopy early in his workup. Other alarm symptoms
warranting endoscopy include weight loss, anorexia, recurrent vomiting, dysphagia,
and gastrointestinal bleeding.
A promotility agent is useful when gastroparesis has been confirmed or when severe
reflux responds only partially to maximal proton pump inhibitor therapy. This
diagnosis is usually seen in patients with diabetes. Abdominal CT scan is not needed
at this stage of his workup but may be performed if symptoms persist and endoscopy
does not reveal the cause.
Primary Care Medicine:Question 93
The correct answer is D
Educational Objectives
Understand the role of carotid sinus massage in the diagnosis of neurocardiogenic
syncope.
Critique
This patient likely has neurocardiogenic carotid-sinus syncope precipitated by turning
her head. Although she was dizzy for a few seconds, this is not likely to be vertigo.
Carotid sinus massage for 5 to 20 seconds on each side of the neck (sequentially)
while checking the pulse (or a rhythm strip) and blood pressure is diagnostic if
asystole >3 seconds or a decrease in systolic blood pressure of 50 mm Hg occur.
This maneuver is safe in patients without carotid bruits. Conversely, patients with
known or suspected cardiac disease, should have a full cardiac evaluation beginning
with echocardiography. Cardiovascular disease is the most important diagnosis to
exclude, because syncope associated with cardiovascular disease predicts increased
mortality. Finally, if the patient were suspected of having an arrhythmia, it would be
important to use a continuous loop recorder, not a Heart Card, since she would not
have had time to activate the device.

Primary Care Medicine:Question 94


The correct answer is E
Educational Objectives
Treat subacute laryngitis in the outpatient setting.
Critique
This patient without risk factors for malignancy has had a 3-month history of
laryngitis. The most likely cause is gastroesophageal reflux disease, and an empirical
trial with a proton pump inhibitor would be warranted. Patients with tobacco
exposure history or who are voice professionals should have visualization of the
vocal cords by an otolaryngologist. There is no benefit in treatment of chronic
laryngitis with antibiotics. In one recent study of 30 patients with laryngopharyngeal
symptoms, posterior laryngitis was confirmed in 26/30, with a success rate of
medical antireflux treatment in 90%. Proton pump inhibitor therapy has been
effective in small clinical trials in the treatment of idiopathic laryngitis.
Primary Care Medicine:Question 95
The correct answer is E
Educational Objectives
Manage symptomatic peripheral artery disease.
Critique
This patient presents with classic claudication, most likely secondary to peripheral
artery disease (superficial femoral artery disease is most common cause of calf
symptoms). The two main goals of therapy are to manage risk factors in an effort to
limit the progression and to reduce symptoms. Antiplatelet therapy, such as aspirin
and clopidogrel, are primarily used to reduce vascular complications. The calcium
antagonists such as amlodipine have no proven benefit in increasing peripheral blood
flow. Cilostazol, a phosphodiesterase inhibitor, and pentoxifylline are the two
approved medications for claudication. Cilostazol has been more effective than
pentoxifylline and has been shown to improve walking distance, functional status,
and quality of life.
Primary Care Medicine:Question 96
The correct answer is C
Educational Objectives
Recognize need for imaging when audiogram shows asymmetric sensorineural
hearing loss.
Critique
Although the history given by this patients wife is classic for presbycusis, or
symmetric sensorineural hearing loss associated with aging, his audiogram describes
asymmetric sensorineural hearing loss (asymmetry is defined as a difference of
greater than 10 dB loss between the ears). Asymmetric sensorineural hearing loss
suggests the possibility of a tumor of the 8th cranial nerve or of the cerebellopontine
angle, and this patient should have an MRI directed at that area. A CT scan with
contrast does not visualize the posterior fossa well enough to exclude a tumor there.
A stapedectomy is a surgical procedure which is done to treat otosclerosis, an
inherited form of conductive hearing loss. If the patients MRI does not show an
anatomic lesion, the patient should then be referred for hearing aides.

Primary Care Med icine:Question 97


The correct answer is E
Educational Objectives
Recognize abridged somatization.
Critique
This patient has four unexplained symptoms. True somatization disorder requires the
presence of eight unexplained symptoms, including at least four pain symptoms, two
gastrointestinal symptoms, a sexual symptom, and a neurologic symptom.
Somatization disorder is rare, whereas patients with somatic symptoms are common
in a general medicine practice. This patient has abridged somatization, defined as the
presence of four or more unexplained symptoms. Abridged somatization is common,
occurring in close to a quarter of patients in general medicine practices. Many
patients with abridged somatization have had thorough medical work-ups of their
symptoms before the diagnosis of abridged somatization is considered.
Primary Care Medicine:Question 98
The correct answer is A
Educational Objectives
Recognize altered renal function as a contributor to inappropriate medication dosing
in older adults.
Critique
This patient has decreased appetite, intermittent nausea, and weight loss, all of
which could be attributable to digoxin side effects. Of the medications she is taking,
digoxin is the most likely to be at an excessive dose because of her reduced renal
function. Although her serum creatinine level is within normal limits, her calculated
creatinine clearance (which takes her advanced age and relatively low weight into
account) is 26 mL/min. This makes it fairly likely that her daily digoxin dose of 0.25
mg may be excessive and result in a higher than anticipated digoxin level. In a
recent study of adverse drug reactions in the elderly, digoxin was among the most
commonly implicated medications, with frequent adverse effects being
gastrointestinal and/or cardiac arrhythmias. Furthermore, a recent study suggested
that women may be more likely than men to suffer adverse mortality effects from
digoxin, perhaps due to dosing considerations. Clinicians must recognize that
reduced renal function and diminished renal clearance of medications is often not
accurately reflected by serum creatinine levels in older adults, and dosing of renally
cleared medications should be based on calculated creatinine clearance rather than
serum creatinine levels. Decreasing her glipizide dose would not be a good choice as
this patients diabetes is apparently well controlled (hemoglobin A1C 7.4%), and the
history does not indicate hypoglycemia or other reasons to suspect her glipizide dose
is excessive. Lower doses of mirtazapine do improve appetite and sleep more than
higher doses, but 15 mg is already at the low end of this agents therapeutic dose
range and she appears to be doing well in terms of mood and sleep. Therefore,
further reduction to a subtherapuetic dose of mirtazapine is not warranted. Although
older adults are more sensitive to warfarin and this patients INR is at the upper end
of the desired range for atrial fibrillation, a dose reduction in warfarin would not be
expected to affect her appetite or weight. Lastly, lisinopril and other ACE inhibitors
are associated with altered taste perception (dysgeusia), but this patients blood
pressure is well controlled on her current dose, her electrolytes are within normal
limits, and this agent may be renoprotective as well as cardioprotective in this highrisk patient. Furthermore, dysgeusia is not a dose-related phenomenon so
decreasing her lisinopril dose is not likely to improve this symptom and is not as
prudent a first step as a trial of reducing her digoxin dose.

Primary Care MediCine:Question 99


The correct answer is B
Educational Objectives
Recognize inadequate sleep (sleep deprivation and undiagnosed sleep disorders) as a
major cause of
accidents and repeat near-misses.
Critique
Taking a sleep history is the best next step, as sleep deprivation and undiagnosed,
untreated sleep disorders (particularly obstructive sleep apnea) are common,
underrecognized causes of impaired driving, and because this patients history does
not clearly suggest another cause. The risk of motor vehicle accidents is significantly
increased (at least 5-fold) in each of these situations: (1) sleepy driver, (2) driver
with untreated obstructive sleep apnea and prior near-misses, and (3) during
predawn hours. The trucking industry and sleep specialists are most aware of this
problem; awareness among the public and among most physicians is lower.
Treatment of obstructive sleep apnea with nocturnal CPAP reduces the risk of motor
vehicle accidents and near-misses to that of the general population. After obtaining a
sleep history, vision testing and drug testing would both be reasonable to perform.
This patients history of momentary reduced awareness and driving performance,
without prodrome or postictal state, make seizure disorder or hemodynamically
significant arrhythmia unlikely as an explanation for his recurrent driving
impairment.
Primary Care Medicine:Question 100
The correct answer is C
Educational Objectives
Manage diabetes mellitus in a patient scheduled to undergo major surgery.
Critique
Management of hospitalized patients with diabetes mellitus is challenging. Not only
are there pathophysiologically distinct types of diabetes mellitus, there are numerous
distinct types of insulin therapy and pharmacologically distinct classes of oral
medications. Disease processes often impact diabetes care. Patients may eat less
and thus require less medication. Infection or inflammation may increase the risk for
diabetic ketoacidosis in insulin-deficient patients or worsen glycemic control in
previously well-controlled patients.
In the patient described, the simplest approach to managing his diabetes would be to
institute an insulin drip with frequent monitoring of blood glucose to maintain blood
glucose within a desirable range. A conservative target for glycemic control in
hospitalized patients is in the range of 120 to 180 mg/dL. This range reduces the risk
of symptomatic hypoglycemia and hyperglycemia. A recent study suggests that
patients hospitalized in intensive care (primarily surgical patients) have reduced
mortality and morbidity with very tight glycemic control targeted to 80 to 110
mg/dL. The decision to begin an insulin drip is simplified in this example because the
patient is hospitalized for an esophagectomy; he will be unable to eat or drink by
mouth and will be supported with parenteral nutrition in the postoperative period.
Combinations of parenteral nutrition and continuous insulin drips make glycemic
control relatively straightforward. Furthermore, his glycemic control prior to
hospitalization is poor. His hemoglobin A1c is in excess of 9%, more than 2% higher
than recommended levels. Maximizing his oral medications would be unlikely to
result in a further 2% decline in his hemoglobin A1c. In any event, metformin may
increase the risk for lactic acidosis in an acutely ill hospitalized patient and should be
stopped in most instances.
So-called physiologic replacement of basal and prandial insulin with a long-acting

insulin preparation (glargine insulin) and short-acting insulin (lispro or aspartate


insulin) is a reasonable alternative for a hospitalized patient, not scheduled for
surgery, who continues to eat. Glargine insulin has a rapid onset and predictable
action in most patients. It may be administered once daily before bedtime and
adjusted according to fasting morning blood glucose levels. Combinations of
intermediate-acting insulins (NPH) and short-acting insulins (lispro or aspartate
insulin) are reasonable alternatives for hospitalized medical patients who continue to
eat, but NPH insulin has peaks and valleys of action through the day that can present
management problems. Generally, glargine insulin is associated with less
symptomatic hypoglycemia than regimens including NPH insulin. Traditional sliding
scale insulin regimens, with post-hoc adjustments of glycemic control with regular
insulin are no longer an acceptable approach to glycemic control of hospitalized
patients. Peak action of regular insulin may not occur for 3 hours, and its effects last
for 8 or more hours. It is both difficult to catch up to hyperglycemia with slidingscale regular insulin and easy to accumulate serial doses of insulin sufficient to
precipitate symptomatic hypoglycemia.
Primary Care Medicine:Question 101
The correct answer is C
Educational Objectives
Diagnose and treat head lice.
Critique
Because they are relatively benign, head lice should generally be treated only when a
diagnosis is made. Thus, the patient should enlist another adult to perform a
systematic combing of her hair. The finding of live lice or egg capsules attached to
the hair shaft (flits) would warrant treatment with either permethrin or synergized
pyrethrin. Lindane can produce neurotoxicity and is not recommended. Follow-up
combing of the mother and daughter should be performed to ensure cure.
Laundering clothes, bedclothes, and hats, as well as washing brushes and combs is
sensible. Because other fomites likely play a minimal role disinfection of the
household is unnecessary.
Primary Care Medicine:Question 102
The correct answer is B
Educational Objectives
Identify dysplastic nevus syndrome.
Critique
This patient appears to have a single atypical mole in a sun-exposed area. The lesion
has features of dysplasia and is >6mm, and therefore requires biopsy. The lesion is
suspicious for dysplastic nevus or early melanoma arising de novo from previously
normal skin.
Although this patient has childhood sun exposure and numerous moles, she does not
have multiple atypical moles (dysplastic nevi) in sun-protected areas to suggest
dysplastic nevus syndrome. In the inherited form of dysplastic nevus syndrome
(autosomal-dominant transmission), family members may have history of
melanoma, multiple atypical moles, or both. This patients mothers history is not
convincing. Therefore, the patient does not require serial total body photography for
monitoring of many lesions in high-risk patients.
Avoidance of sun is important in persons at risk for any kind of skin cancer, although
the evidence for benefit of sunscreen for prevention of melanoma is modest and less
convincing than for nonmelanoma skin cancer. Sunscreens recommended for skin
cancer prevention are of potency higher than SPF30. It is prudent to avoid tanning
booths; however, data for tanning lamps as a risk factor for melanoma are

conflicting. This is not as important as biopsy of the worrisome lesion.


Primary Care Medicine:Question 103
The correct answer is C
Educational Objectives
Manage acute knee pain.
Critique
The acute onset of pain with warmth, erythema, and pain with passive movement
suggests synovitis. Typical causes include crystal-induced arthropathy and septic
arthritis. The joint should be aspirated and the fluid examined for crystals and
bacteria to rule out joint infection and rapidly confirm a suspicion of gout or calcium
pyrophosphate deposition (CPPD) disease. Low-dose salicylates can precipitate gout.
Rest, elevation, and use of a nonsteroidal anti-inflammatory medication would
alleviate gout and CPPD disease if the dosage of the analgesic were sufficient.
However, treating without confirming the diagnosis would not be optimal. In the
absence of trauma, radiographs will add little to the acute management of this
patient. Blood cultures may be positive in some cases of septic arthritis, but
examination and culture of the joint fluid is more likely to assist in confirming the
diagnosis. Allopurinol therapy should not be started during an acute attack of gout.
Primary Care Medicine:Question 104
The correct answer is B
Educational Objectives
Know the recommendations for immunizations before travel.
Critique
Hepatitis A is the most important vaccine and providing this will prevent the greatest
number of potentially vaccine-related illnesses. Titers require 2 to 5 weeks and
immunization 1 month before travel is adequate time. For travel to remote areas of
lesser developed nations, typhoid vaccination (either a live oral vaccine [Ty21a]
requiring four doses taken every other day on an empty stomach at least 2 weeks
prior to departure or a parenteral Vi capsular polysaccharide typhoid vaccine) and a
one time polio booster are recommended. The combined hepatitis A and hepatitis B
vaccination (Twinrix TM) provides immunization against both viruses; however, at
least two doses are necessary to provide adequate protection against hepatitis A. If
the patient could return for a second visit in 1 month this may be an option but
would likely be logistically difficult. If the visit occurred after graduation, the cost of
the vaccine would be borne by the student. During the visit the traveler did not
provide any reason to strongly advise he get a hepatitis B immunization. With all
infants getting universal hepatitis B vaccinations this will become a moot issue.
Primary Care Medicine:Question 105
The correct answer is B
Educational Objectives
Recognize the clinical presentation of ulnar nerve compression in the cubital tunnel
at the elbow.
Critique
Cubital tunnel syndrome is the second most common entrapment neuropathy in the
upper extremity and occurs with compression of the ulnar nerve as it passes through
the cubital tunnel at the medial elbow. The floor of the tunnel is formed by the
medial collateral ligament of the elbow, and the roof is formed by the two heads of
the flexor carpi ulnaris muscle with the associated connecting aponeurosis.
Compression can occur if the tunnel is congenitally narrowed, or with repetitive
elbow motions. Cubital tunnel syndrome typically presents with numbness or

paresthesias in the 4th and 5th digits and may have associated medial elbow pain. In
severe cases, hand clumsiness and reduced grip strength occur due to interosseous
muscle weakness.
Carpal tunnel syndrome affects the median nerve, causing numbness and/or
paresthesia in the thumb, index, and middle fingers and lateral half of the ring
finger. Patients with medial epicondylitis experience well-localized medial elbow pain
without radiation into the hand; elbow pain is exacerbated by lifting or use of the
forearm or wrist. Saturday night palsy occurs with compression of the radial nerve in
the spiral groove of the humerus. Ensuing symptoms include sensory loss over the
dorsum of the hand and weakness in the brachioradialis, wrist extensors, and finger
extensors. A C-7 radiculopathy, the most common form of cervical radiculopathy,
should cause sensory changes in digits 2 to 4, and motor weakness in the triceps,
forearm pronation, and wrist flexion and extension. Additionally, one might expect
symptoms to be exacerbated by neck movement.
Primary Care Medicine:Question 106
The correct answer is E
Educational Objectives
Manage sciatica without red flags.
Critique
This patient has no red flags except possible urinary problems, but has normal
sensory and rectal examinations, making cauda equina syndrome unlikely. This
patient has sciatica (-2% of all patients with back pain) involving the S1 nerve root
as demonstrated by his weak ankle jerk and dorsiflexion. The most cost-effective
method of caring for a patient in this situation is educational materials and
appropriate pain medication with bedrest and activity as tolerated for the next few
days. Although chiropractic care and/or physical therapy are slightly better than
traditional care and are preferred by patients for low-back pain, this patient has
sciatica for which neither mode of treatment has been shown to be effective. Indeed,
vigorous exercise programs or prolonged bed rest impedes recovery. Traction
therapy has not been shown to be effective for sciatica. Of note, the evidence for
corticosteroid injection or chymopapain injection suggests that, while corticosteroid
injections may provide pain relief in some patients, these treatments do not affect
intermediate or long-term outcomes and carry some risk.
Finally, although he may eventually need an MRI (-50% of asymptomatic patients in
this age group have a bulging or degenerated disc on MRI) or even surgery, at this
point he will likely recover with conservative care. In fact, 65% to 70% of patients
with sciatica recover in 12 weeks, and 10-year outcomes are almost identical (84%
recover), for return to work and neurologic findings, with or without surgery. A
recent, cohort study (Atlas et al) reported that at 5 years 19% of surgical patients
had at least one additional procedure and 16% of nonsurgical patients had opted for
surgery. Early relief from pain was reported by 70% of surgical vs 56% of
nonsurgical patients (P < 0.001) and more surgical patients were satisfied with their
condition (64% vs 46%, P < 0.001), but they were just as likely as nonsurgical
patients to be receiving disability.
Therefore, this patient should be managed with NSAIDs and perhaps muscle
relaxants or tricyclic antidepressants (for neuropathic pain) and if he is not improving
or has worsened after 4 weeks, referral should be considered. Restriction to lifting
<20 lbs is appropriate.

Primary Care Medicine:Question 107


The correct answer is D
Educational Objectives
Treat hot flushes.
Critique
Raloxifene may increase hot flushes; and sertraline will not affect her hot flushes
(and may induce sweats as a side effect). Soy is possibly effective at reducing hot
flushes, but patients must consume 25 g (soy milk contains about 5 g/8 oz).
Preliminary evidence suggests that black cohosh may be effective, but its safety is
unknown. Estrogen will certainly decrease her hot flushes. Risks according to
evidence from the Womens Health Initiative include an increased risk for breast
cancer (8/10000 person years) that appears at about 3 years. The risk of
thrombophlebitis (8/10000 person years) and cardiovascular disease (7/10000
person years) are also significant, and the risk appears to be highest in the first
year. This woman does not have significant risks for either category of disease and
so it would be reasonable to recommend estrogen therapy for 1 to 2 years.
Primary Care Medicine:Question 108
The correct answer is E
Educational Objectives
Recognize and treat tinea versicolor.
Critique
This patient has had recurrent episodes of rash involving the chest and back. This
rash has become hypopigmented in the past, a typical history for tinea versicolor.
The location involving the chest and back is also typical for tinea versicolor. Tinea
versicolor usually involves the chest, upper back, neck, and upper extremities.
Hypopigmented lesions occur in persons exposed to the sun because the causative
organism, Pityrosporum orbiculare, produces azelaic acid, which inhibits pigment
transfer to keratinocytes. Several treatments are effective. The most appropriate
therapy for this patient who has had frequent recurrences would be an oral
antifungal agent. Ketoconazole can be used either as 400 mg single-dose therapy or
as daily therapy for 1 to 2 weeks. Fluconazole in doses of 300 to 400 mg as a single
dose appears to have equal efficacy. Ketoconazole 2% shampoo is also an effective
therapy, and may be effective as a single-dose application. Selenium sulfidecontaining shampoos have traditionally been used as treatment, but require repeated
applications for efficacy. Antibiotics (amoxicillin or azithromycin) would not be
effective. Nystatin has a narrow spectrum, effective only for candidal infections.
Primary Care Medicine:Question 109
The correct answer is E
Educational Objectives
Understand that many discolored and deformed nails are not due to fungal infection.
Critique
This patient has had discolored nails for several years and has had no improvement
with oral itraconazole treatment. The most appropriate approach to this patients care
is to evaluate for the presence of fungal infection before prescribing any further
treatment. Up to 50% of patients with thickened, discolored nails have a nonfungal
cause. The likelihood of fungal infection as a cause for dystrophic nails increases with
age. In one recent study of a geriatric population, only 23% of the patients with
suspected onychomycosis had negative fungal cultures.

Primary Care Medicine:Question 110


The correct answer is C
Educational Objectives
Understand use of D-dimer in evaluation of venous thromboembolic disease.
Critique
This patient presents with pleuritic chest pain after a probable viral syndrome. Viral
pleurisy is the most likely diagnosis. Although, her hormone replacement therapy
puts her at increased risk of thromboembolic disease, the D-dimer is quite sensitive
for thromboembolism; therefore, it is most appropriate in ruling out the diagnosis
(its negative predictive value is high). However, as its specificity is low, a positive
test is not diagnostic. Given that the likelihood of pulmonary embolism is low, further
diagnostic testing is in order. Ventilation/perfusion scan would be the next
appropriate step. If this likelihood were higher, empiric anticoagulation might be
appropriate. In this case, transesophageal echocardiography would be most
appropriate in evaluating for aortic dissection. Pulmonary angiogram would typically
be reserved for higher risk patients with negative less-invasive evaluation.
Primary Care Medicine:Question 111
The correct answer is E
Educational Objectives
Understand that transtelephonic monitoring is a superior diagnostic test to
continuous loop monitoring.
Critique
This patient has palpitations that could be due to panic disorder or anxiety disorder,
but cardiac causes cannot yet be excluded. Although her electrocardiogram was
normal, she should have further testing to determine whether her symptoms are due
to a cardiac arrhythmia.
Transtelephonic monitors have a higher yield than continuous loop monitors and
should be the test of choice in patients who are physically and mentally well enough
to activate the monitor.
A cardiac cause should be considered before treatment for a psychiatric disorder.
Although the patient has symptoms that warrant further evaluation, they are not
severe enough to warrant immediate electrophysiologic studiesshe has not had
syncope or presyncope, and she does not have other risk factors for malignant
arrhythmias.
Primary Care Med icine:Question 112
The correct answer is B
Educational Objectives
Recognize dental abnormality as sign of bulimia.
Critique
Erosions on the inner surfaces of teeth signify that excess stomach acid has
frequently come into contact with the enamel and has eroded it. This finding is
consistent with a diagnosis of bulimia. The next step in the care of this patient is to
ask her about a current or past history of self-induced vomiting.
This patient gets regular dental care, and erosions of enamel are a chronic condition,
not a dental emergency. This patient does not require immediate dental referral.
Tooth grinding at night, or bruxism, causes shortened height of rear teeth, and is not
associated with problems of the enamel. Although severe, recurrent, chronic reflux
can cause changes in the enamel, also on the basis of contact of the enamel with
stomach acid, it would not be appropriate to treat empirically for reflux disease on

the basis of the tooth findings alone. In addition, it would be quite unusual for an 18year-old to have changes in her teeth from reflux in the absence of a clinical history
or symptoms. Fluoride exposure in young teeth would not result in erosions of the
enamel.
Primary Care Medicine:Question 113
The correct answer is A
Educational Objectives
Manage long-term warfarin anticoagulation in a patient with potential warfarin
sensitivity.
Critique
A rise in the INR at 48 hours signals high sensitivity to warfarin, which can be due to
other medications or to an underlying polymorphism in the cytochrome P-450
system. Older patients are also more sensitive. The standard maintenance dose of
warfarin is 4 to 5 mg/d, but this patient would likely require only 2 to 4 mg/d. After
this change in dosage, the INR should again be checked in 2 to 3 days.
Primary Care Medicine:Question 114
The correct answer is A
Educational Objectives
Understand management strategies of melanoma based on depth of lesion.
Critique
The depth of melanoma is an important feature in determining optimal management
and prognosis. The case described had a tumor depth of 0.88 mm by Breslow
oculometer, and is a stage II lesion (0.76-1 .49 mm depth: 87-94% 5-year survival).
Re-excision of the primary site is recommended with a 2-cm margin, as randomized
trials have shown similar local recurrence and overall survival rates with a narrow 2cm re-excision margin compared to a wide margin of 4 to 5 cm that is more
disfiguring and more likely to require skin grafting.
Sentinel lymph node detection is not generally indicated for thin melanomas, defined
as those <1 mm thick, which have a small chance of metastasizing. Sentinel lymph
node detection by blue dye or scanning is now commonly used for intermediate (1 .0
to 4.0 mm) or thick (>4 mm) melanomas. If there is a positive sentinel lymph node,
lymph node dissection is performed. Elective lymph node dissection is not usually
performed without clinical evidence of metastases, as several randomized trials have
not shown a benefit and the procedure has a significant rate of complications.
Primary Care Medicine:Question 115
The correct answer is D
Educational Objectives
Treat mild, intermittent symptoms of carpal tunnel syndrome.
Critique
The mainstay of treatment for intermittent, classic carpal tunnel symptoms, as this
patient has, is wrist splints, with the optional addition of anti-inflammatory
medication. Electromyography is reserved for patients who do not respond to such
conservative therapy, or to confirm the diagnosis in patients with atypical,
persistent, or severe symptoms who may require surgery. Surgical release of the
radial nerve will not help with carpal tunnel as carpal tunnel affects the median
nerve, not the radial nerve. Median nerve release is reserved for patients with
severe, persistent symptoms, or progression during conservative therapy. Although

MRI may reveal space-occupying lesion in patients with carpal tunnel syndrome,
such lesions are very rare and MRI is expensive and currently is not used in the
diagnosis of carpal tunnel syndrome, especially for such a patient with mild and
classic symptoms potentially reversible with a simple trial of conservative therapy.
Vitamin has been shown to have no benefit for patients with carpal tunnel syndrome.
Primary Care Medicine:Question 116
The correct answer is C
Educational Objectives
Recognize the signs of systemic disease associated with carpel tunnel syndrome.
Critique
Between one fifth and two thirds of patients with occupationally related carpal tunnel
syndrome were also found to have concurrent medical conditions, most commonly
diabetes mellitus, thyroid disease, and obesity. Although concurrent medical
conditions may already be known at the time of diagnosis of the carpal tunnel
syndrome, a careful review of symptoms should be performed in patients with carpal
tunnel syndrome. Bilateral carpal tunnel syndrome makes a systemic disease
somewhat more likely. In addition, recurrent, or persistent symptoms despite
therapy, or a review of symptoms suggesting pregnancy, rheumatoid arthritis,
systemic lupus erythematosus, acromegaly, myeloma, amyloidosis, diabetes
mellitus, or thyroid disease are also concerning for an associated systemic disorder.
Neither the presence or absence of Phalens or Tinels signs, nor the results of
electromyography testing have any utility in predicting presence of systemic disease.
Primary Care Medicine:Question 117
The correct answer is D
Educational Objectives
Treat vaginal symptoms in a patient who has had breast cancer.
Critique
A vaginal estrogen ring (Estring) is a ring about the size of a small diaphragm, that
slowly releases estrogen such that local symptoms are improved, but serum estrogen
levels are not changed. However, independent randomized clinical trials have not
been done. Vaginal lubricants might be effective for this patients dyspareunia, but
local estrogen therapy is likely to be more effective. The old standard, vaginal
estrogen cream, results in detectable serum levels, which may be a risk in a patient
with invasive breast cancer. Most patients also find vaginal estrogen cream messy
and inconvenient, and patients overwhelmingly prefer the vaginal ring, which is left
in place for 3 months. Increasing her tamoxifen dosage or beginning raloxifene
would not be effective or appropriate.
Primary Care Medicine:Question 118
The correct answer is D
Educational Objectives
Recognize a sudden varicocele and hydrocele as a possible harbinger of renal cell
carcinoma.
Critique
This man presents with a sudden onset new hydrocele and varicocele on the left.
Virtually all varicoceles are on the left, but appearance of a new varicocele in a man
older than 50 years, that does not reduce when the patient lies down, suggests
obstruction of the venous drainage. Since the left testicular vein drains into the left

renal vein (up to 11% of men with renal cell carcinoma have a significant varicocele
on the left), an abdominal CT should be obtained to evaluate the patient for renal cell
carcinoma. Similarly, since the right testicular vein drains into the inferior vena cava
and right varicoceles are much less common, appearance of a new right,
nondraining, varicocele should arouse suspicion for a right renal cell cancer or other
cause of vena cava obstruction.
Primary Care Medicine:Question 119
The correct answer is B
Educational Objectives
Manage warfarin drug interaction.
Critique
The single drug involved in the greatest number of serious drug interactions is
warfarin. Among the many medications that interfere with warfarin metabolism and
prolong the INR are COX-2 inhibitors, although it this case that effect is more
profound than usually observed. For that reason, it would be helpful to review other
medications that the patient might be taking, in particular Dong Quai, which contains
several natural warfarin derivatives.
The response to an elevated INR depends on its magnitude and the risk of bleeding.
For minor prolongation of the INR up to 5.0, it is usually sufficient to lower the
average dose by 5% to 20%. INR values between 5.0 and 9.0 in the absence of
active bleeding are typically managed by withholding warfarin and restarting at a
lower dose. If the patients risk of bleeding is high or the INR continues to rise after
stopping warfarin, vitamin K, 1.0 to 2.5 mg, should be administered orally. Patients
with INR values over 9.0 to 10.0 should usually receive vitamin K. Those who are
actively bleeding or have severely elevated INRs, should be hospitalized for
administration of higher doses of vitamin K in addition to fresh plasma or
prothrombin complex as required. In this case, the very high INR and history of
gastrointestinal bleeding place the patient at high risk and warrant administration of
vitamin K in a dose that is low enough not to reverse anticoagulation and risk
thrombosis of the mechanical valve.
Primary Care Med icine:Question 120
The correct answer is B
Educational Objectives
Recognize and treat benign prostatic hypertrophy as a cause of hematuria.
Critique
Several prospective trials have shown significant decreases in hematuria and
decreased need for surgery in patients with benign prostatic hypertrophy-hematuria.
A prospective study randomized 42 patients to finasteride, cyproterone acetate, or
watchful waiting and showed a significant reduction in bleeding at 9 and 12 months
in both the groups treated with finasteride and cyproterone acetate compared with
control (no difference between the two), suggesting that at least these two
antiandrogens can control bleeding in benign prostatic hypertrophy. One
retrospective review of patients treated with finasteride concluded that finasteride, 5
mg/d, decreases gross hematuria in 94% of patients with benign prostatic
hypertrophy-related hematuria, including patients taking anticoagulants, so that in
this patient, continuing his warfarin and starting finasteride is a reasonable
approach.

Primary Care Med icine:Question 121


The correct answer is C
Educational Objectives
Know the indications for the use of antibiotics for the treatment of acute pharyngitis
in adults.
Critique
The pretest probability of group A hemolytic Streptococcus (GABHS) pharyngitis in
an unselected office-based adult setting is 5% to 10%. The probability of GABHS
increases in the presence of four clinical findings: history of fever, tonsillar exudates,
tender anterior cervical lymphadenopathy, and absence of cough. Patients with none
or one of the cardinal findings have a very low risk of GABHS and do not require
testing or antibiotics. Patients with all four features can be treated with antibiotics
without additional testing. This patient has one of the four features. Further testing is
necessary before initiating antibiotic treatment for this adult who is in an
intermediate category. Antibiotic therapy in this case is based on the results of rapid
antigen testing. A 10-day course of penicillin is the regimen of choice for GABHS. A
5-day course of azithromycin achieves similar eradication rates but increases the
chance of antibiotic resistance.

Vous aimerez peut-être aussi